Download as pdf or txt
Download as pdf or txt
You are on page 1of 113

INDEX

TM

PAST YEARS
BOARD PAPERS + SAMPLE PAPERS
CLASS - 10

2024 Syllabus IMP Terms & Formulas

2024 Paper Design Past Years Question Paper


2015 (Delhi)
CBSE Official SQP 2024 2015 (Outside Delhi)
2023 (Series WX1YZ/4)
SQP 1 (Fully Solved) 2023 (Series WX1YZ/6)

The Only Book which has no out of syllabus questions

The only book which has no out of syllabus questions


Note
This PDF is for sample purposes only! Our paperback edition contains
10+1 SQPs and 7+ years PYQs for a better preparation + revision.

Click here to access full book


ALL RIGHTS RESERVED
• No part of this book may be reproduced or copied in any form or any means without the written
permission of the publishers.
• The publishers have taken all possible precautions in publishing this book, yet if any mistake has crept
in, the publishers shall not be responsible for the same.
• All disputes shall be subject to the jurisdiction of court at Delhi only.

Published by:

C-8, Sector 6, Noida, Uttar Pradesh-201 301


Phone: +91-99100 55604, 78400 40400
E-mail: info@shivdas.in • www.shivdas.in
Trade contact: sales@shivdas.in

© Publishers

Printed at:
Ashok Printing Press
Industrial Area, Bawana
Delhi-110 039

Click here to access full book!


Contents
☛ Latest Syllabus (2024 EXAMINATION) ... (iv)
☛ Latest CBSE Question Paper Design (STANDARD) ... (vii)
(Issued by CBSE for 2024)
☛ CBSE Sample Question Paper—2024 (Fully Solved) ... P-1
☛ Sample Question Papers (1 to 5) (Fully Solved) ... P-17
☛ Important Terms & Formulae (Chapterwise) ... CH-1
☛ Past Years’ CBSE Examination Papers (Fully Solved)

2015 (Delhi) ... ... 1

2015 (Outside Delhi) ... ... 9

2016 (Delhi) ... ... 18

2016 (Outside Delhi) ... ... 28

2017 (Delhi) ... ... 35

2017 (Outside Delhi) ... ... 44

2018 (Common for Delhi & Outside Delhi) ... ... 52

2018 Comptt. (Common for Delhi & Outside Delhi) ... ... 60

2019 (Series: JMS/1) ... ... 69

2019 (Series: JMS/4) ... ... 81

2020 (Series: JBB/1) ... ... 93

2020 (Series: JBB/3) ... ... 105

2021 (Term-I) ... ... 119

2022 (Term-II) ... ... 131

2023 (Series: WX1YZ/4) ... ... 139

2023 (Series: WX1YZ/6) ... ... 158

❑❑❑❑

(iii)

Click here to access full book!


Syllabus
Latest Syllabus Issued by CBSE (31st March 2023)
2023-24
MATHEMATICS (041)
Course Structure—Class X
Units Unit Name Marks
I. Number Systems 06
II. Algebra 20
III. Coordinate Geometry 06
IV. Geometry 15
V. Trigonometry 12
VI. Mensuration 10
VII. Statistics & Probability 11
Total 80

UNIT I: NUMBER SYSTEMS


1. Real Numbers (15 Periods)
Fundamental Theorem of Arithmetic—statements after reviewing work
done earlier and after illustrating and motivating through examples, Proofs
of irrationality of 2 , 3 , 5 .
UNIT II: ALGEBRA
1. Polynomials(8 Periods)
Zeros of a polynomial. Relationship between zeros and coefficients of
quadratic polynomials.
2. Pair of Linear Equations in Two Variables (15 Periods)
Pair of linear equations in two variables and graphical method of their
solution, consistency/inconsistency.
Algebraic conditions for number of solutions. Solution of a pair of linear
equations in two variables algebraically—by substitution, by elimination.
Simple situational problems.

(iv)

Click here to access full book!


3. Quadratic Equations (15 Periods)
Standard form of a quadratic equation ax + bx + c = 0, (a  0). Solutions of
2

quadratic equations (only real roots) by factorization, and by using quadratic


formula. Relationship between discriminant and nature of roots.
Situational problems based on quadratic equations related to day-to-day
activities to be incorporated.
4. Arithmetic Progressions (10 Periods)
Motivation for studying Arithmetic Progression Derivation of the nth term
and Sum of the first n terms of A.P. and their application in solving daily
life problems.

UNIT III: COORDINATE GEOMETRY


1. Coordinate Geometry (15 Periods)
Review: Concepts of Coordinate geometry, Graphs of linear equations.
Distance formula. Section formula (internal division).

UNIT IV: GEOMETRY


1. Triangles (15 Periods)
Definitions, examples, counter examples of similar triangles.
1. (Prove) If a line is drawn parallel to one side of a triangle to intersect
the other two sides in distinct points, the other two sides are divided in
the same ratio.
2. (Motivate) If a line divides two sides of a triangle in the same ratio, the
line is parallel to the third side.
3. (Motivate) If in two triangles, the corresponding angles are equal, their
corresponding sides are proportional and the triangles are similar.
4. (Motivate) If the corresponding sides of two triangles are proportional,
their corresponding angles are equal and the two triangles are similar.
5. (Motivate) If one angle of a triangle is equal to one angle of another
triangle and the sides including these angles are proportional, the two
triangles are similar.
2. Circles(10 Periods)
Tangent to a Circle at, point of contact.
1. (Prove) The tangent at any point of a circle is perpendicular to the radius
through the point of contact.
2. (Prove) The lengths of tangents drawn from an external point to a circle
are equal.

(v)

Click here to access full book!


UNIT V: TRIGONOMETRY
1. Introduction to Trigonometry (10 Periods)
Trigonometric ratios of an acute angle of a right-angled triangle. Proof of
their existence (well defined); Motivate the ratios whichever are defined at 0º
and 90º. Values of the trigonometric ratios of 30º, 45º and 60º. Relationships
between the ratios.
2. Trigonometric Identities (15 Periods)
Proof and applications of the identity sin2A + cos2A = 1. Only simple
identities to be given.
3. Heights and Distances: Angle of elevation, Angle of Depression.(10
Periods)
Simple problems on heights and distances. Problems should not involve
more than two right triangles. Angles of elevation/depression should be
only 30°, 45°, and 60°.

UNIT VI: MENSURATION


1. Areas Related to Circles (12 Periods)
Area of sectors and segments of a circle. Problems based on areas and
perimeter/ circumference of the above said plane figures. (In calculating
area of segment of a circle, problems should be restricted to central angle of
60°, 90° and 120° only.)
2. Surface Areas and Volumes (12 Periods)
Surface areas and volumes of combinations of any two of the following:
cubes, cuboids, spheres, hemispheres and right circular cylinders/cones.

UNIT VII: STATISTICS AND PROBABILITY


1. Statistics(18 Periods)
Mean, Median and Mode of grouped data (bimodal situation to be avoided).
2. Probability (10 Periods)
Classical definition of Probability. Simple problems on finding the probability
of an event.

❑❑❑❑

(vi)

Click here to access full book!


Latest CBSE Question Paper Design
MATHEMATICS : STANDARD
CLASS X
(2024 EXAMINATION)
Time: 3 Hours Maximum Marks: 80

S.No. Typology of Questions Total Marks % Weightage


1. Remembering : Exhibit memory of previously learned
material by recalling facts, terms, basic concepts, and
answers.
Understanding: Demonstrate understanding of facts 43 54
and ideas by organizing, comparing, translating,
interpreting, giving descriptions, and stating main
ideas.
2. Applying : Solve problems to new situations by
applying acquired knowledge, facts, techniques and 19 24
rules in a different way.
3. Analysing: Examine and break information into parts
by identifying motives or causes. Make inferences and
find evidence to support generalizations.
Evaluating: Present and defend opinions by making
judgments about information, validity of ideas, or 18 22
quality of work based on a set of criteria.
Creating: Compile information together in a different
way by combining elements in a new pattern or
proposing alternative solutions.
Total 80 100

Internal Assessment 20 Marks


• Pen Paper Test and Multiple Assessment (5+5) 10 Marks
• Portfolio 05 Marks
• Lab Practical (Lab activities to be done from the prescribed books)
05 Marks
❑❑❑❑

(vii)

Click here to access full book!


For more
Sample Question Papers
Kindly SCAN

(viii)

Click here to access full book!


 Code No. 041
Roll No.  Candidates must write the Q.P. Code
on the title page of the answer-book.

• Please check that this question paper contains 7 printed pages.


• Q.P. Code given on the right hand side of the question paper should be written
on the title page of the answer-book by the candidate.
• Please check that this question paper contains 38 questions.
• Please write down the Serial Number of the question in the answer-book
before attempting it.
• 15 minute time has been allotted to read this question paper. The question
paper will be distributed at 10.15 a.m. From 10.15 a.m. to 10.30 a.m., the
candidates will read the question paper only and will not write any answer
on the answer-book during this period.

MATHEMATICS (Standard) –X

a s
iv D
SAMPLE QUESTION PAPER

Sh
Issued by CBSE for 2024 Examination
Time Allowed : 3 hours Maximum Marks : 80
GENERAL INSTRUCTIONS:
(i) This questions paper has 5 Sections —A, B, C, D and E.
(ii) Section A has 20 MCQs carrying 1 mark each.
(iii) Section B has 5 questions carrying 02 marks each.
(iv) Section C has 6 questions carrying 03 marks each.
(v) Section D has 4 questions carrying 05 marks each.
(vi) Section E has 3 case based integrated units of assessment (04 marks each) with sub-parts of the
values of 1, 1 and 2 marks each respectively.
(vii) All Questions are compulsory. However, an internal choice in 2 Questions of 5 marks, 2 Questions
of 3 marks and 2 Questions of 2 marks has been provided. An internal choice has been provided in
the 2 marks Questions of Section E.
22
(viii) Draw neat figures wherever required. Take p = 7 wherever required, if not stated.

Section-A
Section A consists of 20 questions of 1 mark each.
Q.1. If two positive integers a and b are written as a = x3y2 and b = xy3, where x, y are prime
numbers, then the result obtained by dividing the product of the positive integers by the
LCM (a, b) is 1
(a) xy (b) xy2 (c) x3y3 (d) x2y2

MATHEMATICS – CBSE SAMPLE QUESTION PAPER–2024  P-1

Click here to access full book!


Q.2. The given linear polynomial y = f(x) has 1
y

5
4 (0, 4)
3
2
1 )
,0
x
(3 x
–4 –3 –2 –1 0 1 2 3 4 5
–1
–2
–3

y

(a) 2 zeroes (b) 1 zero and the zero is ‘3’


(c) 1 zero and the zero is ‘4’ (d) No zeroes
Q.3. The given pair of linear equations is non-intersecting. Which of the following statements
is true? 1

a s
iv D
y

Sh
0
03 =
= c2
c1 y+
y+
2 b2
b1 +
x+
1 a 2x
a1
x′ x
–5 –4 –3 –2 –1 0 1 2 3 4
–1
–2
–3
–4

y′

a1 b1 c1 a1 1 b 1 c 1 1 a 1 b c a1 b1 c1
(a) = = (b) a = b ≠ c (c) a ≠ b = c (d) a ≠ b ≠ c
a2 b2 c 2 2 2 2 2 2 2 2 2 2

Q.4. Write the nature of roots of the quadratic equation 9x2 – 6x – 2 = 0. 1
(a) No real roots (b) 2 equal real roots
(c) 2 distinct real roots (d) More than 2 real roots
Q.5. Two APs have the same common difference. The first term of one of these is –1 and that
of the other is –8. Then the difference between their 4th terms is 1
(a) 1 (b) –7 (c) 7 (d) 9
Q.6. Find the ratio in which the line segment joining (2,–3) and (5, 6) is divided by x-axis.1
(a) 1 : 2 (b) 2 : 1 (c) 2 : 5 (d) 5 : 2
Q.7. (x,y) is 5 units from the origin. How many such points lie in the third quadrant? 1
(a) 0 (b) 1 (c) 2 (d) infinitely many

P-2  Shiv Das CBSE SECONDARY SERIES (X)

Click here to access full book!


Q.8. In ∆ABC, DE || AB. If AB = a, DE = x, BE = b and EC = c. A
Express x in terms of a, b and c.1
ac ac D
(a) b (b) b + c
ab ab
(c) c (d) b + c C
B E
P R
Q.9. If O is centre of a circle and Chord PQ makes an angle
50° with the tangent PR at the point of contact P, find the
angle made by the chord at the centre. 1 Q
O
(a) 130º (b) 100º
(c) 50º (d) 30º
Q.10. A Quadrilateral PQRS is drawn to circumscribe a circle.
If PQ = 12 cm, QR = 15 cm and RS = 14 cm, find the length of SP. 1
(a) 15 cm (b) 14 cm (c) 12 cm (d) 11 cm
a
Q.11. Given that sin θ = b , find cos θ.1
b b b 2 − a2 a
(a) b 2 − a2
(b) a (c) (d) b 2 − a2
b

s
Q.12. (sec A + tan A) (1 – sin A) = 1

a
(a) sec A (b) sin A (c) cosec A (d) cos A

iv D
Q.13. A pole 6 m high casts a shadow 2 3 m long on the ground, then the Sun’s elevation is 1

Sh
(a) 60º (b) 45º (c) 30º (d) 90º
Q.14. If the perimeter and the area of a circle are numerically equal, then the radius of the circle
is  1
(a) 2 units (b) p units (c) 4 units (d) 7 units
Q.15. It is proposed to build a single circular park equal in area to the sum of areas of two
circular parks of diameters 16 m and 12 m in a locality. The radius of the new park is 1
(a) 10 m (b) 15 m (c) 20 m (d) 24 m
Q.16. There is a green square board of side ‘2a’ units circumscribing a red circle. Jayadev is
asked to keep a dot on the above said board. Find the probability that he keeps the dot
on the green region. 1
π 4
(a) 4 (b) 4 − π (c) π − 4 (d) π
4 4
Q.17. 2 cards of hearts and 4 cards of spades are missing from a pack of 52 cards. What is the
probability of getting a black card from the remaining pack?  1
22 24 24
(a) 22 (b) 46 (c) 52 (d) 46
52
Q.18. Find the upper limit of the modal class from the given distribution. 1

Height (in cm) Below 140 Below 145 Below 150 Below 155 Below 160 Below 165
Number of girls 4 11 29 40 46 51
(a) 165 (b) 160 (c) 155 (d) 150

MATHEMATICS – CBSE SAMPLE QUESTION PAPER–2024  P-3

Click here to access full book!


DIRECTION: In the Questions number 19 and 20, a statement of assertion (A) is followed by a
statement of Reason (R). Choose the correct option.
(a) Both A and R are true and R is the correct explanation of A.
(b) Both A and R are true and R is not the correct explanation of A.
(c) A is true but R is false.
(d) A is False but R is true.
Q.19. Statement (A): Total Surface area of the top is the sum of the curved surface
area of the hemisphere and the curved surface area of the cone.  1
Statement (R): Top is obtained by fixing the plane surfaces of the hemisphere
and cone together.

Q.20. Statement (A): –5, −5 , 0, 5 ,…. is in Arithmetic Progression.  1


2 2
Statement (R): The terms of an Arithmetic Progression cannot have both positive and

negative rational numbers.
Section-B
Section B consists of 5 questions of 2 marks each.

Q.21. Prove that 2 is an irrational number. 2

a s
D Q C

i v D
Q.22. ABCD is a parallelogram. Point P divides AB in the ratio 2 : 3

h
O
and point Q divides DC in the ratio 4 : 1. Prove that OC is half

S
of OA. 2
A B
P

Q.23. From an external point P, two tangents, PA and PB are drawn to a circle with centre O. At a
point E on the circle, a tangent is drawn to intersect PA and PB at C and D, respectively. If
PA = 10 cm, find the perimeter of ∆PCD. 2
A
C
O
P
E

B D

1
Q.24. If tan (A + B) =3 and tan (A – B) = ; 0º < A + B < 90º; A > B, find A and B. 2
3
Or
Find the value of x.
2 cosec2 30° + x sin2 60° – 3 tan2 30° = 10
4
Q.25. With vertices A, B and C of ΔABC as centres, arcs are drawn with radii 14 cm and the three
portions of the triangle so obtained are removed. Find the total area removed from the
triangle. 2
Or

P-4  Shiv Das CBSE SECONDARY SERIES (X)

Click here to access full book!


Find the area of the unshaded region shown in the given figure.

14 cm

3 cm

3 cm

14 cm
3 cm

3 cm

Section-C
Section C consists of 6 questions of 3 marks each.
Q.26. National Art convention got registrations from students from all parts of the country, of
which 60 are interested in music, 84 are interested in dance and 108 students are interested
in handicrafts. For optimum cultural exchange, organisers wish to keep them in minimum
number of groups such that each group consists of students interested in the same artform

s
and the number of students in each group is the same. Find the number of students in each

iv D a
group. Find the number of groups in each artform. How many rooms are required if each
group is allotted a room? 3

Sh
Q.27. If a, β are zeroes of quadratic polynomial 5x2 + 5x + 1, find the value of 3
2
1. a + b 2 –1
2. a + b –1

Q.28. The sum of a two-digit number and the number obtained by reversing the digits is 66. If the
digits of the number differ by 2, find the number. How many such numbers are there? 3
Or
2 3 4 9
Solve : + = 2; − = –1
x y x y
A
Q.29. PA and PB are tangents drawn to a circle of centre O from
an external point P. Chord AB makes an angle of 30° with
the radius at the point of contact. O P
If length of the chord is 6 cm, find the length of the tangent
PA and the length of the radius OA. 3
B
Or
Two tangents TP and TQ are drawn to a circle with centre O from an external point T. Prove
that ∠PTQ = 2 ∠OPQ
1
Q.30. If 1 + sin2 θ = 3 sin θ . cos θ , then prove that tan θ = 1 or 2 . 3
Q.31. The length of 40 leaves of a plant are measured correct to nearest millimetre and the data
obtained is represented in the following table: 3
Length (in mm) 118-126 127-135 136-144 145-153 154-162 163-171 172-180
Number of leaves 3 5 9 12 5 4 2
Find the average length of the leaves.

MATHEMATICS – CBSE SAMPLE QUESTION PAPER–2024  P-5

Click here to access full book!


Section-D
Section D Consists of Long Answer (LA) type 4 questions of 5 marks each.
Q.32. A motor boat whose speed is 18 km/h in still water takes 1 hr. more to go 24 km upstream
than to return downstream to the same spot. Find the speed of stream. 
Or
3
Two water taps together can fill a tank in 9 8 hours. The tap of larger diameter takes 10
hours less than the smaller one to fill the tank separately. Find the time in which each tap
can separately fill the tank. 

Q.33. (a) State and prove Basic Proportionality theorem.  5


Or A


(b) In the given figure ∠CEF = ∠CFE. F is the midpoint of DC.
AB AE E
Prove that BD = FD D
F C

s
Q.34. Water is flowing at the rate of 15 km/h through a pipe of diameter 14 cm into a cuboidal

iv D a
pond which is 50 m long and 44 m wide. In what time will the level of water in pond rise by
21 cm?

Sh
What should be the speed of water if the rise in water level is to be attained in 1 hour? 5
Or
A tent is in the shape of a cylinder surmounted by a conical top. If the height and radius
of the cylindrical part are 3 m and 14 m respectively, and the total height of the tent is
13.5 m, find the area of the canvas required for making the tent, keeping a provision of
26 m2 of canvas for stitching and wastage. Also, find the cost of the canvas to be purchased
at the rate of `500 per m2.
Q.35. The median of the following data is 50. Find the values of ‘p’ and ‘q’, if the sum of all
frequencies is 90. Also find the mode. 5
Marks obtained 20–30 30–40 40–50 50–60 60–70 70–80 80–90
Number of students p 15 25 20 q 8 10

Section-E
Q.36. Manpreet Kaur is the national record holder for women in
the shot-put discipline. Her throw of 18.86 m at the Asian
Grand Prix in 2017 is the biggest distance for an Indian
female athlete. Keeping her as a role model, Sanjitha is
determined to earn gold in Olympics one day. Initially her
throw reached 7.56 m only. Being an athlete in school, she
regularly practiced both in the mornings and in the
evenings and was able to improve the distance by 9 cm
every week. During the special camp for 15 days, she
started with 40 throws and every day kept increasing the
number of throws by 12 to achieve this remarkable
progress.

P-6  Shiv Das CBSE SECONDARY SERIES (X)

Click here to access full book!


Based on the above information, answer the following questions :
(i) How many throws Sanjitha practiced on 11th day of the camp? 1
(ii) What would be Sanjitha’s throw distance at the end of week 6? 2
Or
When will she be able to achieve a throw of 11.16 m?
(iii) How many throws did she do during the entire camp of 15 days ? 1
Q.37. Tharunya was thrilled to know that the football tournament is fixed with a monthly
timeframe from 20th July to 20th August 2023 and for the first time in the FIFA Women’s
World Cup’s history, two nations host in 10 venues. Her father felt that the game can be
better understood if the position of players is represented as points on a coordinate plane.

J
FULL-BACK
D FORWARD C
I
CENTRAL
G CENTRE-BACK MIDFIELDER B E
GOALKEEPER A ATTACKING STRIKER
MIDFIELDER
H DEFENSIVE
SWEEPER MIDFIELDER

s
K

a
F

iv D
WING-BACK WINGER

Based on the above information, answer the following questions :

Sh
(i) At an instance, the midfielders and forward formed a parallelogram. Find the
position of the central midfielder (D) if the position of other players who formed
the parallelogram are :- A(1,2), B(4,3) and C(6,6) 1
(ii) Check if the Goal Keeper G(-3,5), Sweeper H(3,1) and Wing-back K(0,3) fall on a
same straight line. 2
Or, Check if the Full-back J(5, –3) and centre-back I(–4, 6) are equidistant from forward
C(0, 1) and if C is the mid-point of IJ.
(iii) If Defensive midfielder A(1, 4), Attacking midfielder B(2, –3) and Striker E(a, b) lie
on the same straight line and B is equidistant from A and E, find the position of E.1
Q.38. One evening, Kaushik was in a park. Children were playing cricket. Birds were singing on
a nearby tree of height 80 m. He observed Ball
F Bird A D Bird
a bird on the tree at an angle of elevation
of 45°. When a sixer was hit, a ball flew
through the tree frightening the bird to Height
Tree
fly away. In 2 seconds, he observed the 60º 80 m 80 m
bird flying at the same height at an angle
of elevation of 30° and the ball flying 45
º

towards him at the same height at an angle 30º Ground level


C G B E
of elevation of 60°.
Based on the above information, answer the following questions :
(i) At what distance from the foot of the tree did he observe the bird sitting on the tree? 1
(ii) How far did the bird fly in the mentioned time? 2
Or, After hitting the tree, how far did the ball travel in the sky when Kaushik saw the ball?
(iii) What is the speed of the bird in m/min if it had flown 20( 3 + 1) m? 1
]••••]
MATHEMATICS – CBSE SAMPLE QUESTION PAPER–2024  P-7

Click here to access full book!


SOLUTIONS

Section-A 8. (b) A

1. (b) Given. a = x3y2


b = xy3 a
D

L.C.M. (a, b) = x3y3 x

ab ( x 3 y 2 )( xy 3 )
A.T.Q, H.C.F. (a, b) = LCM( a , b ) = x3y 3 B b E c C

ab ( x 3 y 2 )( xy 3 ) In ∆DEC and ∆ABC 


= = xy2
LCM( a , b ) = x3y 3 ∠C = ∠C  ...[Common
∠DEC = ∠ABC  ...[Corresponding angles
2. (b) Line intersects x-axis at one point i.e.,
DDEC∼DABC  ...[AA Similarity criterion
at x = 3.
DE EC
3. (b) As the given lines are Parallel ⇒ AB = BC ...[CPST
a b c x c
so, 1 = 1 ≠ 1 ⇒ a = b + c
a2 b2 c 2
4. (c) Given. 9x2 – 6x – 2 = 0 ac
\ x = b + c
a = 9, b = –6, c = –2 

s
Now, D = b2 – 4ac 9. (b) P R

a
= (–6)2 – 4(9) (–2)

D
50
º

v
= 36 + 72 = 108 > 0

i
O

h
\ Roots are real and unequal. Q

S
5. (c) Let A = 1st term of 1st AP = –1,
a = 1st term of 2nd AP = –8
d = Common difference of both APs. ∠QPR = 50º  ...[Given
\ Difference between 4th terms = A4 – a4 ∠OPR = 90º  ...[Tangent is ⊥ to the radius
 through the point of contact
= (A + 3d) – (a + 3d) Now, ∠QPR + ∠OPQ = 90º
= –1 – (–8) ⇒ 50º + ∠OPQ = 90º
= –1 + 8 = 7 \ ∠OPQ = 90º – 50º = 40º
6. (a) Let C(x, 0) be any point on x-axis. OP = OQ ...[Radius of circle
(2, –3) k (x, 0) 1 (5, 6) ∠OQP = ∠OPQ = 40º  ...(i)
In DPOQ,
A C B
∠POQ + ∠OQP + ∠OPQ = 180º
Let AC = CB = k : 1  ...[Angle-sum-property of a D
6 k + ( −3)
Using section formula we get, =0 ⇒ ∠POQ + 40º + 40º = 180º ...[From (i)
k+1
\ ∠POQ = 180º – 80º = 100º
⇒ 6k – 3 = 0
3 1 S 14 cm R
\ k = 6 = 2 10. (d) QR + SP = PQ + RS
\ Required Ratio = 1 : 1 or 1 : 2 ⇒ 15 + SP = 12 + 14
15 cm

2
\ SP = 26 – 15 = 11 cm
y

7. (d) Let P(x, y), O (0,0)


Given. OP = 5 units P 12 cm Q

OP2 = 25 x’ –5 11. (c) As we know, sin2 q + cos2 q = 1


O x
A 5 units
⇒ (x – 0)2 + (y – 0)2 = 25 5 units
⇒ cos2 q = 1 – sin2 q
⇒ x2 + y2 = 25
(b)
–5 2
⇒ cos2 q = 1 – a 
B
a (given)
\ All Points lie on the arc AB. ... sin θ =
y’  b

P-8  Shiv Das CBSE SECONDARY SERIES (X)

Click here to access full book!


b 2 − a2 Area of Square (excluding circle) = 4a2 – pa2
⇒ cos2 q =  ...[Given = (4 – p) a2
b2
\ P (dot on the green region)
b 2 − a2 b2 - a2
\ cos2 q = =
b Area of Square (excluding circle)
b2  = Area of Square
2nd Method ( 4 − π )a 2 4 − π
a P = =
Given. sin2 q = b = H 4 a2 4
bk
Let P = ak, H = bk
ak 17. (b) Total Cards = 52 – 2 – 4 = 46
P2 + B2 = H2 θ Black Cards = 26 – 4 = 22
⇒ a2 k2 + B2 = b2k2 22 11
\ P (a black card) = 46 or 23
⇒ B2 = b2k2 – a2k2
2 2 2 2 2 18. (d)
Now, B = k (b − a ) = k b − a
B k b 2 − a2 b2 − a2 Height No. of
\ cos q = H = = c.i f
bk b (in cm) girls
12. (d) Given. (sec A + tan A) (1 – sin A) Below 140 4 Below 140 4
Below 145 11 140-145 7
( 1 sin A
)
= cos A + cos A (1 – sin A) Below 150
Below 155
29
40
145-150
150-155
18 (Maximum)
11
= ( cos A ) (1 – sin A)

s
1 + sin A Below 160 46 155-160 6

a
Below 165 51 160-165 5

iv D
2 2
= 1 − sin A = cos A = cos A Now, Modal class = 145–150
cos A cos A

Sh
\ Upper limit of Modal class = 150
BC C
13. (a) tan q = AB 19. (a) Both (A) and (R) are true and (R) is
6 m pole

6 3
the correct explanation of (A).
⇒ tan q = 2 3 × 3 20. (c)
θ
3 3 A B d = a2 – a1 d = a3 – a2 d = a4 – a3
⇒ tan q = 3
= 3 2 3m

⇒ tan q = tan 60º


\ q = 60º
=
−5
2
– (–5) =0– 2 ( −5 ) 5
=2 –0
−5 + 10 5 5
14. (a) A.T.Q. Perimeter of a circle = 2 =2 =2
 = Area of a circle +5
⇒ 2pr = pr2 = 2
\ r = 2 units \ A is true as common difference of an A.P.
15. (a) A.T.Q. A = A1 + A2 is the same but An A.P can have both
pR2 = pr12 + pr22 positive and negative rational numbers.
 r = 16 = 8 m Therefore A is true but R is false.
pR2 = p (r12 + r22) 1 2
... 
R2 = 82 + 62 = 64 + 36  r = 12 = 6 m Section-B
R2 = 100 = 102  2 2 21. Let us assume, to the contrary, that 2
\ R = 10 m is rational That is, we can find integers
16. (b) Area of Square = (side)2 a
a and b (≠ 0) such that 2 = .
= (2a)2 b
a a Suppose a and b have a common factor
= 4a2
Area of circle = pr 2 other than 1, then we can divide by the
= pa2 ...[r = a common factor and assume that a and b
2a are coprime.
MATHEMATICS – CBSE SAMPLE QUESTION PAPER–2024  P-9

Click here to access full book!


A
So, b 2 = a
C
Squaring on both sides, and rearranging,
we get 2b2 = a2. O
P
Therefore, a2 is divisible by 2 and a is E
also divisible by 2.
B D
So, we can write a = 2c for some integer c.
Substituting for a, we get 2b2 = 4c2, that Peri (D PCD) = PC + CD + DP
is b2 = 2c2. This means that b2 is divisible = PC + CE + DE + DP
by 2 and so b is also divisible by 2. = PC + CA + DB + DP ...[From (i) and (ii)
Therefore, a and b have at least 2 as a = PA + PB 
common factor. = PA + PA
But this contradicts the fact that a and b = 10 + 10  ...[PA = 10 cm (given)
are coprime. = 20 cm
This contradiction has arisen because
24. We have
of our incorrect assumption that 2 is
rational. 1
tan (A + B) = 3 tan (A – B) =
So, we conclude that 2 is irrational. 3
⇒ tan (A + B) = tan 60º ⇒ tan (A – B) = tan 30º
22. 5y ⇒ 30º + B + B = 60º ⇒ A – B = 30º

s
4y
...[From (i) \ A = 30º + B ...(i)
D Q 1y C

iv D a
2B = 60º – 30º
30

Sh
O \ B = 2 = 15º

A 2x 3x B
From (i), A = 30º + 15º = 45º
P
5x \ A = 45ºand B = 15º
Given. Point P divides AB in the ratio 2 : 3 Or
Let AP = 2x, PB = 3x, then AB = 2x + 3x = 5x We have,
3
Point Q divides DC in the ratio 4 : 1 2 cosec2 30° + x sin2 60° – 4 tan2 30° = 10
Let DQ = 4y, QC = 1y, then DC = 4y + y = 5y 2 2
 3 3 1
AB = DC ...[opposite sides of a || gm ⇒ 2(2)2 + x  2  − 4   = 10
5x = 5y  3
⇒ x=y
In DAOP and DCOQ
...(i) 3
()
3 1
⇒ 8 + x 4 − 4 × 3 = 10
⇒ ∠AOP = ∠COQ ...[Vertically opposite angles 32 + 3x − 1 10
⇒ ∠OAP = ∠OCQ  ...[Alternate Interior angles ⇒ 4
=
1
\ DAOP ∼ DCOQ  ...[AA Similarity Criterion ⇒ 31 + 3x = 40 ⇒ 3x = 40 – 31
OA AP 9
⇒ OC = CQ ...[CPST \ x= 3 =3
OA 2x
⇒ OC = 1y 25. B1
4 cm
cm

⇒ OA = 2x  ...[From (i)
14

OC 1x 14 c
cm

m
⇒ 2 OC = OA
14

C
1 A 14 cm 14 cm
\ OC = 2 OA  (Hence Proved)
Total area removed from DABC
23. CA = CE ...(i) Tangents drawn from an
DB = DE ...(ii) ...  A B C
external point are equal = 360 πr 2 + 360 πr 2 + 360 πr 2
PA = PB ...(iii)
P-10  Shiv Das CBSE SECONDARY SERIES (X)

Click here to access full book!


1 \ Total rooms = 5 + 7 + 9 = 21 rooms
= 360° πr 2 (∠A + ∠B + ∠C )
1 22 27. Given. 5x2 + 5x + 1
= × × 14 × 14 × 180° a = 5, b = 5, c = 1
360° 7
− b −5
… In DABC, ∠A + ∠B + ∠C = 180º, Sum of zeroes, a + b = a = 5 = –1
 by Angle Sum-Property of a D
= 308 cm2 c 1
Product of zeroes, ab = a = 5
Or
1. a2 + b2 = (a + b)2 – 2ab
Let the side of big square, A = 14 cm
Let the radius of circle, (1)
= (–1)2 – 2 5
r = d = 4 = 2 cm
... ∵ d =
14
−3 = 4 5−2 3
2 2  2 = =
5 5
A B 1 1 β + α −1
3 cm 2. a–1 + b–1 = α + β = αβ = 1
2 5
5
14 cm

P Q
4 = – 1 × 1 = –5
S R
28. Let ten’s place digit be x
3 cm and one’s place digit be y

s
D C \ Orginal number = 10x + y

a
14 cm
Reversed number = x + 10y

iv D
Let the side of small square, a = 4 cm A.T.Q

Sh
Area of square ABCD = (Side)2 = (A)2 10x + y + x + 10y = 66
Area of square PQRS = (a)2 ⇒ 11x + 11y = 66
= (4)2 = 16 cm2 x + y = 6 ...(i)  …[Dividing both sides by 11
1 x – y = 2 ...(ii) …[Given
Area of semicircles = 4 × 2 pr2 or, x – y = –2 ...(iii)
= 4 × 1/2 ×3.14(2)2 cm2 On solving (i) & (ii), On solving (i) & (iii),
= 25.12 cm2 we get we get
\ Required Area = (Area of small square +
x + y = 6 ...(i) x + y = 6 ...(i)
 Area of 4 semicircles)
x – y = 2 ...(ii) x – y = –2 ...(iii)
= 16 + 25.12 = 41.12 cm2
2x =8 2x =4
Section-C \x=4 \ x=2
Putting value of x in (i) Putting value of x in (i)
26. 60 = 22 × 3 × 5 2 60 2 84 2 108
4+y=6 2+y=6
84 = 22 × 3 × 7 2 30 2 42 2 54
108 = 22 × 33 3 15 3 21 3 27 ⇒y=2 ⇒y=4
5 7 3 9 \ Orginal no. = 10x + y \ Orginal no. = 10x + y
3 = 10(4) + 2 = 20 + 4
HCF of 60, 84, 108 = 22 × 3 = 12 = 42 = 24
The number of students in each group = 12
There are two such numbers 42 and 24.
60 Or
No. of groups for music = 12 = 5 rooms
1 1
84 Let = p, =q
No. of groups for dance = 12 = 7 rooms x y

108 2p + 3q = 2  …(i)
No. of groups for handicrafts = 12 = 9 rooms
4p – 9q = –1  …(ii)

MATHEMATICS – CBSE SAMPLE QUESTION PAPER–2024  P-11

Click here to access full book!


Multiplying (i) by 3 and adding in (ii) 2nd Method
A
6p + 9q = 6
4p – 9q = –1
1
10p    = 5 ⇒ p = 2 O P

1
Putting the value of p in (i), q = 3
B
On using the values of p and q
We have, ∠OAB = 30°
1 1 ∠OAP = 90°...[Angle between the tangent and
p=2 q=3  the radius at the point of contact]
1 1 1 1 \ ∠PAB = 90° – 30° = 60º
⇒ = ⇒ =
x 2 y 3 Now, AP = BP 
 ...[Tangents to a circle from an external point
⇒ x =2 ⇒ y =2 ∠PAB = ∠PBA
or x = 4 or y = 9  ...[Angles opposite to equal sides of a triangle
\ x = 4, y = 9 In DABP, ∠PAB + ∠PBA + ∠APB = 180º
 ...[Angle Sum Property
29. A
⇒ 60º + 60º + ∠APB = 180º
\ ∠APB = 60º
3 cm

s
Hence DABP is an equilateral triangle,
30º

a
O P
M
where AP = BP = AB.

iv D
3 cm

PA = 6 cm  ...[Given

Sh
B In rt. DOAP, ∠OPA = 30°
OA 1 OA
OP is the ⊥ bisector of AB ⇒ tan 30° = PA     ⇒ =
6
3
6 6
AM = MB = = 3 cm \ OA = = 2 3 cm
2 3
AM
In rt. DAMO, cos 30º = OA Or
Given. A circle with centre O, an external
3 = 3 point T and two tangents TP and TQ to the
2 OA
⇒ 3 OA = 6 circle, where P, Q are the points of contact.
To Prove.
6 3 6 3 ÐPTQ = 2ÐOPQ
P
\ OA = × = = 2 3 cm
3 3 3
Proof : Let ÐPTQ = q
In rt. DOMA, Now, TP = TQ T θ O
∠OAM + ∠AMO + ∠AOM = 180º 
 …[Angle-Sum-Property of a D
⇒ 30º + 90º + ∠AOM = 180º
Q
∵ Lengths of tangents drawn from
... 
an external Point to a ciircle are equal
⇒ ∠AOM = 180º – 30º – 90º = 60º
Now, ∠OAP = 90º So, TPQ is an isosceles triangle
 …[Tangent is ⊥ to the radius through 1
the point of contact \ ÐTPQ = ÐTQP = 2 (180º – q)
PA 1
In rt. DOAP, tan 60º = OA = 90º– q...(i)
2
PA
3= 2 3 Also, ÐOPT = 90°
\ PA = 6 cm ∵ Tangent at any point of a circle is ⊥
... 
 to radius through the pt. of contact

P-12  Shiv Das CBSE SECONDARY SERIES (X)

Click here to access full book!


So, ÐOPQ = ÐOPT – ÐTPQ = 90° – 90 º - 2 q ( 1
) According to the Question,

24

24 
... Time =
Distance
 ...[From (i) 18 − x 18 + x = 1  Speed
1 1 24[18 + x − (18 − x )]
ÐOPQ = 2 q Þ ÐOPQ = ÐPTQ Þ (18 − x )(18 + x )
=1
2
\ ÐPTQ = 2ÐOPQ ...(Hence Proved) 24( 2 x )
2
30. Given. 1 + sin q = 3 sin q cos q Þ =1 Þ 48x = 324 – x2
324 − x 2
Dividing both sides by cos2 q Þ x2 + 48x – 324 = 0
1 sin 2 θ 3 sin θ cos θ Þ x2 + 54x – 6x – 324 = 0
+ =
2
cos θ cos 2 θ cos 2 θ Þ x(x + 54) – 6(x + 54) = 0
⇒ sec2 q + tan2 q = 3 tan q Þ (x – 6) (x + 54) = 0
⇒ 1 + tan2 q + tan2 q = 3 tan q  x – 6 = 0 or x + 54 = 0
 ...[∵ 1 + sec2 q = tan2 q x = 6 or x = –54 (rejected)
Let tan q be x Since speed cannot be negative.
⇒ 1 + x2 + x2 = 3x  ...[Let tan q = x \ Speed of stream, x = 6 km/hr
⇒ 2x2 – 3x + 1 = 0 Or
⇒ 2x2 – 2x – x + 1 = 0
⇒ 2x (x – 1) –1 (x – 1) = 0
⇒ (x – 1) (2x – 1) = 0

s
⇒ x – 1 = 0 or 2x – 1 = 0

iv D a
1
⇒ x = 1 or x=2

Sh
1
\ tan q = 1 or tan q = 2 (Hence Proved)
31. Let V be the total volume of the tank
No. of Let smaller water tap alone can fill the tank
Length di = xi
leaves C.I. xi fi di in x hrs
(in mm) – 149
(fi) then larger water tap alone can fill the tank
118-126 3 117.5 – 126.5 122 –27 –81  in (x – 10) hrs.
127-135 5 126.5 – 135.5 131 –18 –90 –252 A.T.Q.
136-144 9 135.5 – 144.5 140 –9 –81
145-153 12 144.5 – 153.5 149 = a 0 0 ⇒ V+ V
=
V
154-162 5 153.5 – 162.5 158 9 45
xx - 10 75
8 ( )
( )
163-171 4 162.5 – 171.5 167 18 72 171
1 1
172-180 2 171.5 – 180.5 176 27 54 ⇒ V x + x - 10 = V. 8
75
Sfi = 40 Sfidi = –81
x - 10 + x 8
Σf d ( −81) ⇒ =
x( x - 10 ) 75
Mean = a + Σif i = 149 + 40
i ⇒ 8x (x – 10) = 75 (2x – 10)
= 149 – 2.025 = 146.975 Þ 8x2 – 80x = 150x – 750
\ Average length of the leaves = 146.975 mm Þ 8x2 – 80x – 150x + 750 = 0
Þ 8x2 – 230x + 750 = 0
Section-D Þ 4x2 – 115x + 375 = 0 ...[¸ both sides by 2
Þ 4x2 – 100x – 15x + 375 = 0
32. Let speed of the stream be x km/hr,
Þ 4x(x – 25) – 15(x – 25) = 0
Speed of the boat upstream = (18 – x)  km/hr
Þ (x – 25) (4x – 15) = 0
and Speed of the boat downstream
Þ x – 25 = 0 or 4x – 15 = 0
 = (18 + x) km/hr 15 3
Given, Distance = 24 km Þ x = 25 or x = 4 = 3 4 hrs.

MATHEMATICS – CBSE SAMPLE QUESTION PAPER–2024  P-13

Click here to access full book!


15 In DCEF, ∠CEF = ∠CFE  ...[Given
Rejecting x = 4 hrs. CF = EC  ...(iii)
Because one tap alone will fill the tank in  ...[Sides opposite to equal angles are equal
more than 9 hrs. From (ii) and (iii), GE = CF  ...(iv)
\ x = 25 hrs. From (P) and (iv), FD = GE  ...(v)
33. (a) Statement: If a line is drawn parallel to AB AE
\ =
BD FD
 ...[From (i)
one side of a triangle to intersect the other two
sides in distinct points, the other two sides are ⇒ AB = AE ...[From (v)  (Hence Proved)
BD GE
divided in the same ratio.
A 34. Given. H = 15 km = 15000 m
Given: In DABC, DE || BC. (∵ 1 km = 1000 m)
M N
To prove: AD = AE 14 7
DB EC D E
Radius of pipe (r) = 2 = 7 cm = 100 m
Const.: Draw EM ^AD Volume of pipe (cylinder) = pr2H
and DN ^ AE.
Join B to E and C to D. \ Volume of water flowing through the
B C
Proof: In DADE and DBDE, cylindrical pipe in an hour at the rate of
ar ( DADE ) ½ ¥ AD ¥ EM AD ...(i) 15 km/hr.
= =
ar ( DBDE ) ½ ¥ DB ¥ EM DB 22 7 7
= 7 × 100 × 100 × 15000 = 231 m3
 ...[Area of D = ½ × base × corresponding altitude

a s
In DADE and DCDE, Speed of water flowing = 15 km/hr 7 cm

iv D
ar ( DADE ) ½ ¥ AE ¥ DN AE  ...(ii)
= =
ar ( DCDE ) ½ ¥ EC ¥ DN EC

Sh
Tank
∵ DE || BC ...[Given
(cuboid)
\ ar (DBDE) = ar(DCDE)
...[∵Ds on the same base and between the same
 parallel sides are equal in area

From (i), (ii) and (iii), AD = AE


44 m
DB EC 50 m
 (Hence proved) Volume of tank (cuboid) (tank) = lbh
Or 21
= 50 × 44 × 100  ...[h = 21 cm = 21/100 m
Given. ∠CEF = ∠CFE and F A
= 462 m 3
is the mid-point of DC.
AB AE Since 231 m3 of water falls into tank in = 1 hr.
To Prove. BD = FD \ 462 m3 of water falls into tank in
G
Construction. From D, draw 1
E = 231 × 462 = 2 hours
DG || BE.
Proof : In DABE, D Hence Speed of water if the rise in
F C
DG || BE  ...[By construction
water level is to be attained in one hour
AB AE = 15 km/hr × 2 = 30 km/hr
\ =  ...(i) B
BD GE Or
 ...[By corollary of Basic Proportionality theorem Radius, r = 14 m
In DCDG, FE || DG Height of cylinder, h = 3 m 10.5 m

DF GE  ...[By BPT Height of cone, H = 13.5 –3


=
13.5 m
CF EC 14 m
21
⇒ DF = GE  ...[F is the mid point of DC DF = CF HH = 10.5 m or = 2 m
DF EC 14 m

⇒ GE = EC ...(ii) l = r 2 + H2

P-14  Shiv Das CBSE SECONDARY SERIES (X)

Click here to access full book!


(ii) Since Maximum frequency = 25
( 212 )
2
= 14 2 + = 196 + 441 \ Modal Class = 40 – 50,
4
f1 − f0
784 + 441 1225 35 Mode = l + 2 f − f − f × h
= 4
=
4
= 2 = 17.5 m 1 0 2


C.S. Area of canvas = 2prh + prl
= pr (2h + l)
( 25 − 15
= 40 + 50 − 15 − 20 × 10 
…Here,
) l = 40
f0 = 15
f1 = 25
10 × 10 – – f2 = 20
22 = 40 + 15 = 40 + 6.6 = 46.6
= 7 × 14 × (2(3) + 17.5) h = 10
= 44 × (23.5) 36. (i) a = 40, d = 12
= 1,034 m2 \ a11 = a + 10d  ...[an = a + (n – 1) d
Total Canvas required = 1,034 + wastage = 40 + 10(12)
= 1,034 + 26 = 40 + 120 = 160 throws
= 1,060 m2 (ii) a = 7.56 m, d = 9 cm or 0.09 m
Now, Cost of Canvas @ `500 per m2 n = 6 weeks
= `500 × 1,060 Now, a6 = a + 5d
= `5,30,000 = 7.56 + 5 (0.09) = 7.56 + 0.45
35.  = 8.01 m
Or
Marks No. of Students
Cfi an = 11.16 m, n = ?

s
obtained (fi)

a
As we know, a + (n – 1) d = an

iv D
20-30 p p
30-40 15 f0 p + 15 7.56 + (n – 1) (0.09) = 11.16
⇒ (n – 1) (0.09) = 11.16 – 7.56

Sh
40-50 25 f1 p + 40
3.60 36 100
50-60 20 f2 p + 60 ⇒ (n – 1) = 0.09 ⇒ (n – 1) = 9 × 10
60-70 q p + q + 60 ⇒ n – 1 = 40 ⇒ n = 40 + 1 = 41
70-80 8 p + q + 68
80-90 10 e p + q + 78 \ n = 40 + 1 = 41
Sam Therefore Sanjita will be able to throw
Sfi = 90
11.16 m in 41 weeks.
Given. p + 15 + 25 + 20 + q + 8 + 10 = 90
(iii) a = 40, d = 12, n = 15
⇒ p + q = 90 – 78 n
Sn = 2 [2a + (n – 1) d]
\ p + q = 12  ...(i)
Given. Median = 50, n = 90 15 15
= 2 ( 80 + (14)12) = 2 × (80 + 168)
\ n = 90 = 45 15
2 2 = 2 × 248 = 1,860
Median class is 50-60 Therefore Total throws = 1,860
n l = 50 37. (i) D (x, y)
− cf n
Median = l + 2 f × h ...  2 = 45 , cf = p + 40 

 f = 20 , h = 10
A C (6, 6)
(1, 2)
 45 − ( p + 40 ) 
⇒ 50 = 50 +  20
× 10 
 
B (4, 3)
50 − 50 45 − p − 40
⇒ = Let Coordinates of point D be (x, y)
1 2
Mid-point of BD = Mid-point of AC
⇒ 5–p=0 \ 5=p
From (i), 5 + q = 12 ⇒ q=7 ⇒  2 , 2 = 2 , 2  (
4 + x y + 3 1 + 6 2 + 6
)
\ p = 5, q = 7  ...[Diagonals of a || gm bisect each other

MATHEMATICS – CBSE SAMPLE QUESTION PAPER–2024  P-15

Click here to access full book!


y+3 8 38.
⇒ 4+x = 7 ; = F Bird A D Bird
2 2 2 2 Ball
⇒ x=3 ; y=5
\ Coordination of point D is (3,5)
Height
(ii) (–3, 5) (3, 1) (0, 3) Tree
80 m 80 m
60º
G H K
º
45
2 2
GH = ( 3 + 3) + ( 1 − 5) = 36 + 16 = 52 30º Ground level
C G B E
= 4 × 13 = 2 13 units
AB
(i) In rt. DABC, tan 45º = BC
HK = (0 − 3)2 + ( 3 − 1)2 = 9+4
80
= 13 units 1 = BC   ⇒ BC = 80 m

GK = ( 0 + 3 )2 + ( 3 − 5 )2 = 9+4 (ii) In rt DCDE, tan 30º = DE


CE
= 13 units 1 80
= =
As HK + GK = GH = 13 + 13 = 2 13 3 CE

∴ G,H and K lie on the same straight line ⇒ CE = 80 3 m


Or Distance the bird flow, AD = BE

s
Given. J (5, –3), I (–4, 6) and C(0, 1) AD = CE – BC

iv D a
AD = 80 3 – 80 m
CJ = ( 5 − 0 )2 + ( −3 − 1)2 = 25 + 16 
AD = 80 ( 3 - 1) m

Sh
= 41 units
Or
2 2
CI = ( −4 − 0 ) + (6 − 1) = 16 + 25 FG 80
In rt DCGF, tan 60º = CG ⇒ 3 = CG
= 41 units
Yes, C (0, 1) is equidistant from Point J and I. 80 3 80 3
\ CG = × = m
( ) ( ) 3 3 3
5 + ( −4 ) −3 + 6 1 3
Mid-point of IJ = , = 2,2
2 2 \ Distance the ball travelled after hitting
But Co-ordinates of CI (0, 1) ...[Given the tree = FA = GB
As (0, 1) ≠ 2 , 2 (1 3) = BC – CG
 3
= 80 – 80 3 = 80  1 − 3 
Therefore, C is not the Mid-Point of IJ. 3
(iii) As B is equidistant from A and E 80
= 3 (3 − 3 ) m 
\ Mid-point of AE = B
(1, 4) (2, –3) (a, b) Distance
(iii) Speed = Time
A B E
20 ( 3 + 1)
( 1 +2 a , 4 +2 b ) = (2, –3) = 2
m/sec

20 ( 3 + 1)
= 2
m/min
1+ a 4+b
2 =2 2
= –3 60
⇒ 1 + a = 4 ⇒ 4 + b = –6 60
= 20 ( 3 − 1) × 2
\ a = 3 \ b = –10
= 600 ( 3 − 1) m/min
\ E (3,–10)
]••••]

P-16  Shiv Das CBSE SECONDARY SERIES (X)

Click here to access full book!


Code : 041  Roll No.
• Please check that this question paper contains 6 printed pages.
• Code number given on the right hand side of the question paper should be
written on the title page of the answer-book by the candidate.
• Please check that this question paper contains 38 questions.
• Please write down the Serial Number of the question before attempting it.

MATHEMATICS–X
Sample Question Paper–01
Time Allowed: 3 hours Maximum Marks: 80
GENERAL INSTRUCTIONS:
1. This Question Paper has 5 Sections A-E.
2. Section A has 20 MCQs carrying 1 mark each.

s
3. Section B has 5 questions carrying 02 marks each.

D a
4. Section C has 6 questions carrying 03 marks each.

Sh i v
5. Section D has 4 questions carrying 05 marks each.
6. Section E has 3 case based integrated units of assessment (04 marks each) with
sub-parts of the values of 1, 1 and 2 marks each respectively.
7. All Questions are compulsory. However, an internal choice in 2 Qs of 5 marks, 2
Qs of 3 marks and 2 Questions of 2 marks has been provided. An internal choice
has been provided in the 2 marks questions of Section E
8. Draw neat figures wherever required. Take π = 22/7 wherever required if not
stated.
SECTION-A
Section A consists of 20 questions of 1 mark each.

1. The ratio of LCM and HCF of the least composite and the least prime numbers is
(a) 1 : 2 (b) 2 : 1 (c) 1 : 1 (d) 1 : 3
1
2. If 2 and 2 are the zeros of px2 + 5x + r, then
(a) p = r = 2 (b) p = r = –2 (c) p = 2, r = –2 (d) p = –2, r = 2
3. The lines x = a and y = b, are
(a) intersecting (b) parallel
(c) overlapping (d) None of these

MATHEMATICS – SAMPLE QUESTION PAPER–01  P-17

Click here to access full book!


4. Mohan and Sohan solve an equation. In solving Mohan commits a mistake in constant term
and finds the roots 8 and 2. Sohan commits a mistake in the coefficient of x and finds roots
–9 and –1. The correct roots are
(a) 9, 1 (b) –9, 1 (c) 9, –1 (d) –9, –1
5. The distance of point A(-5, 6) from the origin is
(a) 11 units (b) 61 units (c) 11 units (d) 61 units
6. ∆ABC is such that AB = 3 cm, BC = 2 cm, CA = 2.5 cm. If ∆ABC ~ ∆DEF and EF = 4 cm,
then perimeter of ∆DEF is
(a) 7.5 cm (b) 15 cm (c) 22.5 cm (d) 30 cm
7. If 2 sin2β – cos2β = 2, then β is
(a) 0º (b) 90º (c) 45º (d) 30º
A

8. In the figure, if DE || BC, AD = 3 cm, BD = 4 cm and BC = 14 cm,


then DE equals
(a) 7 cm (b) 6 cm D E
(c) 4 cm (d) 3 cm
9. The next term of the sequence B C
1 1 1
, , is (x ≠ 1).
1+ x 1 − x 1− x

a s
(a) 1 + 2 x (b) 1 − 2 x (c) 1 − 2 x (d) 1 + 2 x

D
1−x 1−x

i v
PX PY 1 P

h
10. In the given figure, XY || QR and XQ = YR = 2 , then

S
1
(a) XY = QR (b) XY = 3 QR X Y

1
(c) XY2 = QR2 (d) XY = 2 Q R
P
11. In the given figure, CP and CQ are tangents to a circle with centre O. A

ARB is another tangent touching the circle at R. If CP = 11 cm and O R C


BC = 6 cm then the length of BR is
(a) 6 cm (b) 5 cm Q
B

(c) 4 cm (d) 3 cm
12. The number of revolutions made by a circular wheel of radius 0.7 m in rolling a distance
of 176 m is
(a) 22 (b) 24 (c) 75 (d) 40
13. A cylinder and a cone are of same base radius and of same height. The ratio of the
volume of the cylinder to that of the cone is
(a) 2 : 1 (b) 3 : 1 (c) 2 : 3 (d) 3 : 2
14. For the following distribution:
C.I. 0-10 10-20 20-30 30-40 40-50
f 20 30 24 40 18
the sum of lower limits of the modal class and the median class is
(a) 20 (b) 30 (c) 40 (d) 50
2
15. The area of the circle is 154 cm . The radius of the circle is
(a) 7 cm (b) 14 cm (c) 3.5 cm (d) 17.5 cm
P-18  Shiv Das CBSE SECONDARY SERIES (X)

Click here to access full book!


16. For the following distribution:
Marks obtained No. of students
More than or equal to 0 63
More than or equal to 10 58
More than or equal to 20 55
More than or equal to 30 51
More than or equal to 40 48
More than or equal to 50 42
the frequency of the class 20–30 is
(a) 35 (b) 4 (c) 48 (d) 51
17. Two fair coins are tossed. What is the probability of getting at the most one head?
3 1 1 3
(a) 4 (b) 4 (c) 2 (d) 8
4 sin β - 3 cos β
18. If 4 tan β = 3, then =
4 sin β + 3 cos β
1 2 3
(a) 0 (b) 3 (c) 3 (d) 4
Direction: In the question number 19 and 20, a statement of Assertion (A) is followed by a
statement of Reason (R). Choose the correct option.
(a) Both A and R are true and R is the correct explanation of A.
(b) Both A and R are true and R is not the correct explanation of A.

a s
(c) A is true but R is false.

D
(d) A is false but R is true.

i v
19. Assertion: x2 + 4x + 5 has two zeroes.

Sh
Reason: A quadratic polynomial can have at the most two zeroes.
20. Assertion: In a circle of radius 6 cm, the angle of a sector is 60°. Then the area of the sector is
6
18 cm2.
7
Reason: Area of the circle with radius r is pr2.

SECTION-B
Section B consists of 5 questions of 2 marks each.
21. Determine the value of α for which the following system of linear equations has an infinitive
number of solutions:
A
αx + 3y = α – 3; 12x + αy = α
m
1c

22. In given Fig., XY || BC. Find the length of XY. X Y


m
3c

B C
6 cm
23. Two concentric circles are of radii 7 cm and r cm respectively, where r > 7. A chord of the
larger circle, of length 48 cm, touches the smaller circle. Find the value of r.
P

24. Find the area of the major segment APB, in Fig., of a circle of radius 35 cm O
22
and AOB = 90°. (Use p = ) 90º
7 A B

Fig. 4
MATHEMATICS – SAMPLE QUESTION PAPER–01  P-19

Click here to access full book!


Or
A
In given figure, arcs are drawn by taking vertices A, B and C of an
equilateral triangle ABC of side 14 cm as centres to intersect the sides BC,
F E
CA and AB at their respective mid-points D, E and F. Find the area of the
shaded region. [Use p = 22 and 3 = 1.73]
7 B C
D
Fig. 6
25. If x sin3 + y cos3 θ = sin θ cos θ, and x sin θ = y cos θ , then prove that x2 + y2 = 1.
Or
1 + sin q 1 - sin q
Prove the identity:
+ = 2 sec θ.
1 - sin q 1 + sin q

SECTION-C
Section C consists of 6 questions of 3 marks each.
26. Check whether 6n can end with the digit 0 for any natural number n?
27. If α and β are zeroes of the polynomial 6x2 – 7x – 3, then form a quadratic polynomial whose
1 1
zeroes are α and .
β
28. The sum of the digits of a two-digit number is 12. The number obtained by interchanging the

a s
two digits exceeds the given number by 18. Find the number.

iv D
Or

Sh
The age of the father is twice the sum of the ages of his 2 children. After 20 years, his age will

be equal to the sum of the ages of his children. Find the age of the father.
sin  sin 
29. Prove that: cot   cosec  = 2 + cot   cosec  . C
27 cm
D R
30. In the figure, quadrilateral ABCD is circumscribing a circle
with centre O and AD ^ AB. If radius of a circle is 10 cm,

cm
S O
then find the value of x.
38
10 cm

Or A
Q
P B
x cm

Prove that the line segment joining the points of contact of two parallel tangents of a circle,
passes through its centre.
31. A box contains cards numbered 3, 5, 7, 9, ..., 35, 37. A card is drawn at random from the box.
Find the probability that the number on the drawn card is a prime number.

SECTION-D
Section D consists of 6 questions of 5 marks each.
32. Two pipes running together can fill a cistern in 3 1 minutes. If one pipe takes 3 minutes
13
more than the other to fill it, find the time in which each pipe would fill the cistern.
Or
A motor boat whose speed is 20 km/h in still water, takes 1 hour more to go 48 km upstream
than to return downstream to the same spot. Find the speed of the stream.

P-20  Shiv Das CBSE SECONDARY SERIES (X)

Click here to access full book!


33. In ∆ABC, altitudes AD and CE intersect each other at the point P. Prove that
(i) ∆APE ~ ∆CPD (ii) AP × PD = CP × PE
(iii) ∆ADB ~ ∆CEB (iv) AB × CE = BC × AD
34. Water is flowing at the rate of 15 km/hour through a pipe of diameter 14 cm into a cuboidal
pond which is 50 m long and 44 m wide. In what time will the level of water in the pond rise
by 21 cm?
Or
A solid is in the shape of a cone surmounted on a hemisphere, the radius of each of them
22
being 3.5 cm and the total height of solid is 9.5 cm. Find the volume of the solid. [Use p =
7
35. Find the median of the following data:

Monthly consumption (in units) No. of Consumers


Below 85 4
Below 105 9
Below 125 22
Below 145 42
Below 165 56
Below 185 64
Below 205 68

s
SECTION-E

D a
Case study based questions are compulsory.

Sh i v
36. The given figure alongside shows the path of a diver, when she takes
a jump from the diving board. Clearly it is a parabola.
Annie was standing on a diving board, 48 feet above the water level.
She took a dive into the pool. Her height (in feet) above the water
level at any time ‘t’ in seconds is given by the polynomial h(t) such
that h(t) = –16t2 + 8t + k.
(i) What is the value of k?1
(ii) At what time will she touch the water in the pool? 1
(iii) A polynomial q(t) with sum of zeroes as 1 and the product as –6 is modelling Anu’s
height in feet above the water at any time t(in seconds). Then find the value of q(t).
2
Or
The zeroes of the polynomial r(t) = –12t² + (k – 3)t + 48 are negative of each other.
Find value of k.
37. The concept of the Cartesian coordinate system may seem simple, but it can be extended
and analyzed for much more complex functions. Now-a-days, these concepts are essential in
fields such as engineering, physics, architecture, astronomy, data processing, 3D modeling,
and computer graphics. 2D Cartesian plane concept can be extended to 3D space by adding
a third axis – the Z-axis. This third axis is mutually perpendicular to both the X and Y axes.
All three axes must share a common origin. Every point 3D space can then be represented
in terms of X, Y, and Z coordinates.
An architect by using this concept designed a hut on a catesian plane.

MATHEMATICS – SAMPLE QUESTION PAPER–01  P-21

Click here to access full book!


E

F
D
G N
H S R K
M C
L

A P Q B

Y Y
F V E F
H G
G D
K N S R

L M

B C A P Q B
O Side View X O Front View X

Based On the information given above answers the questions that follow.

s
(i) Find the mid–point of the segment joining H(1, 4) and G(4, 4). (Refer to Front View)

a
1

iv D
(ii) What is the distance between B and E? (Refer to Side View) 1

Sh
(iii) If a point V (x, y) is equidistant from F(2, 5) and E(7, 5), then find the polynomial
equation. (Refer to Side View) 2
Or
Find the coordinates of the point which divides the line segment joining the points
A(1, 1) and B(4, 1) in the ratio 1 : 2 internally. (Refer to Front View)
38. A Satellite flying at height h is watching the top of the
two tallest mountains in Uttarakhand and Karnataka,
them being Nanda Devi (height 7,816 m) and
Mullayanagiri (height 1,930 m). The angles of depression
from the satellite, to the top of Nanda Devi and
Mullayanagiri are 30° and 60° respectively. If the
distance between the peaks of two mountains is
1973 km, and the satellite is vertically above the mid-
point of the distance between the two mountains.
(i) Find the distance of the satellite from the top of Nanda Devi. 2
Or
Find the distance of the satellite from the top of Mullayanagiri. 2
(ii) Find the distance of the satellite from the ground. 1
(iii) What is the angle of elevation if a man is standing at point C a distance of 7816 m
from Nanda Devi? 1
__________________

P-22  Shiv Das CBSE SECONDARY SERIES (X)

Click here to access full book!


Sample Question Paper-01

SECTION-A 7. (b) Given. 2 sin2b – cos2b = 2


⇒ 2 sin2b – (1 – sin2b) = 2
1. (b) Least composite number is 4 and the
least prime number is 2. ⇒ 2 sin2b – 1 + sin2b = 2
\ LCM (4, 2) : HCF (4, 2) = 4 : 2 = 2 : 1. ⇒ 3 sin2b = 3 ⇒ sin2b = 1
2. (b) We know,    \ b = 90º
-coefficient of x A
Sum of zeroes = 8. (b) Since DE || BC,

m
2
coefficient of x

3c
\ DABC ∼ DADE
D E
5 -5
⇒ 2 + 1 = -5 \ 2= p ..[By AA rule of similarity

m
2 p

4c
\ p = –2 Using Thales’ Theorem, B 14 cm C
1 r AD DE

s
Also, Product of zeroes = 2 ¥ 2 = p =

a
AB BC

D
r 3 DE 42
⇒ p = 1      ⇒ r=p ⇒ = \ DE = 7 = 6 cm

i v
(3 + 4) 14

Sh
\ r = p = –2 9. (d) Given.
3. (a) Lines x = a is a line parallel to y-axis 1 1 1 1− x 1 1+ x
and y = b is a line parallel to x-axis. So , , = , ,
1+ x 1−x 1− x 1−x 1−x 1−x
they will intersect.
Common difference, d
4. (a) Correct sum = 8 + 2 = 10 from Mohan
1 1− x x 1+ x 1 x
Correct product = –9 × (–1) = 9 from Sohan = − = and − =
1−x 1−x 1−x 1−x 1−x 1−x
\ x2 – (10)x + 9 = 0
\ Given sequence is an A.P. with common
⇒ x2 – 10x + 9 = 0
x
\ x2 – 9x – x + 9 = 0 difference .
1− x
⇒ x(x – 9) –1(x – 9) = 0 Hence, the next term, (a4) = a3 + d

⇒ (x – 9) (x – 1) = 0 1+ x x 1+ 2 x
\ Correct roots are 9 & 1. = + =
1−x 1−x 1− x
5. (d) Distance of Point A(–5, 6) from the 10. (b) Since XY || QR and PQ
Origin (0, 0) = (0 + 5)2 + (0 - 6)2 is a transversal, then
= 25 + 36 = 61 units PXY = PQR...(i)
...[corresponding angles
6. (b) Given. DABC ∼ DDEF
Since XY || QR and PR is a
Perimeter of DABC BC transversal, then
⇒ Perimeter of DDEF = EF
PYX = PRQ...(ii) [corresponding angles
3 + 2 + 2.5 2
⇒ Perimeter of DDEF = 4 In DPXY and DPQR,
PXY = PQR ...[From (i)
\ Perimeter of DDEF = 15 cm
DPXY ~ DPQR ...[AA similarity
MATHEMATICS – SAMPLE QUESTION PAPER–01  P-23

Click here to access full book!


PX XY PY 17. (a) Possible outcomes are (HH), (HT),

 = =
(TH), (TT).
PQ QR PR
PY XY XY 1 Favourable outcomes(at the most one
 =  QR = 3
PR QR head) are (HT), (TH), (TT)
1
\ XY = 3 QR \ P(at the most 1 head) = 3
4
Given. CP = 11 cm
11. (b) 4 sin β - 3 cos β
18. (a) We have, 4 sin β + 3 cos β
and BC = 6 cm
Dividing both numerator and denominator
Since BQ = BR ...(i) [∵ Tangents drawn
by cos b,
from external points are equal
4 sin b 3 cos b
CQ = CP ...[Using (i) -
cos b cos b
∴ BC + BQ = CQ P ⇒ 4 sin b 3 cos b
A +
cos b cos b
O R C
⇒ 6 + BR = 11 4 tan β - 3 3 - 3
\ 4 tan β + 3 3 + 3 = 0
=
\ BR = 11 – 6 = 5 cm Q
B
...[ 4 tan b = 3 (given)
12. (d) Number of revolutions
19. (d) Assertion (A) is false but Reason (R) is
Total distance
= Circumference of wheel true.
Given. x2 + 4x + 5
176 = 1760 = 40

s
= As we know, D = b2 – 4ac

a
2¥ 22 ¥ 0.7 44

iv D
7 = (4)2 – 4 × 1 × 5 = –4 < 0 (Imaginary roots)
Volume of cylinder
13. (b) Required Ratio = 20. (b) Both (A) and (R) are true and (R) is not
Volume of cone

Sh
2 the correct explanation of (A).
= πr h = 1 × 3 = 3 θ
1 πr 2 h 1 Area of sector = πr2
3 360∞
∴ Required ratio = 3 : 1 60 ∞ 22 132
= ¥ ×6×6= or 18 6 cm2
14. (d) Modal class = 30 – 40 and 360 ∞ 7 7 7
lower limit of modal class = 30 SECTION-B
Here, n = 132 = 66 21. ax + 3y = a – 3; 12x + ay = a
2 2 The system of linear equations will have
c.f. 20 50 74 114 132 infinite solutions
 Median class = 20 – 30 and lower limit If a1 = b1 = c1  ...(i)
a2 b2 c2
of median class = 20
Here a1 = a b1 = 3 c1 = a – 3
\ Required sum = 30 + 20 = 50
a2 = 12 b2 = a c2 = a
15. (a) Area of circle = 154 cm2
⇒ pr2 = 154 cm2 α 3 α−3 α 3
= =  =
22 12 α α 12 α
⇒ × r2 = 154 ⇒ r2 = 154 × 7  a2 = 36  a = 6
7 22
⇒ r2 = 7 × 7 = 49 \ r=
a = 6 satisfies (i)
49 = 7 cm
\ a=6
16. (b)
22. Here, AX = 1 cm, XB = 3 cm
Marks 0–10 10–20 20–30 30–40 40–50 50–60
Now, AB = AX + XB = 1 + 3 = 4 cm
No. of
students
5 3 4 3 6 0  AB = 4 cm
\ Frequency of class 20–30 = 4

P-24  Shiv Das CBSE SECONDARY SERIES (X)

Click here to access full book!


In DAXY and DABC  Shaded Region = area of major segment
∠XAY = ∠BAC ...[Common = ar (circle) – ar (minor segment)
∠AXY = ∠ABC ...[Corresponding angles = 3850 – 350 = 3500 cm2
\ DAXY ~ DABC ...[AA similarity Or
A 24. ABC = BAC = ACB = 60°
AX XY  ...[In an equilateral ∆ all angles are of 60° each

m
\ =

1c
AB BC 14
...[sides are proportional X Y Let q = 60°, r = 2 = 7 cm

m
Area of shaded region = ar(DABC)
3c
1 XY
\ =    B C – 3(ar of minor sector)
4 6 6 cm
6 3 3 θ

\ XY = = = 1.5 cm = 4 (side)2 – 3 . 360 pr2
4 2
23. Given: OC = 7 cm, 1.73 60º 22
= 4
(14 × 14) – 3 × 360º × 7 × 7 × 7
AB = 48 cm
= 84.77 – 77 = 7.77 cm2
To find: r = ?
O
r 7 cm 25. x sin3 q + y cos3 q = sin q cos q ...(i)
Sol. OCA = 90° A
C B ...[x sin q = y cos q; x sin θ = y
...[Tangent is ^ to the radius cos θ
sin θ
through the point of contact  x sin3 q + x cos θ × cos3 q = sin q cos q
As, OC ^ AB ...[From (i)
 x sin3 q + x sin q cos2 q = sin q cos q
1

s
AC = 2 (AB)  x sin q (sin2 q + cos2 q) = sin q cos q

D a
...[^ from the centre bisects the chord  x sin q .1 = sin q cos q

v
sin θ cos θ

i
1  x=  x = cos q

Sh
\ AC = (48) = 24 cm sin θ
2 sin θ
Put value of x in (i), y = cos q.
In rt. DOCA, OA2 = OC2 + AC2 cos θ
y = sin q
...[Pythagoras’ theorem
As per question, x2 + y2 = 1
r2 = (7)2 + (24)2 = 49 + 576 = 625
L.H.S. = x2 + y2 = sin2 q + cos2 q = 1 = R.H.S.
\ Radius, r = 625 = 25 cm Hence L.H.S. = R.H.S.
24. Here q = 90°, r = 35 cm Or
Area of minor segment = Area of minor 1 + sin θ 1 − sin θ
25. L.H.S. = +
1 − sin θ 1 + sin θ
sector – ar(DAOB)
θ [ 1 + sin θ ]2 + [ 1 − sin θ ]2
= 360º pr2 – 1 × OA × OB =
2 1 − sin θ 1 + sin θ
90º 22
= 360º × 7 × 352 – 1 × 35 × 35 =
1 + sin θ + 1 − sin θ
=
2
2
P 1 − sin 2 θ cos 2 θ
 1 22 1  2 1
= 352  4 × 7 − 2  = =2× = 2 sec q = R.H.S.
cos θ cos θ
 11 1  O
= 352  14 − 2 
90º
SECTION-C
26. Any positive integer ending with the digit
 11 − 7  A B
= 35 × 35  14  zero is divisible by 5 and so its prime
Fig. 4
factorization must contain the prime no. 5.
4
= 35 × 35 × 14 = 350 cm2 6n = (2 × 3)n = 2n × 3n
22 \ Prime factorisation of 6n does not con-
Also, Area of circle = pr2 = 7 × 35 × 35 tain 5 as a factor.
= 110 × 35 = 3850 cm2 Hence, 6n can never end with digit 0 for
any natural number.
MATHEMATICS – SAMPLE QUESTION PAPER–01  P-25

Click here to access full book!


27. 6x2 – 7x – 3 ...[Given polynomial According to the Question,
Here a = 6, b = –7, c = –3 2(x + y) + 20 = x + 20 + y + 20
−b −( −7) 7 2x + 2y + 20 = x + y + 40
a+b= = = ,
a 6 6 2x + 2y – x – y = 40 – 20
c −3 −1
ab = = = x + y = 20
a 6 2
1 1 \ From (i), Present age of father = 2(20)
To form a polynomial whose zeroes are & β
α = 40 years
7 sin θ
29. L.H.S. = cot θ + cosec θ
1 1 β+α −7
Sum of zeroes, S = + = = 6 =
α β αβ –3 3 sin θ sin θ
= =

Product of zeroes, P =
1 1
× =
6
1
=
1
= –2
( cos θ
+
1
sin θ sin θ ) (
cos θ + 1
sin θ )
α β αβ −1 2 (1 − cos 2 θ)
sin θ
2 = (cos θ + 1) = (cos θ + 1)
2
A quadrilateral polynomial is x – Sx + P

( )
(1 − cos θ)(1 + cos θ)
7 = = (1 – cos q)
= x2 – − x + (–2) (cos θ + 1)
3
=
2
3x + 7 x − 2
3
=
1
3
(3x2 + 7x – 2) R.H.S. = 2 + cot θ − cosec θ( sin θ
)
28. Let the unit’s place digit be x and the ten’s sin θ sin θ

s
   
= 2 +  cos θ
( )
a
place digit be y 1  = 2 +  cos θ − 1 

D
\ Original number = x + 10y and  sin θ − sin θ   sin θ 

i v
Interchanged number = y + 10x

h

sin θ 2  (1 − cos 2 θ)
Given, x + y = 12 = 2 +  cos θ − 1  = 2 + cos θ − 1

S
 y = 12 – x...(i)
(1 − cos θ)(1 + cos θ)
According to Questions. =2+ (cos θ − 1)
(10x + y) – (x + 10y) = 18
 10x + y – x – 10y = 18 = 2 + [–(1 + cos q)]
 9x – 9y = 18 = 2 – 1 – cos q = 1 – cos q = L.H.S.
 x – y = 2 ...[Dividing by 9 30. Given. Quadri- m C
27 c
lateral ABCD is
 x – (12 – x) = 2 ...[From (i) R
circumscribing a circle D
 x – 12 + x = 2 with centre O and AD
38 cm
 2x = 2 + 12 = 14 \ x = 7 ^ AB. S O
10 cm
Putting the value of x in (i), we get Const. Join OP. Q

y = 12 – 7 = 5 ∠ASO = ∠APO = 90°


\ The original number = x + 10y Tangent is perpendicular
A P
x cm
B

to the radius through the


= 7 + 10(5) = 57 ... point of contact
Or
28. Let the present ages of his children be: ∠SAP = 90° ...[Given
x years and y years ∴ ∠SOP = 90°
Then the present age of the father  ...[By angle-sum-property of a quadrilateral
= 2(x + y) ...(i) ∴ AS = AP ...(i)
After 20 years, his children’s ages will be ...[Tangents drawn from an external point are equal
(x + 20) and (y + 20) years ∴ OSAP is a square. 
After 20 years, father’s age will be AP = OS = 10 cm ...[∵ sides of a square
2(x + y) + 20
P-26  Shiv Das CBSE SECONDARY SERIES (X)

Click here to access full book!


CR = CQ = 27 cm
 13x(x + 3) = 40(2x + 3)
...[Tangents drawn from an external point are equal

 13x2 + 39x = 80x + 120
BQ = BC – CQ = 38 – 27 = 11 cm
 13x2 + 39x – 80x – 120 = 0
BP = BQ = 11 cm

 13x2 – 41x – 120 = 0
...[Tangents drawn from an external point are equal
Now x = AB
 13x2 – 65x + 24x – 120 = 0
x = AP + BP    ∴  x = 10 + 11 = 21 cm
 13x(x – 5) + 24(x – 5) = 0
Or
 (x – 5)(13x + 24) = 0
30. Given: CD and EF are C A D x–5=0 13x + 24 = 0
two parallel tangents 1 −24
or x = 5 x = (rejected)
at points A and B of a 2 13
circle with centre O. M
3
O ∵ Time cannot be –ve
To prove: AB passes Hence, the faster pipe fills the cistern in
through centre O or 5 minutes, and the slower pipe fills the cis-
E B F
AOB is diameter of tern in 8 i.e., 5 + 3 = 8 minutes.
the circle. Or
Construction.: Join OA and OB. 32. Let the speed of the stream be x km/hr
Draw OM || CD. \ Speed of the boat upstream = (20 – x) km/hr.
Proof: ∠1 = 90º ...(i) and speed of the boat downstream =
...[∵ Tangent is ^ to the radius through (20 + x) km/hr
the point of contact Distance = 48 km

s
According to the Question,

a
OM || CD

D
48 48 = 1

v
\ ∠1 + ∠2 = 180º ...[Co-interior angles −

i
20 − x 20 + x

Sh
90º + ∠2 = 180º ...[From (i)
 48[20 + x − (20 − x )] = 1
∠2 = 180º – 90º = 90º (20 − x )(20 + x )
Similarly, ∠3 = 90º 48[20 + x − 20 + x ]
∠2 + ∠3 = 90º + 90º = 180º  =1
400 − x 2
\ AOB is a straight line.
 96x = 400 – x2  x2 + 96x – 400 = 0
Hence AOB is a diameter of the circle with
 x2 + 100x – 4x – 400 = 0
centre O.

 x (x + 100) – 4 (x + 100) = 0
\ AB passes through centre O.
 31. Total number of cards = 18  (x – 4) (x + 100) = 0
Prime numbers are:  x – 4 = 0 or x + 100 = 0
3, 5, 7, 11, 13, 17, 19, 23, 29, 31, 37, i.e., 11  x=4 or x = –100 (rejected)
...[∵ radius cannot be –ve
11
 P(Prime number) = 18
\ Speed of the stream = 4 km/hr
33. Given. In DABC, A

SECTION-D AD ^ BC and 1
E 2P

CE ^ AB
7 3

32. Let the quicker pipe take to fill the cistern


= x minutes To prove. B
6 5 4
C
Then the slower pipe takes to fill the (i) DAPE ~ DCPD
D

cistern = (x + 3) minutes (ii) AP × PD = CP × PE


According to Question, (iii) DADB ~ DCEB
1 x + 3 + x 13
+ 1 = 1  x( x + 3) = 40 (iv) AB × CE = BC × AD
x x + 3 40
13

MATHEMATICS – SAMPLE QUESTION PAPER–01  P-27

Click here to access full book!


Proof (i) In DAPE and DCPD Vol. of solid
∠1 = ∠4 ...[Each 90º = Vol. of hemisphere
∠2 = ∠3 ...[Vertically opposite angles + Vol. of cone
2 3 1 2

\ DAPE ~ DCPD ...[AA similarity = 3 πr + 3 πr h
(ii) \ AP = PE  ...[In ~ ∆s corresponding 3.5 cm
1
CP PD sides are proportional
= 3 πr 2 (2r + h)
3.5 cm
\ AP × PD = CP × PE 1 22 7 7 7
= 3 × 7 × 2 × 2  2 × 2 + 6 
(iii) In DADB and DCEB  
∠5 = ∠7 ...[Each 90º 77 1001
= 6 × 13 = 6
= 166.83 cm3
∠6 = ∠6  ...[Common
\ DADB ~ DCEB ...[AA similarity 35.
...[In ~ ∆s corresponding Monthly No. of Class fi
(iv) \ AB = AD 
CB CE sides are proportional consumption Consumers Intervals
AB × CE = BC × AD (in units) (c.f.) (C.I.)
Below 85 4 Below 85 04
34. 1 km = 1000 m \ 15 km = 15000 m
Below 105 09 85–105 05
14 7 Below 125 22 105–125 13
Radius of pipe = 2 = 7 cm = 100 m
Below 145 42 125–145 20
Volume of pipe (cylinder) = pr2h Below 165 56 145–165 14
\ Vol. of water flowing through the cylidrical Below 185 64 165–185 08

s
Below 205 68 185–205 04

a
pipe in an hr. at the rate of 15 km/hr.

iv D
68
22 7 7
= × × × 15000 = 231 m3 n 68
7 100 100 = th C.I. = h = 20
2 = 34 item;

Sh
2
Speed of water flowing = 15 km/hr 7 cm
Median class is 125–145.
n − c. f .
Median = l + 2
×h
Tank f
34 − 22
= 125 + 20 × 20 = 137 units

SECTION-E
21 m

44 m
36. (i) Initially when time, t = 0 then Annie’s
50 m height, h(t)= 48 ft.
Volume of cuboid (tank) = lbh ⇒ 48 = –16(0)2 + 8(0) + k
\ Volume of required quantity of water in the \ k = 48
21 (ii) When Annie touches the pool, her height,
tank (cuboid) = 50 × 44 × 100 = 462 m3
h = 0 feet
...[Since 21 cm = 21/ m
100 h(t) = –16t2 + 8t + 48
Since 231 m3of water falls into tank in = 1 hr. ⇒ 2t2 – t – 6 = 0
\ 462 m3 of water falls into tank in ⇒ 2t2 – 4t + 3t – 6 = 0
1 ⇒ 2t(t – 2) + 3(t – 2) = 0
= 231 × 462 = 2 hours
⇒ (2t + 3) (t – 2) = 0
Or
7 ⇒ t = 2 or t = - 3
34. Given. Radii, r = 3.5 cm = cm 2
2
 (Since time cannot be negative)
Height of cone = 9.5 – 3.5 = 6 cm
So, t = 2 seconds

P-28  Shiv Das CBSE SECONDARY SERIES (X)

Click here to access full book!


(iii) a + b = 1 and ab = –6 AG
The required polynomial, p(t) = k[t2 – t – 6] 38. (i) In FAG, cos 30º = AF
...(i) F
We know that t = 0, we have p(0) = 48 30º 60º
Using (i), k(0 – 0 – 6) = 48 A 30º
G
⇒ k = –8
60º P
Putting k = –8 in (i), we get –8(t2 – t – 6)

7816 m
H

1930 m
\ Required Polynomial = –8t2 + 8t + 48
Or
D C I Q S
Given. r(t) = –12t2 + (k – 3)t + 48 1973 km
When the zeroes are negative of each
other, Sum of the zeroes = 0 1973
coefficient of x AG 2
Also, Sum of zeroes = - \ AF = =
cos 30º 3
coefficient of x2
(k - 3) −b 2
⇒ - ( -12 ) = 0
...[Sum of zeroes = a 1973
= = 1139.15 km
⇒ k – 3 = 0 \k=3 3

37. (i) Using mid-point formula we have, Or


1 4 4 4 In FPH,

, i.e.,  5 , 4 
 2 2  2  PH

s
(ii) Distance between B(2, 1) and E(7, 5) \ cos 60º = ...[PH is the mid-point of DS

a
PF

D
= (7  2 )2  ( 5  1)2  25  16  41 1973

i v
PH 2

Sh
(iii) (x, y) is equidistant from F(2, 5) and \ PF = cos 60º = 1 = 1973 km
E(7, 5), then 2
(ii) In FAG,
PF = PE
or, PF2 = PE2 FG
tan 30º = AG
(x – 2)2 + (y – 5)2 = (x – 7)2 + (y – 5)2
 x2 – 4x + 4 = x2 – 14x + 49 1
=
FG

⇒ 3 1973
 –4x + 14x + 4 – 49 = 0
 10x – 45 = 0  2x – 9 = 0 2
Or 1973
⇒ FG = = 569.6 km
Let the coordinates of point P be (x, y). 2 3
The ratio in which P divides A(1, 1) and \ FI = GF + GI
B(4, 1) is 1 : 2 = 569.6 km + 7816 km
Using the section formula we have = 577.42 km

x = 41 21  4  2 = 2 P 7816
12 3 (iii) In DACD, tan θ = B = 7816 = 1

y = 11  21  1  2 = 1
12 3
⇒ tan θ = tan 45º
Thus, the coordinates of point Q are (2 : 1)
\ θ = 45º
__________________

MATHEMATICS – SAMPLE QUESTION PAPER–01  P-29

Click here to access full book!


IMPORTANT TERMS AND
FORMULAE (CHAPTER-WISE)

1 Real Numbers
L E T's R E C A L L
☞ R = Real Numbers: All rational and irrational
numbers are called real numbers. All integers can be expressed
as rationals,
5
for example, 5 =
1
☞ I = Integers: All numbers from
(…–3, –2, –1, 0, 1, 2, 3…) are called integers.

Roots of primes like


p 2, 3, 5 etc. are irrational
☞ Q = Rational Numbers: Real numbers of the form q ,
q ¹ 0, p, q Î I are rational numbers.
ALERT!
☞ Q¢
¢ = Irrational Numbers: Real numbers which
Decimal expansion
p
cannot be expressed in the form q and whose of rational numbers is terminating
decimal expansions are non-terminating and non- or non-terminating recurring.
recurring.
☞ N = Natural Numbers: Counting numbers are
Remember this!
called natural numbers.
¨ All Natural Numbers are whole numbers.
N = {1, 2, 3, …}
¨ All Whole Numbers are Integers.
☞ W = Whole Numbers: Zero along with all ¨ All Integers are Rational Numbers.
natural numbers are together called whole ¨ All Rational Numbers are Real Numbers.
numbers.
{0, 1, 2, 3, …}
☞ Even Numbers: Natural numbers of the form 2n are called
even numbers. {2, 4, 6, …} Why can’t we write
☞ Odd Numbers: Natural numbers of the form 2n – 1 are called the form as 2n + 1?
odd numbers. {1, 3, 5, …}

IMPORTANT TERMS AND FORMULAE „ CH-1

Click here to access full book!


☞ Prime Numbers: The natural numbers greater than 1 which are 1 is not a prime
divisible by 1 and the number itself are called prime numbers. number as it has only
Prime numbers have two factors i.e., 1 and the number itself. For one factor.
example, 2, 3, 5, 7 & 11 etc.
☞ Composite Numbers: The natural numbers which are divisible by 1, itself and any other number
or numbers are called composite numbers. For example, 4, 6, 8, 9, 10 etc.
Note: 1 is neither a prime nor a composite number.
What does this mean
MAIN CONCEPTS a b q r
18 = 2 × 9 + 0
I. Euclid’s Division lemma 18 = 3 × 5 + 3
Given two positive integers a and b, there exist unique integers q 18 = 5 × 3 + 3
and r satisfying a = bq + r, 0 £ r < b. 18 = 7 × 2 + 4
18 = 11 × 1 + 7
Notice this. Each time ‘r’ is less than b. Each ‘q’ and ‘r’ is unique.

••
……•

…••
II. Application of lemma
Euclid’s Division lemma is used to find HCF of two positive integers. Example: Find HCF of 56
and 72?
Steps:
• Apply lemma to 56 and 72.
• Take bigger number and locate ‘b’ and ‘r’. 72 = 56 × 1 + 16
• Since 16 ¹ 0, consider 56 as the new 56 = 16 × 3 + 8
dividend and 16 as the new divisor.
• Again, 8 ¹ 0, consider 16 as new dividend 16 = 8 × 2 + 0
and 8 as new divisor.
Since remainder is zero, divisor (8) is HCF.
Although Euclid’s Division lemma is stated for only positive integers, it can be extended for
all integers except zero, i.e., b ¹ 0.
III. Constructing a Factor Tree
Steps Example:
• Write the number as a product of Factorize 48
prime number and a composite number.

• Repeat the process till


all the primes are
obtained.
\ Prime factorization
of 48 = 24 × 3

CH-2 „ SHIV DAS SECONDARY SERIES [X]

Click here to access full book!


IV. Fundamental theorem of Arithmetic
Every composite number can be expressed as a product of primes, and this expression is unique,
apart from the order in which they appear.
Applications:
1. To locate HCF and LCM of two or more positive integers.
2. To prove irrationality of numbers.
3. To determine the nature of the decimal expansion of rational numbers.
1. Algorithm to locate HCF and LCM of two or more positive integers:
Step I: Factorize each of the given positive integers and express them as a product of
powers of primes in ascending order of magnitude of primes.
Step II: To find HCF, identify common prime factor and find the least powers and multiply
them to get HCF.
Step III: To find LCM, find the greatest exponent and then multiply them to get the LCM.
2. To prove Irrationality of numbers:
• The sum or difference of a rational and an irrational number is irrational.
• The product or quotient of a non-zero rational number and an irrational number is
irrational.
3. To determine the nature of the decimal expansion of rational numbers:
• Let x = p/q, p and q are co-primes, be a rational number whose decimal expansion
terminates. Then the prime factorization of ‘q’ is of the form 2m5n, m and n are non-
negative integers.
• Let x = p/q be a rational number such that
the prime factorization of ‘q’ is not of the Alert!
2 3 can be written as: 23 = 2350
m n
form 2 5 , ‘m’ and ‘n’ being non-negative
integers, then x has a non-terminating 52 can be written as: 52 = 2052
repeating decimal expansion.

2 Polynomials
L E T’s R E C A L L
☞ “Polynomial” comes from the word ‘Poly’ (Meaning Many) and ‘nomial’ (in this case
meaning Terms)—so it means many terms.
☞ A polynomial is made up of terms that are only added, subtracted or multiplied.
☞ A quadratic polynomial in x with real coefficients is of the form ax2 + bx + c, where a, b, c are real
numbers with a ¹ 0.
☞ Degree—The highest exponent of the variable in the polynomial is called the degree of polynomial.
Example: 3x3 + 4, here degree = 3.
☞ Polynomials of degrees 1, 2 and 3 are called linear, quadratic and cubic polynomial respectively.
☞ A polynomial can have terms which have Constants like 3, –20, etc., Variables like x and y and
Exponents like 2 in y2.
☞ These can be combined using addition, subtraction and multiplication but NOT DIVISION.

IMPORTANT TERMS AND FORMULAE „ CH-3

Click here to access full book!


☞ The zeroes of a polynomial p(x) are precisely the x-coordinates of the points, where the graph of y
= p(x) intersects the x-axis.
☞ If a and b are the zeroes of the quadratic polynomial ax2 + bx + c, then
−b −Coefficient of x
Sum of zeroes, α + β = a = Coefficient of x 2

c Constant term
Product of zeroes, αβ = a = Coefficient of x 2

☞ If a, b, g are the zeroes of the cubic polynomial ax3 + bx2 + cx + d = 0, then


−b −Coefficient of x 2
a + b + g= a =
Coefficient of x 3
c Coefficient of x
ab + bg + ga = a = Coefficient of x 3

−d −Constant term
and, abg = a = Coefficient of x 3

☞ Zeroes (a, b) follow the rules of algebraic indentities, i.e.,


(a + b)2 = a2 + b2 + 2ab \ (a 2 + b2) = (a + b)2 – 2ab
☞ DIVISION ALGORITHM: If p(x) and g(x) are any two polynomials with g(x) ¹ 0, then
p(x) = g(x) × q(x) + r(x) Dividend = Divisor × Quotient + Remainder
Remember this!
¨ If r (x) = 0, then g (x) is a factor of p (x).
¨ If r (x) ≠ 0, then we can subtract r (x) from p (x) and then the new
polynomial formed is a factor of g(x) and q(x).

3 Pair Of Linear Equations


In Two Variables
MAIN CONCEPTS
☞ For any linear equation, each solution (x, y) corresponds to a point on the line. General form is
given by ax + by + c = 0.
☞ The graph of a linear equation is a straight line.
☞ Two linear equations in the same two variables are called a pair of linear equations in two
variables. The most general form of a pair of linear equations is:
a1x + b1y + c1 = 0; a2x + b2y + c2 = 0
where a1, a2, b1, b2, c1, and c2 are real numbers, such that a12 + b12 ¹ 0, a22 + b22 ¹ 0.
☞ A pair of values of variables ‘x‘ and ‘y’ which satisfy both the equations in the given system of
equations is said to be a solution of the simultaneous pair of linear equations.
☞ A pair of linear equations in two variables can be represented and solved, by

CH-4 „ SHIV DAS SECONDARY SERIES [X]

Click here to access full book!


(i) Graphical method (ii) Algebraic method
(i) Graphical method. The graph of a pair of linear equations in two variables is presented by
two lines.
(ii) Algebraic methods. Following are the methods for finding the solution (s) of a pair of linear
equations:
(i) Substitution method
(ii) Elimination method
(iii) Cross-multiplication method.
☞ There are several situations which can be mathematically represented by two equations that are
not linear to start with. But we allow them so that they are reduced to a pair of linear equations.
☞ Consistent system. A system of linear equations is said to be consistent if it has at least one
solution.
☞ Inconsistent system. A system of linear equations is said to be inconsistent if it has no solution.
CONDITIONS FOR CONSISTENCY
Let the two equations be:
a1x + b1y + c1 = 0
a2x + b2y + c2 = 0
Then,
Relationship between Graph Number of Consistency
coefficients or the pair Solutions of System
of equations
a
1 b
1
1. a ≠ b Intersecting lines Unique solution Consistent
2 2

a
1 1b 1c
2. a = b ≠ c Parallel lines No solution Inconsistent
2 2 2

a
1 1b 1c
3. a = b = c Co-incident lines Infinite solutions Consistent
2 2 2

4 Quadratic Equations
L E T’s R E C A L L
☞ A quadratic polynomial of the form ax2 + bx + c, where a ¹ 0 and a, b, c are real numbers, is called
a quadratic equation
when ax2 + bx + c = 0.
Here a and b are the coefficients of x2 and x respectively and ‘c’ is a constant term.
☞ Any value is a solution of a quadratic equation if and only if it satisfies the quadratic equation.
☞ Quadratic formula: The roots, i.e., a and b of a quadratic equation ax2 + bx + c = 0 are given
−b ± D 2
by or − b ± b − 4ac , provided b2 – 4ac ³ 0.
2a 2a

IMPORTANT TERMS AND FORMULAE „ CH-5

Click here to access full book!


Here, the value b2 – 4ac is known as the discriminant and is generally denoted by D. ‘D’ helps us
to determine the nature of roots for a given quadratic equation. Thus D = b2 – 4ac.
The rules are:
• If D = 0 Þ The roots are Real and Equal.
• If D > 0 Þ The two roots are Real and Unequal.
• If D < 0 Þ No Real roots exist.
☞ If a and b are the roots of the quadratic equation, then Quadratic equation is
x2 – (a + b) x + ab = 0 Or x2 – (sum of roots) x + product of roots = 0
–Coefficient of x –b
where, Sum of roots (a + b) = =
Coefficient of x2 a

Product of roots (a × b) = Constant term c


=
Coefficient of x 2
a

5 Arithmetic Progression
MAIN CONCEPTS
☞ SEQUENCE:
A sequence is an arrangement of numbers in a definite order and according to some rule.
Example: 1, 3, 5, 7, 9, … is a sequence where each successive item is 2 greater than the preceding
term and 1, 4, 9, 16, 25, … is a sequence where each term is the square of successive natural
numbers.
☞ TERMS:
The various numbers occurring in a sequence are called ‘terms’. Since the order of a sequence is
fixed, therefore the terms are known by the position they occupy in the sequence.
Example: If the sequence is defined as
1 3 5 7 9 .. n
¯ ¯ ¯ ¯
First Second Third nth general
term (a1) term (a2) term (a3) term (an) or term

☞ ARITHMETIC PROGRESSION (A.P.):


An Arithmetic progression is a special case of a sequence, where the difference between a term and
its preceding term is always constant, known as common difference, i.e., d. The arithmetic
progression is abbreviated as A.P.
The general form of an A.P. is \ a, a + d, a + 2d,... For example, 7, 9, 11, 13.., Here the common
difference is 2. Hence it is an A.P.
☞ In an A.P. with first term a and common difference d, the nth term (or the general term) is given
by
an = a + (n – 1)d.
...where [a = first term, d = common difference, n = term number
Example: To find seventh term put n = 7 \ a7 = a + (7 – 1)d or a7 = a + 6d

CH-6 „ SHIV DAS SECONDARY SERIES [X]

Click here to access full book!


☞ The sum of the first n terms of an A.P. is given by
n n
Sn = [2a + (n – 1)d] or [a + l]
2 2
where, l is the last term of the finite AP.
a+c
☞ If a, b, c are in A.P. then b = and b is called the arithmetic mean of a and c.
2

6 Triangles
MAIN CONCEPTS
☞ SIMILAR FIGURES
• Two figures having the same shape but not necessarily the same size are called similar figures.
• All congruent figures are similar but all similar figures are not congruent.
☞ SIMILAR POLYGONS
Two polygons are said to be similar to each other, if:
(i) their corresponding angles are equal, and
(ii) the lengths of their corresponding sides are proportional
Example: l1
Any two line segments are similar
l2 since length are proportional

Any two circles are similar since radii


r2
r1 are proportional

Any two squares are similar since


corresponding angles are equal and
lengths are proportional.

Note: Similar figures are congruent if there is one to one correspondence between the figures.
\ From above we deduce: A P

Any two triangles are similar, if their

B C Q R
(i) Corresponding angles are equal –
ÐA = ÐP
ÐB = ÐQ
ÐC = ÐR

IMPORTANT TERMS AND FORMULAE „ CH-7

Click here to access full book!


(ii) Corresponding sides are proportional
AB AC BC
= =
PQ PR QR

☞ THALES’ THEOREM OR BASIC PROPORTIONALITY THEORY


Theorem 1: State and prove Thales’ Theorem.
Statement: If a line is drawn parallel to one side of a triangle to intersect the other two
sides in distinct points, the other two sides are divided in the same ratio.
Given: In DABC, DE || BC. A

AD AE M N
To prove: DB = EC
D E
Const.: Draw EM ^ AD and DN ^ AE. Join B to E and C to D.
Proof: In DADE and DBDE,
ar ( ΔADE) ½ × AD × EM AD B C
= = ....(i) ...[Area of Δ = ½ × base × corresponding altitude
ar ( ΔBDE) ½ × DB × EM DB
In DADE and DCDE,
ar ( ΔADE) ½ × AE × DN AE
= = …(ii)
ar ( ΔCDE) ½ × EC × DN EC
∵ DE || BC …[Given
\ ar(DBDE) = ar(DCDE) …(iii)
∵ Δs on the same base and between the same parallel sides are equal in area
…[∵
AD AE
From (i), (ii) and (iii), DB = EC
☞ CRITERION FOR SIMILARITY OF TRIANGLES
Two triangles are similar if either of the following three criterions are satisfied:
AAA similarity Criterion. If two triangles are equiangular, then they are similar.
Corollary(AA similarity). If two angles of one triangle are respectively equal to two angles of
another triangle, then the two triangles are similar.
SSS Similarity Criterion. If the corresponding sides of two triangles are proportional, then they
are similar.
SAS Similarity Criterion. If in two triangles, one pair of corresponding sides are proportional
and the included angles are equal, then the two triangles are similar.
Results in Similar Triangles based on Similarity Criterion:
(i) Ratio of corresponding sides = Ratio of corresponding perimeters
(ii) Ratio of corresponding sides = Ratio of corresponding medians
(iii) Ratio of corresponding sides = Ratio of corresponding altitudes
(iv) Ratio of corresponding sides = Ratio of corresponding angle bisector segments.
☞ AREA OF SIMILAR TRIANGLES
Theorem 2. The ratio of the areas of two similar triangles is equal to the square of the
ratio of their corresponding sides.
Given: DABC ~ DDEF
ar ( ΔABC) AB 2 BC 2 AC 2
To prove: ar ( ΔDEF) = = =
DE2 EF 2 DF 2
Const.: Draw AM ^ BC and DN ^ EF.

CH-8 „ SHIV DAS SECONDARY SERIES [X]

Click here to access full book!


Proof: In DABC and DDEF A D

ar ( ΔABC) ½ × BC × AM BC AM
= = . …(i)
ar ( ΔDEF) ½ × EF × DN EF DN
...[Area of Δ = ½ × base × corresponding altitude
∵ DABC ~ DDEF …[Given
AB BC B M C E N F
\ DE = EF …(ii) …[Sides are proportional
Ð –B = Ð E …[∵
∵ D ABC ~ D DEF
ÐM = ÐN …[each 90°
\ DABM ~ DDEN …[AA Similarity
AB AM
\ DE = DN …(iii) ...[Sides are proportional

BC AM
From (ii) and (iii), we have: EF = DN …(iv)
ar ( ΔABC) BC BC BC 2
From (i) and (iv), we have: ar ( ΔDEF) = EF . EF =
EF 2
Similarly, we can prove that
ar ( ΔABC) AB 2 AC 2 ar (ΔABC) AB2 BC 2 AC 2
= = \ = = =
ar ( ΔDEF) DE2 DF 2 ar (ΔDEF) DE2 EF 2 DF 2
Results based on Area Theorem:
(i) Ratio of areas of two similar triangles = Ratio of squares of corresponding altitudes
(ii) Ratio of areas of two similar triangles = Ratio of squares of corresponding medians
(iii) Ratio of areas of two similar triangles = Ratio of squares of corresponding angle bisector
segments.
Note: If the areas of two similar triangles are equal, the triangles are congruent.

☞ PHYTHAGORAS’ THEOREM
Theorem 3: State and prove Pythagoras’ Theorem.
Statement: Prove that, in a right triangle, the square of the hypotenuse is equal to the
sum of the squares of the other two sides. C
Given: DABC is a right triangle right-angled at B.
To prove: AB2 + BC2 = AC2
Const.: Draw BD ^ AC D
Proof: In Ds ABC and ADB,
ÐA = ÐA …[common
Ð ABC = Ð ADB …[each 90°
\ DABC ~ DADB …[AA Similarity A B
AB AC
\ AD = AB ...[Sides are proportionals

• Þ AB2 = AC.AD …(i)


Now in DABC and DBDC
ÐC = ÐC …[common
Ð ABC = Ð BDC …[each 90°
\ DABC ~ DBDC …[AA Similarity
BC AC
\ DC = BC …[Sides are proportional

IMPORTANT TERMS AND FORMULAE „ CH-9

Click here to access full book!


BC2 = AC.DC …(ii)
On adding (i) and (ii), we get
AB2 + BC2 = AC.AD + AC.DC Þ AB2 + BC2 = AC.(AD + DC)
AB2 + BC2 = AC.AC \ AB2 + BC2 = AC2
☞ CONVERSE OF PHYTHAGORAS’ THEOREM
Theorem 4: State and prove the converse of Pythagoras’ Theorem.
Statement: Prove that, in a triangle, if square of one side is equal to the sum of the
squares of the other two sides, then the angle opposite the first side is a right angle.
Given: In DABC, AB2 + BC2 = AC2 C F
To prove: Ð ABC = 90°
Const.: Draw a right angled DDEF in
which DE = AB and EF = BC
Proof: In DABC,
AB2 + BC2 = AC2 …(i) [Given
In rt. DDEF
DE2 + EF2 = DF2 …[by Pythagoras’ Theorem A B D E
2 2
AB + BC = DF 2 …(ii) …[∵ DE = AB; EF = BC
From (i) and (ii), we get
AC2 = DF2 •Þ AC = DF
Now, DE = AB …[by const.
EF = BC …[by const.
DF = AC …[Proved above
\ DDEF @ DABC …[SSS congruence
\ Ð DEF = Ð ABC …[C.P.C.T
Ð DEF = 90° A
…[by const.
\ Ð ABC = 90°
Results based on Pythagoras’ Theorem:
(i) Result on obtuse Triangles.
If DABC is an obtuse angled triangle, obtuse angled at B,
If AD ^ CB, then
AC2 = AB2 + BC2 + 2 BC.BD D C
B
A
(ii) Result on Acute Triangles.
If DABC is an acute angled triangle, acute angled
at B, and AD ^ BC, then
AC2 = AB2 + BC2 – 2 BD.BC.
B C
D

7 Co-ordinate Geometry
L E T’S R E C A L L
☞ Position of a point P in the Cartesian plane with respect to co-ordinate axes is represented by the
ordered pair (x, y).

CH-10 „ SHIV DAS SECONDARY SERIES [X]

Click here to access full book!


☞ The line X¢OX is called the X-axis and YOY¢is called the Y-axis.
☞ The part of intersection of the X-axis and Y-axis is called the origin
O and the co-ordinates of O are (0, 0).
☞ The perpendicular distance of a point P from the Y-axis is the ‘x’
co-ordinate and is called the abscissa.
☞ The perpendicular distance of a point P from the X-axis is the ‘y’
co-ordinate and is called the ordinate.

☞ Signs of abscissa and ordinate in different quadrants are as


given in the diagram:

☞ Any point on the X-axis is of the form (x, 0).


☞ Any point on the Y-axis is of the form (0, y).
☞ The distance between two points P(x1, y1) and Q (x2, y2) is given by
PQ = (x2 − x1 )2 + ( y 2 − y1 )2
Note. If O is the origin, the distance of a point P(x, y) from the origin O(0, 0) is given by

OP = x2 + y2
☞ Section formula. The coordinates of the point which divides the line segment joining the points
A(x1, y1) and B(x2, y2) internally in the ratio m : n are:
m:n
A(x1, y1) B(x2, y2)
P(x, y)

P(x, y) = ( mx2 + nx1 my 2 + ny1


m+n
,
m+n )
m
The above formula is section formula. The ratio m : n can also be written as : 1 or k : 1, The
n
kx2 + x1 ky2 + y1
co-ordinates of P can also be written as P(x,y) = , .
k +1 k+1
☞ The mid-point of the line segement joining the points P(x1, y1) and Q(x2, y2) is
A(x, y)
P(x1, y1) Q(x2, y2)

A(x, y) = ( x1 + x2 y1 + y 2
2
,
2 )
Here m : n = 1 : 1.
☞ Area of a Triangle. The area of a triangle formed by points A(x1, y1), B(x2, y2) and C(x3, y3) is
given by |D |,
1
where D = [ x1(y2 – y3) + x2(y3 – y1) + x3(y1 – y2)]
2
where D represents the absolute value.

IMPORTANT TERMS AND FORMULAE „ CH-11

Click here to access full book!


• Three points are collinear if |D| = 0.
• If P is centroid of a triangle then the median divides it in the ratio 2 : 1.
Co-ordinates of P are given by

P= ( x1 + x2 + x3 y1 + y 2 + y 3
3
,
3 )
☞ Area of a quadrilateral, ABCD = ar(DABC) + ar(DADC)

8 Introduction to Trigonometry
L E T’S R E C A L L
☞ Trigonometry is the science of relationships between the sides and angles of a right-angled
triangle.
☞ Trigonometric Ratios: Ratios of sides of right triangle are called trigonometric ratios.
Consider triangle ABC right-angled at B. These ratios are always defined with respect to acute
angle ‘A’ or angle ‘C’.
☞ If one of the trigonometric ratios of an acute angle is known, the remaining trigonometric ratios of
an angle can be easily determined.
☞ How to identify sides: Identify the angle with respect to which the t-ratios have to be calculated.
Sides are always labelled with respect to the ‘q
q’ being considered.
Let us look at both cases:

A A

q
Perpendicular
Adjacent side

Opposite side

H H
yp yp
ot
Base

en ot
en
us us
e e

B C q
B C
Opposite side Adjacent side
Perpendicular Base

Case I: ∠A = θ Case II: ∠C = θ

In a right triangle ABC, right-angled at B. Once we have identified the sides, we can define
six t-Ratios with respect to the sides.
Case I Case II
Perpendicular BC Perpendicular AB
(i) sine A = Hypotenuse = AC (i) sine C = Hypotenuse = AC

CH-12 „ SHIV DAS SECONDARY SERIES [X]

Click here to access full book!


Base AB Base BC
(ii) cosine A = Hypotenuse = AC (ii) cosine C = Hypotenuse = AC
Perpendicular BC Perpendicular AB
(iii) tangent A = = (iii) tangent C = =
Base AB Base BC
Hypotenuse AC Hypotenuse AC
(iv) cosecant A = Perpendicular = BC (iv) cosecant C = Perpendicular = AB
Hypotenuse AC Hypotenuse AC
(v) secant A = = (v) secant C = =
Base AB Base BC
Base AB Base BC
(vi) cotangent A = Perpendicular = BC (vi) cotangent C = Perpendicular = AB
Note from above six relationships:
1 1 1
cosecant A = sin A , secant A = , cotangent A = tangent A
cosine A
However, it is very tedious to write full forms of t-ratios, therefore the abbreviated
notations are:
sine A is sin A cosine A is cos A tangent A is tan A
cosecant A is cosec A secant A is sec A cotangent A is cot A
☞ TRIGONOMETRIC IDENTITIES
An equation involving trigonometric ratio of angle(s) is called a trigonometric identity, if it is true
for all values of the angles involved. These are:
sin θ cos θ
tan θ = cos θ cot θ =
sin θ
• sin2 q + cos2 q = 1 Þ• sin2 q = 1 – cos2 q Þ cos2 q = 1 – sin2 q
• cosec2 q – cot2 q = 1 Þ cosec2 q = 1 + cot2 q Þ cot2 q = cosec2 q – 1
• sec2 q – tan2 q = 1 Þ sec2 q = 1 + tan2 q Þ tan2 q = sec2 q – 1
• sin q cosec q = 1 Þ cos q sec q = 1 Þ tan q cot q =1
C
☞ ALERT: A t-ratio only depends upon the angle ‘q’ and stays the same for same
angle of different sized right triangles.
Value of t-ratios of specified angles: A B

ÐA 0° 30° 45° 60° 90°


1 1 3
sin A 0 1
2 2 2
3 1 1
cos A 1 0
2 2 2
1
tan A 0 1 3 Not defined
3

2
cosec A Not defined 2 2 1
3

2
sec A 1 2 2 Not defined
3
1
cot A Not defined 3 1 0
3

IMPORTANT TERMS AND FORMULAE „ CH-13

Click here to access full book!


The value of sin q and cos q can never exceed 1 (one) as opposite side is 1. Adjacent side can never
be greater than hypotenuse since hypotenuse is the longest side in a right-angled D.
☞ ‘t-RATIOS’ OF COMPLEMENTARY ANGLES A
If DABC is a right-angled triangle, right-angled at B, then
∵ ∠ A+ ∠ B+ ∠ C = 180 °
Ð A + Ð C = 90° ...
angle-sum-property
or Ð C = (90° – Ð A) B C
Thus, Ð A and Ð C are known as complementary angles and are related by the following
relationships:
sin (90° – A) = cos A; cosec (90° – A) = sec A
cos (90° – A) = sin A; sec (90° – A) = cosec A
tan (90° – A) = cot A; cot (90° – A) = tan A

9 Applications of Trigonometry
L E T’S R E C A L L
☞ Line of Sight t •O
igh
When an observer looks from a point E (eye) at an object O then e of S
Lin
the straight line EO between the eye E and the object O is called
the line of sight. •
E
☞ Horizontal
When an observer looks from a point E (eye) to another point Q Line of Sight
which is horizontal to E, then the straight line, EQ between E • •
E Q
and Q is called the horizontal line.
☞ Angle of Elevation O
ht
When the eye is below the object, then the observer has to look fS
ig
up from the point E to the object O. The measure of this rotation eo
Lin
(angle q) from the horizontal line is called the angle of q Angle of Elevation
Q
elevation. E Horizontal
☞ Angle of Depression Q
q Angle of Depression
When the eye is above the object, then the observer has to look
down from the point E to the object. The horizontal line is now
parallel to the ground. The measure of this rotation (angle q)
from the horizontal line is called the angle of depression. O
☞ How to convert above figure into right triangle. O
Case I: Angle of Elevation is known ht
Draw OX perpendicular to EQ. Now Ð OXE = 90° f Sig
o
DOXE is a rt. D, where e
Lin
OE = hypotenuse Angle of Elevation

OX = opposite side (Perpendicular) E Horizontal X Q


EX = adjacent side (Base)
CH-14 „ SHIV DAS SECONDARY SERIES [X]

Click here to access full book!


Case II: Angle of Depression is known Q Horizontal E
(i) Draw OQ¢parallel to EQ o n
ssi
(ii) Draw perpendicular EX on OQ¢. De
pre
of
(iii) Now Ð QEO = Ð EOX = Interior alternate angles gle
An
\ DEXO is a rt. D. Angle of Depression
...where [EO = hypotenuse, OX = adjacent side (base), EX = opposite side O Parallel Horizontal X Q′
(Perpendicular)

M U S T R E M E M B E R
☞ Choose a trigonometric ratio in such a way that it considers the
known side and the side that you wish to calculate.
☞ The eye is always considered at ground level unless the problem
specifically gives the height of the observer.
☞ Object is always considered as a point.
Some People Have Curly Black Hair Turning Permanent Black.
P erpendicular B ase P erpendicular
sin q = H ypotenuse cos q = H yptoenuse tan q =
B ase

10 Circles
L E T’S R E C A L L
☞ Circle. A circle is a collection of all points in a plane which are at a constant distance from a fixed
point.
☞ Centre. The fixed point is called the centre.
☞ Radius. The constant distance from the centre is called the radius.
☞ Chord. A line segment joining any two points on a circle is called a chord.
☞ Diameter. A chord passing through the centre of the circle is called diameter. It is the longest
chord.

MAIN CONCEPTS
☞ Tangent. When a line meets the circle at one point or two coinciding points. The line is known as
a tangent.

☞ The tangent to a circle is perpendicular to the radius through the point of O


contact.
Þ OP ^ AB A B
P

B
☞ The lengths of the two tangents from an external point to a circle are
equal. O P
Þ AP = PB
A

IMPORTANT TERMS AND FORMULAE „ CH-15

Click here to access full book!


LENGTH OF TANGENT SEGMENT
PB and PA are normally called the lengths of tangents from outside point P.
PROPERTIES OF TANGENT TO CIRCLE
Theorem 1: Prove that the tangent at any point of a circle is perpendicular to the radius
through the point of contact.
Given: XY is a tangent at point P to the circle with centre O.
To prove: OP ^ XY O
Const.: Take a point Q on XY other than P and join OQ
Proof: If point Q lies inside the circle, then XY will become a
> >
secant and not a tangent to the circle X P Q Y
\ OQ > OP
This happens with every point on the line XY except the point P
OP is the shortest of all the distances of the point O to the points of XY
\ OP ^ XY …[∵
∵ Shortest side is the perpendicular

Theorem 2: A line drawn through the end point of a radius and perpendicular to it, is the
tangent to the circle.
Given: A circle C(O, r) and a line APB is perpendicular to
OP, where OP is radius. O
To prove: AB is tangent at P.
Construction: Take a point Q on the line AB, different from >•
Q •B
>
P and join OQ. A P
Proof: Since OP ^ AB
\ OP < OQ Þ OQ > OP ...[Given
\ The point Q lies outside the circle.
Therefore, every point on AB, other than P, lies outside the circle. This shows that AB meets the
circle at the point P.
Hence, AP is a tangent to the circle at P.
Theorem 3. Prove that the lengths of tangents drawn from an external point to a circle are
equal.
Given: PT and PS are tangents from an external point P to the circle with centre O.
To prove: PT = PS T
Const.: Join O to P, T and S.
Proof: In DOTP and DOSP.
P O
OT = OS …[radii of same circle
OP = OP …[common
Ð OTP = Ð OSP …[each 90°
S
\ DOTP @ DOSP …[R.H.S.
\ PT = PS …[c.p.c.t.
Note: If two tangents are drawn to a circle from an external point, then: P
(i) They subtend equal angles at the centre i.e., Ð 1 = Ð 2. 1
(ii) They are equally inclined to the segment joining the centre
3 O
A 4 2
to that point i.e., Ð 3 = Ð 4.
Ð OAP = Ð OAQ Q

CH-16 „ SHIV DAS SECONDARY SERIES [X]

Click here to access full book!


12 Areas Related To Circles
MAIN CONCEPTS AND FORMULAE
☞ Circumference of a circle = 2pr
☞ Area of a circle = pr2 ...[where ‘r’ is the radius of circle
πr 2
☞ Area of a semi-circle =
2
☞ Area of a circular path or ring:
Let ‘R’ and ‘r’ be radii of two circles R
Then area of shaded part = pR2 – pr2 r
= p(R2 – r2)
= p(R + r) (R – r)
☞ Minor arc and major arc: An arc length is called a major arc, if the arc length enclosed by the
two radii is greater than a semi circle.
If the arc subtends angle ‘q q’ at the centre, then the
θ θ
Length of minor arc = × 2pr = × pr
360 180

(360º – q)
O
O
q q

A B A B
Minor Arc Major Arc

360 − θ
Length of major arc = 360 × 2pr

☞ SECTOR OF A CIRCLE AND ITS AREA


A region of a circle is enclosed by any two radii and the arc intercepted between two radii is called
the sector of a circle.
(i) A sector is called a minor sector if the minor arc of the circle is part of its boundary.
\ OAB is minor sector.

Major Sector
O

A B
Minor Sector

IMPORTANT TERMS AND FORMULAE „ CH-17

Click here to access full book!


θ θ
Area of minor sector = . (pr2); Perimeter of minor sector = 2r + 360 (2pr)
360
(ii) A sector is called a major sector if the major arc of the circle is part of its boundary.
\ OACB is major sector
360 − θ 360 − θ
Area of major sector = 360 (pr2); Perimeter of major sector = 2r + 360 (2pr)
☞ Minor Segment. The region enclosed by an arc and a chord is called a segment of the circle. The
region enclosed by the chord PQ & minor arc PRQ is called the minor segment.
\ Area of Minor segment S
Major
= Area of corresponding sector – Area of corresponding triangle Segment
O
θ 2 2 1
= 360 π r − 2 r sin θ r q r
P Q
1 2 θ 1 2 θ θ θ R
= 2 r 180 π − sin θ or 2 r 180 π − 2 sin 2 cos 2 Minor
Segment
☞ Major Segment: The region enclosed by the chord PQ & major arc PSQ is called the major segment.
\ Area of major segment = Area of circle – Area of minor segment
θ 1 2 θ sin θ
Area of major sector + Area of triangle = πr 2 − π r 2 + r 2 sin θ = r π − 360 π + 2
360 2
TABLE FOR AREA AND PERIMETER
Figures Area Perimeter

πd2
Circle pr2 or 2pr or pd r : radius
r
4
O
d : diameter
22
p= or 3.14
7
πr 2
Semicircle pr + 2r
2
r

πr 2 πr
Quadrant + 2r
4 2
r

Ring p(R + r) (R – r) 2pR (Outer circu- R : Radius of


R
mference) bigger circle
r 2pr (Inner circum- r : Radius of
ference) smaller circle

θ θ
Sector (i) × pr2 × 2pr + 2r r : Radius of circle
r q
r
360 360
1
(ii) lr l : length of arc
2
l

θ 1 π rθ θ
Segment pr2 – r2 sin q + 2r sin q : angle subtended
O 360 2 180 2
r
q
r
by arc at centre

CH-18 „ SHIV DAS SECONDARY SERIES [X]

Click here to access full book!


13 Surface Area and Volume
L E T’S R E C A L L
TABLE FOR SURFACE AREA AND VOLUME
Solid Figures Curved Plane area (2) Total area Volume Remarks
surface area (1) [1 + 2]
Cuboid Also known as Area of: 2(lb + bh + hl) l.b.h l : length
h lateral surface Top face = lb b : breadth
b area Bottom face = lb h : height
l
= 2(lh + bh) \ lb + lb = 2lb
Cube Lateral surface Area of: 4a2 + 2a2 = 6a2 a3 a : Side of cube
area = 4a2 Top face = a2
Bottom face = a2
a
a
\ a2 + a2 = 2a2
a

Right circu- Curved surface Area of: 2pr2 + 2prh pr2h r : radius
lar cylinder area = 2prh Top face = p r2
Or, h : height of cylinder
closed at top Bottom face = p r2 2pr(r + h)
h

\ pr2 + pr2 = 2pr2


r

Right circu- Curved surface Area of: 2prh + pr2 pr2h r : radius
lar cylinder area = 2prh Top face = 0 Or, h : height of cylinder
open at top Bottom face = pr2 pr(2h + r)
\ 0 + pr2 = pr2
Hollow r 2pRh Area of: 2pRh + 2prh + pR2h – R : Radius of outer base
cylinder • External sur- Top face 2p(R2 – r2) pr2h r : radius of inner base
(Pipe) face area = 2pRh = p(R2 – r2) (External h = height
R • Internal sur- Bottom face Vol. –
= p(R2 – r2) Internal
face area = 2prh
Vol.)
Cone prl Area of: pr2 + prl 1 2 h = height of cone
Bottom Face = pr2 Or, pr(r + l) 3
pr h r = radius of cone
l
h
l = slant height
r
= h2 + r 2
Frustum r pl(R + r) Area of: pr2 + pR2 1 h = height of frustum
ph .
Top Face = pr2 + pl(R + r) 3 r = radius of top face
h l
Bottom Face = pR2 (R2 + r2
R R = Radius of base
+ Rr)
l = slant height
Sphere 4pr2 None 4pr2 4 3 r : radius of sphere
3
pr
r

Hemisphere r 2pr2 pr2 3pr2 2 3 r : radius of


3
pr
hemisphere
Spherical 4pR2 (Outer) None 4 R:Radius of outer shell
4pR2 + 4pr2
shell r 4pr2 (Inner) 3
p.
r:Radius of inner shell
R (R3 – r3)

IMPORTANT TERMS AND FORMULAE „ CH-19

Click here to access full book!


SURFACE AREA AND VOLUME OF COMBINATIONS
(A) Cone on a Cylinder: h1

r : radius of cone & cylinder; h1 = height of cone; h2 = height of cylinder r


Total Surface area = Curved surface area of cone + Curved surface area of cylinder +
area of circular base h2

= prl + 2prh2 + pr2; Slant height, l = r 2 + h12 r


1/ pr2h Circus tent
Total Volume = Volume of cone + Volume of cylinder = 3 1 + pr2h2
(B) Cone on a Hemisphere:
h : height of cone; l : slant height of cone = r 2 + h2 r = radius of cone and hemisphere l
h
Total Surface area = Curved surface area of cone + Curved surface area of hemisphere
= prl + 2pr2 r
Volume = Volume of cone + Volume of hemisphere = 1/3 pr2h + 2/3 pr3
r

(C) Conical Cavity in a Cylinder:


r : radius of cone and cylinder; h : height of cylinder and conical cavity; l : Slant height r
Total Surface area
= Curved surface area of cylinder + Area of bottom face of cylinder h
+ Curved surface area of cone = 2prh + pr2 + prl
r

Volume = Volume of cylinder – Volume of cone = pr2h – 1/3 pr2h = 2/3 pr2h
(D) Cones on Either Side of Cylinder:
r : radius of cylinder and cone; h1 : height of cylinder h2 : height of cones h2 h1 h2

Slant height of cone, l = h22 + r 2 r r


Surface area = Curved surface area of 2 cones + Curved surface area of cylinder
= 2prl + 2prh1
2
Volume = 2(Volume of cone) + Volume of cylinder = 3 pr2h2 + pr2h1

(E) Cylinder with Hemispherical Ends:


r : radius of cylinder and hemispherical ends; h : height of cylinder
Total surface area= Curved surface area of cylinder
+ Curved surface area of 2 hemispheres r r

= 2prh + 4pr2 h
4
Volume = Volume of cylinder + Volume of 2 hemispheres = pr2h + 3 pr3

(F) Hemisphere on Cube or Hemispherical Cavity on Cube:


r
a : side of cube; r : radius of hemisphere.
Surface area = Surface area of cube – Area of hemisphere face
+ Curved surface area of hemisphere a
= 6a2 – pr2 + 2pr2 = 6a2 + pr2
4
Volume= Volume of cube + Volume of hemisphere = a3 + 3 pr3

(G) Hemispherical Cavity in a Cylinder: r


r : radius of hemisphere; h : height of cylinder
Total surface area = Curved surface area of cylinder + Surface area of base
+ Curved surface area of hemisphere h
= 2prh + pr2 + 2pr2 = 2prh + 3pr2
2
Volume = Volume of cylinder – Volume of hemisphere = pr2h – pr3
3

CH-20 „ SHIV DAS SECONDARY SERIES [X]

Click here to access full book!


14 Statistics
MAIN CONCEPTS AND FORMULAE
☞ MEAN (AVERAGE): Mean [Ungrouped Data]—Mean of n observations, x1, x2, x3, … xn, is
1 x + x + x + ... + x
2 3 n 1 x
X = = x \ X = n
n n
☞ MEAN [Grouped Data]. The mean for grouped data can be found by the following three methods:
f i xi
(i) Direct Mean Method: X = fi
Upper class limit + Lower class limit Note : Frequency of a class is centred
Class Mark = ... at its mid-point called class mark.
2
(ii) Assumed Mean Method: In this, an arbitrary mean ‘a’ is chosen which is called, ‘assumed
mean’, somewhere in the middle of all the values of x.
f i di
X =a+ fi ...[where di = (xi – a)
(iii) Step Deviation Method:
f i ui di
X =a+ fi ×h ...[where ui =
h
, where h is a common divisor of di

☞ MEDIAN: Median is a measure of central tendency which gives the value of the middle-most
observation in the data.
th
n+1
(i) Ungrouped data: If n is odd ®– Median = 2 observation
th th
n n
observation + +1 observation
If n is even ® Median = 2 2
2
Remember! For ungrouped data, first arrange the observations in ascending order or descending order.
n
− c. f .
(ii) Median (Grouped Data): Median = l + 2 ×h
f
...where[l = Lower limit of median class; n = Number of observations; f = Frequency of median class;
c.f. = Cumulative frequency of preceding class; h = Class size
(iii) Representing a cumulative frequency distribution graphically as a cumaltive frequency
curve, or an ogive of the less than type and of the more than type. The median of grouped data
can be obtained graphically as the x-coordinate of the point of intersection of the two ogives
for this data.
☞ Mode:(i) Ungrouped Data: The value of the observation having maximum frequency is the mode.
f1 − f0
(ii) Grouped Data: Mode = l + 2 f1 − f0 − f2 ×h
...where[l = Lower limit of modal class; f1 = Frequency of modal class; f0 = Frequency of the class
preceding the modal class; f2 = Frequency of the class succeeding the modal class; h = Size of
class interval
c.f. = Cumulative frequency of preceding class; h = Class size
Mode + 2 Mean 3Median − Mode
☞ 1. Mode = 3 Median – 2 Mean 2. Median = 3
3. Mean =
2

IMPORTANT TERMS AND FORMULAE „ CH-21

Click here to access full book!


15 Probability
MAIN CONCEPTS
☞ PROBABILITY: It is the numerical measurement of the degree of certainty.
1. Theoretical probability associated with an event E is defined as “If there are ‘n’ elementary events
associated with a random experiment and m of these are favourable to the event E then the
m
probability of occurence of an event is defined by P(E) as the ratio ”.
n
Number of outcomes favourable to E m
P(E) = Number of all possible outcomes of the experiment . Thus, P(E) =
n
2. If P(E) = 1, then it is called a ‘Certain Event’.
3. If P(E) = 0, then it is called an ‘Impossible Event’.
4. The probability of an event E is a number P(E) such that: 0 £ P(E) £ 1
5. An event having only one outcome is called an elementary event. The sum of the probabilities
of all the elementary events of an experiment is 1.
6. For any event E, P(E) + P( E) = 1, where E stands for ‘not E’. E and E are called
complementary events.
7. Favourable outcomes are those outcomes in the sample space that are favourable to the
occurrence of an event.
☞ SAMPLE SPACE
A collection of all possible outcomes of an experiment is known as sample space. It is denoted by ‘S’
and represented in curly brackets.
Examples of Sample Spaces:
1. A coin is tossed = Event
\ E1 = Getting a head (H) on upper face; E2 = Getting a tail (T) on upper face
\ S = {H, T} \ Total number of outcomes = 2
2. Two coins are tossed = Event = E
E1 = Getting a head on coin 1 and a tail on coin 2 = (H, T)
E2 = Getting a head on both coin 1 and coin 2 = (H, H)
E3 = Getting a tail on coin 1 and a head on coin 2 = (T, H)
E4 = Getting a tail on both, coin 1 and coin 2 = (T, T)
\ S = {(H, T), (H, H), (T, H), (T, T)}. \ Total number of outcomes = 4
NOTE: In probability the order in which events occur is important. \ E1 &E3 are treated as different outcomes.
☞ IMPORTANT TIPS
1. Coin. A coin has two faces termed as Head and Tail.
2. Dice. A dice is a small cube which has between one to six spots or numbers on its sides, which
is used in games.
3. Cards. A pack of playing cards consists of four suits called Hearts, Spades, Diamonds and
Clubs. Each suit consists of 13 cards.

❖❖❖❖❖

CH-22 „ SHIV DAS SECONDARY SERIES [X]

Click here to access full book!


Series : SDS/01 Code No. 041
Candidates must write the Code on
Roll No. the title page of the answer-book.

• Please check that this question paper contains 4 printed pages.


• Code number given on the right hand side of the question paper should be
written on the title page of the answer-book by the candidate.
• Please check that this question paper contains 30 questions.
• Please write down the Serial Number of the question before attempting it.
• 15 minutes time has been allotted to read this question paper. The question
paper will be distributed at 10.15 a.m. From 10.15 a.m. to 10.30 a.m., the students
will read the question paper only and will not write any answer on the answer-
book during this period.

MATHEMATICS
CBSE–2015 (Delhi)
Time allowed: 3 hours Maximum Marks: 80
GENERAL INSTRUCTIONS:
(i) All questions are compulsory.
(ii) This question paper contains of 30 questions.
(iii) Question No. 1-6 in Section A are very short answer type questions carrying 1 mark each.
(iv) Question No. 7-12 in Section B are short answer type questions carrying 2 marks each.
(v) Question No. 13-22 in Section C are long answer-I type questions carrying 3 marks each.
(vi) Question No. 23-30 in Section D are long answer-II type questions carrying 4 marks
each.

SECTION A
Questions number 1 to 6 carry 1 mark each.
Q.1. In DDEW, AB || EW. If AD = 4 cm, DE = 12 cm and DW = 24 cm, then find the value of DB.
2
Q.2. If 3 sin q = cos q, find the value of 3 cos q + 2 cos q .
3 cos q + 2
1
Q.3. If x = – , is a solution of the quadratic equation 3x2 + 2kx – 3 = 0, find the value of k.
2
Q.4. The tops of two towers of height x and y, standing on level ground, subtend angles of 30°
and 60° respectively at the centre of the line joining their feet, then find x : y.
Q.5. A letter of English alphabet is chosen at random. Determine the probability that the chosen
letter is a consonant.

MATHEMATICS—2015 (DELHI)  1

Click here to access full book!


Q.6. In Fig., PA and PB are tangents to the circle with centre O such A

that –APB = 50°. Write the measure of –OAB.


P 50° O

SECTION B Fig. 1 B
Questions number 7 to 12 carry 2 marks each.
Q.7. Explain why (17 × 5 × 11 × 3 × 2 + 2 × 11) is a composite number?
Q.8. X and Y are points on the sides AB and AC respectively of a triangle ABC such that
AX 1
= , AY = 2 cm and YC = 6 cm. Find whether XY || BC or not. B
AB 4
Or
Q.8. In Fig., AB is the diameter of a circle with centre O and AT is a tangent. O
If –AOQ = 58°, find –ATQ. 58º
Q
3 3
Q.9. Prove the identity: sin q + cos q = 1 – sin q . cos q. A T
sin q + cos q Fig. 2
Q.10. Solve the quadratic equation for x: 4x2
– 4a2x
+ (a4
— b4)= 0.
Q.11. Find the middle term of the A.P. 6, 13, 20, ..., 216.
Q.12. If A(5, 2), B(2, –2) and C(–2, t) are the vertices of a right angled triangle with –B = 90°, then
find the value of t.

SECTION C
Questions number 13 to 22 carry 3 marks each.
Q.13. The length, breadth and height of a room are 8 m 50 cm, 6 m 25 cm and 4 m 75 cm
respectively. Find the length of the longest rod that can measure the dimensions of the
room exactly.
Q.14. The sum of the digits of a two digit number is 8 and the difference between the number
and that formed by reversing the digits is 18. Find the number. E
Q.15. In the Fig., EB ^ AC, BG ^ AE and CF ^ AE F
Prove that: G D
BC BE
(a) DABG ~ DDCB (b) =
BD BA A C
sin 2 q − cos 2q 1 Fig. 3 B
12
Q.16. If sin q = , 0º < q < 90º, find the value of: 2 sin q. cos q ×
13 tan 2 q
Q.17. The average score of boys in the examination of a school is 71 and that of the girls is 73.
The average score of the school in the examination is 71.8. Find the ratio of number of boys
to the number of girls who appeared in the examination.
Q.18. Find the other coordinates of ΔABC with A(1, –4) and mid-points of sides through A being
(2, –1) and (0, –1).
Q.19. Find that non-zero value of k, for which the quadratic equation kx2 + 1 – 2(k – 1)x + x2 = 0
has equal roots. Hence find the roots of the equation.
Q.20. Two different dice are rolled together. Find the probability of getting:
(i) the sum of numbers on two dice to be 5. Q

(ii) even numbers on both dice.


Q.21. In Fig. APB and AQO are semicircles, and AO = OB. If the perimeter A O B

of the figure is 40 cm, find the area of the shaded region.


[Use p = 22/7]
P
Fi 4
2  SHIV DAS CBSE SECONDARY SERIES (X)

Click here to access full book!


7 cm

Q.22. In Fig., from a cuboidal solid metallic block, of dimensions 15


cm × 10 cm × 5 cm, a cylindrical hole of diameter 7 cm is drilled

5 cm
A
out. Find the surface area of the remaining block.
22
[Use p = ]
7 B cm
10
SECTION D 15 cm
Fi
Questions number 23 to 30 carry 4 marks each.
Q.23. Not in Current Syllabus. A

3 cm 5 cm
Q.24. In the given figure AD = 3 cm, AE = 5 cm, BD = 4 cm,
CE = 4 cm, CF = 2 cm, BF = 2.5 cm, find the pair of parallel lines D E
and hence their lengths. 4 cm 4 cm

Q.25. Prove that: B C


sec 3 A − cosec3A 2.5 cm F 2 cm
(1 + cot A + tan A).(sin A – cos A) = 2 2 .
sec A. cosec A
Q.26. In a class test, marks obtained by 120 students are given in the following frequency distri-
bution. If it is given that mean is 59, find the missing frequencies x and y.
Marks 0–10 10–20 20–30 30–40 40–50 50–60 60–70 70–80 80–90 90–100
No. of
students 1 3 7 10 15 x 9 27 18 y
Q.27. The numerator of a fraction is 3 less than its denominator. If 2 is added to both the
numerator and the denominator, then the sum of the new fraction and original fraction is
29
. Find the original fraction.
20
Or
Q.27. Ramkali required `2500 after 12 weeks to send her daughter to school. She saved `100 in
the first week and increased her weekly saving by `20 every week. Find whether she will
be able to send her daughter to school after 12 weeks.

S R
Q.28. In Fig., tangents PQ and PR are drawn from an external point P
to a circle with centre O, such that –RPQ = 30°. A chord RS is O

drawn parallel to the tangent PQ. Find –RQS. 30°


P
Q Fig. 9

Q.29. From a point P on the ground the angle of elevation of the top of a tower is 30° and that of
the top of a flag staff fixed on the top of the tower, is 60°. If the length of the flag staff is
5m, find the height of the tower.
Q.30. Not in Current Syllabus.
✽ • •• • ✽

MATHEMATICS—2015 (DELHI)  3

Click here to access full book!


SOLUTIONS
1. Let BD = x cm –OAB = –OBA ...(i)
then BW = (24 – x)cm, AE = 12 – 4 = 8 cm ...[Angles opposite equal sides
In DDEW, AB || EW –OAP + –AOB + –OBP + –APB = 360º
D ...[Quadratic rule
AD BD
= 90º + –AOB + 90º + 50º = 360º

4 cm
AE BW
...[Thales’ Theorem A B –AOB = 360º – 230º = 130º ...(ii)

24
m
–AOB + –OAB + –OBA

cm
12 c
4 x A
= = 180º
8 24 − x
fi 8x = 96 – 4x ...[Triangle rule
E W P 50º O
96
fix= = 8 cm \ DB = 8 cm
12 130º + 2–OAB = 180º
2. Given: 3 sin q = cos q ...[From (i) & (ii)
B

1 sin q 1 2–OAB = 50º \ –OAB = 25°


= fi tan q =
3 cos q 3 7. 17 × 5 × 11 × 3 × 2 + 2 × 11 ...(i)
tan q = tan 30º fi q = 30º ...(i) = 2 × 11 × (17 × 5 × 3 + 1)
2 cos q(3 cos q + 2) = 2 × 11 × (255 + 1) = 2 × 11 × 256
Now, 3 cos q + 2 cos q = (3 cos q + 2) Number (i) is divisible by 2, 11 and 256
3 cos q + 2
and it has more than 2 prime factors.
3
= cos q = cos 30º = ...[From (i) \ (17 × 5 × 11 × 3 × 2 + 2 × 11) is a
2
3. The given quadratic equation can be composite number.
written as, 3x2 + 2kx – 3 = 0 AX 1
8. = ...[Given
( −21 ) ( −1 ) – 3 = 0
2 AB 4
−1
3 + 2k 2 ...[Putting x =
2 AX = 1K, AB = 4K
3 3 − 12 \ BX = AB – AX
–k–3=0 fi –k + =0
4 4 = 4K – 1K = 3K
9 −9 AX 1K 1
fi –k – =0 \ k= 4 = =
4 XB 3K 3
4. When base is same for both towers and AY 2 1 AX AY
their heights are given, i.e., x and y respectively. = = \ = ...[each is 1/3
YC 6 3 XB YC
Let the base of towers be k. \ XY || BC ...[By converse of Thales’ theorem
x y Or
tan 30º = tan 60º =
k k 1 58º
x = k tan 30º y = k tan 60º 8. –ABQ = –AOQ = = 29°
2 2
k –BAT = 90º ...[Tangent is ^ to the radius
x= ...(i) y= k 3 ...(ii) through the point of contact
3
From equations (i) and (ii), –ATQ = 180° – (–ABQ + –BAT)
= 180º – (29º + 90º) = 180° – 119°
k
= 61º
x k 1 1 =1:3
= 3 = × = 3 3
9. L.H.S. = sin q + cos q
y k 3 3 k 3 3
5. Total English alphabets = 26 sin q + cos q
Number of consonants = 21 (sin q + cos q )(sin 2 q + cos 2 q - sin q cos q)
21 = (sin q + cos q)
\ P (letter is a consonant) =
26
6. PA = PB ...[Tangents drawn from  a3 + b3 = (a+b)(a2 + b2 – ab)
...[
external point are equal = 1 – sin q cos q = R.H.S.  sin2 q + cos2 q = 1
...[
–OAP = –OBP = 90º

4  SHIV DAS CBSE SECONDARY SERIES (X)

Click here to access full book!


10. The given quadratic equation can be \ HCF of L, B and H is 52 = 25 cm
written as, \ Length of the longest rod = 25 cm
4x2 – 4a2x + (a4 – b4) = 0 14. Let unit and tens digit be x and y.
(4x2 – 4a2x + a4) – b4 = 0 \ Original number = 1x + 10y ...(i)
(2x – a2)2 – (b2)2 = 0 Reversed number = 10x + 1y
According to question,
\ (2x – a2 + b2) (2x – a2 – b2) = 0
x+y=8
fi (2x – a2 + b2) = 0 or (2x – a2 – b2) = 0
fiy=8–x ...(ii)
2 2 2 2
fi x = a −b or x = a +b 1x + 10y – (10x + y) = 18
2 2 fi x + 10y – 10x – y = 18
11. The given A.P. is 6, 13, 20, ..., 216 fi 9y – 9x = 18
Let n be the number of terms, fiy–x=2 ...[Dividing both sides by 9
d = 7, a = 6, an = 216 fi8–x–x=2 ...[From (ii)
an = a + (n – 1)d fi 8 – 2 = 2x
\ 216 = 6 + (n – 1).7 fi 2x = 6 fi x=3
216 – 6 = (n – 1)7 From (ii), y = 8 – 3 = 5
210 From (i), Original number = 3 + 10(5) = 53
=n–1 fi 30 + 1 = n
7 E
fi n = 31 15.

( n 2+ 1 )
th F
Middle term = term G

= ( 2 ) = ( 2 ) = 16
2 D
31 + 1 32 th term of the A.P. 3
1 6 5 4

\ a16 = a + 15d = 6 + 15 × 7 = 111 A


B
C

A(5, 2) Given: EB ^ AC, BG ^ AE and CF ^AE.


12.
To prove: (i) DABG ~ DDCB
BC BE
(ii) =
BD BA
Proof: (i) In DABG and DDCB
C(–2, t) B(2, –2) –2 = –5 ...[each 90º
ABC is a right angled triangle, –6 = –4 ...[corresponding angles
\ AC2 = BC2 + AB2 ...(i) ...[Pythagoras’ theorem \ DABG ~ DDCB (Hence Proved)
...[By AA similarity
Using distance formula,
–1 = –3 ...[CPCT
AB2 = (5 – 2)2 + (2 + 2)2 = 9 + 16 = 25
...[In ~ D
Ds, corresponding angles are equal
BC2 = (2 + 2)2 + (t + 2)2 = 16 + (t + 2)2 (ii) In DABE and DDBC
AC2 = (5 + 2)2 + (2 – t)2 = 49 + (2 – t)2 –1 = –3 ...[proved above
Putting values of AB2, AC2 and BC2 in –ABE = –5 ...[each is 90º, EB ^ AC (Given)
equation (i), we get DABE ~ DDBC ...[By AA similarity
49 + (2 – t)2 = 16 + (t + 2)2 + 25 BC BD
=
\ 49 + (2 – t)2 = 41 + (t + 2)2 BE BA
...[In ~ D
Ds, corresponding sides are proportional
fi (t + 2)2 – (2 – t)2 = 8
fi (t2 + 4 + 4t – 4 – t2 + 4t) BC BE
\ = (Hence Proved)
BD BA
fi 8t = 8 fi t=1
13. To find the length of the longest rod, 12 P 12
16. Given: sin q = \ =
13 H 13
We have to find HCF
L, Length = 8 m 50 cm = 850 cm = 21 × 52 × 17 Let, P = 12K, H = 13K
B, Breadth = 6 m 25 cm = 625 cm = 54 P2 + B2 = H2 ...[Pythagoras’ theorem
H, Height = 4 m 75 cm = 475 cm = 52 × 19 (12K)2 + B2 = (13K)2

MATHEMATICS—2015 (DELHI)  5

Click here to access full book!


144K2 + B2 = 169K2 Q is the mid point of AC
B2 = 169 K2 – 144 K2 = 25K2 1+z −4 + t
0= –1 =
B = 5K 2 2
B 5K 5 H –1 = z –2 = –4 + t
\ cos q = = = P
2= t
H 13K 13
P 12K 12 \ C(–1, 2)
tan q = = =
\ Other coordinates of DABC are B(3, 2) and
B 5K 5 B
sin 2 θ − cos 2 θ 1 C(–1, 2).
Now, 2 sin θ. cos θ × 19. The given quadratic equation can be
tan 2 θ
written as
2 2
 12  −  5 
 13   13 
kx2 + x2 – 2(k – 1)x + 1 = 0
    × 1 (k + 1) x2 – 2 (k – 1) x + 1 = 0 ...(i)
= 2  12   5   12 
2
a = (k + 1), b = –2(k – 1), c = 1
 13   13   5 
     For equal roots, D = 0
144 − 25
D = b2 – 4ac
169 × 25 119 25 595 0 = [–2(k – 1)]2 – 4 × (k + 1) × 1
= × =
120 144 = 120 144 3,456 0 = 4(k – 1)2 – 4(k + 1)
169 0 = 4k2 + 4 – 8k – 4k – 4
17. Let the number of boys = n1 0 = 4k2 – 12k fi 4k(k – 3) = 0
and number of girls = n2 k–3=0 or 4k = 0
Average boys’ score = 71 = X 1 fi k = 3 or k = 0 \ k=3
Average girls’ score = 73 = X 2 Putting k = 3 in equation (i), we get
4x2 – 4x + 1 = 0
Average score of school = 71.8 = X 1,2 4x2 – 2x – 2x + 1 = 0
n1 X 1 + n2 X 2 2x(2x – 1) – 1(2x – 1) = 0
Combined mean, X 1,2 = n +n 1 2 (2x – 1) (2x – 1) = 0
n1 (71) + n2 (73) 2x – 1 = 0 or 2x – 1 = 0
71.8 = n1 + n2 1 1
71n1 + 73n2 = 71.8n1 + 71.8n2 x= or x=
2 2
71n1 – 71.8n1 = 71.8n2 – 73n2 1 1
Roots are 2 , 2 .
–0.8n1= –1.2n2
20. Two dice are rolled.
n1 1.2 n1 3
= fi = fi n1 : n2 = 3 : 2 \ Total possible outcomes = 6n = 62 = 36
n2 0.8 n2 2
(i) The possible outcomes of getting a total
\ No. of boys : No. of girls = 3 : 2 of 5 are (2, 3), (3, 2), (1, 4), (4, 1), i.e., 4.
18. 4 1
A(1,–4) \ Required Probability, P(E) = =
36 9
(ii) The possible outcomes of getting even
P(2,–1) Q(0,–1) numbers on both dice are (2, 2), (2, 4), (2,
6), (4, 2), (4, 4), (4, 6), (6, 2), (6, 4), (6, 6)
B(x,y) C(z,t) i.e., 9
Let B(x, y) and C(z, t) be the coordinates 9 1
P is the mid-point of AB \ Required Probability, P(E) = =
36 4
x+1 y−4 21. OA = OB ...[Given
Q
=2 = –1
2 2 Let OA = OB = r
x+1=4 y – 4 = –2 A B
Since, AO is the diameter O
x=4–1=3 y = –2 + 4 = 2
of small semicircle
\ B(3, 2)
P

6  SHIV DAS CBSE SECONDARY SERIES (X)

Click here to access full book!


\ Radius of small –C = –C ...[Common
AO r –CEF = –CAB ...[corresponding angles
semicircle, R = =
2 2 \ DCEF ~ DCAB ...[AA similarity
BO is the radius of big semicircle = r
A.T.Q., Total perimeter = Perimeter of small CE EF CE EF
fi = fi =
semi-circle + Perimeter of big semicircle + OB AC AB AE + EC AD + DB
40 = pR + pr + r 4 EF 28
fi 9 = 7 fi EF = 9
22 r r
40 = 7 2 + 1 + r = ( 22 r 22 r r
14
+
7
+
1 ) and AB = AD + DB
22 r + 44r + 14r AB = 4 + 3 = 7
40 =
14 28
560 = 80r fi r=
560
= 7 cm \ EF = cm and AB = 7 cm
9
80
\ Area of shaded region = (Area of small 25. LHS = (1 + cot A + tan A)(sin A – cos A)
semicircle) + (Area of big semicircle)
2 2 π 2
( cos A sin A
)
= 1 + sin A + cos A (sin A – cos A)
= πR + πr = (R + r2)
2 2 2  sin A cos A + cos 2 A+ sin 2 A 
=   (sin A – cos A)
1 22  r
()  1 22  7
()   
2 2 sin A cos A
= 2 × 7 × + r2  = ×  + (7)2 
...[Using a3 – b3 = (a – b)(a2 + ab + b2)
 2  2 7  2 
11
( 49 49
= 7 × 4 + 1 = 7 × 4 = 4 = 96 cm2) 4
11 245 385 1 3 3
= sin A − cos A
sin A cos A
22. Given: Length, breadth, height of cuboidal
sin 3 A cos 3 A
block be 15 cm, 10 cm and 5 cm respectively. −
sin A cos A sin A cos 3 A
3 3 3
Total surface area of solid cuboidal block = sin A cos A
= 2(lb + bh + lh) sin 3 A cos 3 A
= 2(15 × 10 + 10 × 5 + 15 × 5) cm2 ...[dividing Num. & Denom. by sin3 A . cos3A
= 2(150 + 50 + 75) = 2(275) = 550 cm2
7 sec 3 A − cosec 3 A
Radius of cylindrical hole = r = cm = = RHS (Hence proved)
2 sec 2 A.cosec 2 A
Area of two circular bases 26.
22 7 7
= pr2 + pr2 = 2pr2 fi 2 × × × = 77 cm2 Marks No. of Xi di’ = fidi’
7 2 2
Curved Surface Area of cylinder = 2prh X i − 55
students (fi) 10
22 7
= 2 × 7 × 2 × 5 = 110 cm2
0–10 1 5 –5 –5 
\ Required area= (Area of cuboidal block – 10–20 3 15 –4 –12
Area of two circular bases + Area of cylinder)
–73
20–30 7 25 –3 –21
= (550 – 77 + 110) cm2 = 583 cm2 30–40 10 35 –2 –20
23. Not in Current Syllabus 40–50 15 45 –1 –15
24. To lines to be C
50-60 x A=55 0 0
parallel, the required 60-70 9 65 1 9

117+4y

4 2
condition is equal ratio, 70-80 27 75 2 54
E F
which is true only with 80-90 18 85 3 54
CE CF
5 2.5 90-100 y 95 4 4y
EF and AB as = Sfi = Sfidi¢
EA BF A B
3 D 4 90+x+y = 44 + 4y
4 2 20
fi 5 = 2.5 or 25 ...[Converse of Thales’ theorem
Sfi = 120 = 90 + x + y ...[Given
For lengths of parallel lines. \ 90 + x + y = 120
In DCEF and DCAB x = 120 – 90 – y = 30 – y ...(i)

MATHEMATICS—2015 (DELHI)  7

Click here to access full book!


Σf i di ′ 12
Mean = A + Σf ×h fi S12 = (2(100) + (12 – 1)20)
i
2
[A = 55, h = 10, Sfi = 120, Mean = 59 12
S12 = [2(100) + 11(20)] = 6[420] = `2520
2
 44 + 4 y 
59 = 55 +  × 10  \ She can send her daughter to school.
 120 
28. Given: Join OR,
4(11 + y ) R
59 – 55 = OQ and OP. S
12
4 × 3 = 11 + y fi y = 12 – 11 = 1 PR = PQ O
...[Tangents drawn
From (i), x = 30 – 1 = 29
from external point 30°
\ x = 29, y = 1 are equal
P
Q
27. Let the denominator and numerator of fi –PRQ = –PQR
the fraction be x and x – 3 respectively. ...[In DPQR, Angles opposite equal sides are equal
x−3 In DPQR,
Let the fraction be
x fi –PRQ + –RPQ + –PQR = 180º D Rule
...[D
By the given condition,
fi 30 + 2–PQR = 180º
x−3+2 x−1
New fraction, = (180 − 30)°
x+2 x+2 fi –PQR = = 75°
2
x − 3 x − 1 29
\ + = fi SR || QP and QR is a transversal.
x x + 2 20
 –SRQ = –PQR ...[Alternate interior angles
( x − 3)( x + 2) + x( x − 1) 29
fi = \ –SRQ = 75º
x( x + 2) 20
fi –ORP = 90° ...[Tangents are ^ to the radius
( x − 3)( x + 2) + x( x − 1) 29 –OQP = 90° through the point of contact
fi =
x2 + 2x 20 In Quadrilateral PQOR,
fi 20[(x – 3) (x + 2) + x(x – 1)] = 29(x2 + 2x) \ –PQO + –QOR + –ORP + –RPQ = 360º
fi 20(x2 – x – 6 + x2 – x) = 29x2 + 58x 90º + –QOR + 90º + –30º = 360º
fi 20(2x2 – 2x – 6) = 29x2 + 58x –QOR = 360º – 210º = 150º
fi 40x2 – 29x2 – 40x – 58x = 120 1 150º
fi –QSR = –QOR fi –QSR = = 75º
fi 11x2 – 98x – 120 = 0 2 2
In DRSQ,
fi 11x2 – 110x + 12x – 120 = 0 –RSQ + –QRS + –RQS = 180º ...[D D Rule
fi 11x(x – 10) + 12(x – 10) = 0 75º + 75º + –RQS = 180º
fi (11x + 12) (x – 10) = 0 \ –RQS = 180º – 150º = 30º
fi 11x + 12 = 0 or x – 10 = 0 29. Let DC = x be the tower and AD = 5 m be
−12 the flagstaff, P is the given point so that PC = y.
fi x= (Reject) or x = 10
11
In DPDC,
Now, denominator (x) = 10 Flagstaff
x 1 A
then, numerator = x – 3 = 7 y = tan 30° = 3
5m

7
\ The fraction is . y = 3x ...(i)
10 D Tower
Or In DAPC,
27. Money required for Ramkali for x+5 x
admission of her daughter = `2500 y = tan 60° = 3 60º
30º
P
A.P. formed by saving x+5 y C
3x
= 3 ...[From (i)
100, 120, 140, ... upto 12 terms ...(i)
Let, a, d and n be the first term, common fi 3x = x + 5 or x = 2.5
difference and number of terms respectively. \ Height of Tower = x = 2.5 m
\ a = 100, d = 20, n = 12 30. Not in Current Syllabus.
n ✽ • •• • ✽
Sn = (2a + (n – 1)d)
2

8  SHIV DAS CBSE SECONDARY SERIES (X)

Click here to access full book!


Series : SDS/02 Code No. 041
Candidates must write the Code on
Roll No. the title page of the answer-book.

• Please check that this question paper contains 3 printed pages.


• Code number given on the right hand side of the question paper should be
written on the title page of the answer-book by the candidate.
• Please check that this question paper contains 30 questions.
• Please write down the Serial Number of the question before attempting it.
• 15 minutes time has been allotted to read this question paper. The question
paper will be distributed at 10.15 a.m. From 10.15 a.m. to 10.30 a.m., the students
will read the question paper only and will not write any answer on the answer-
book during this period.

MATHEMATICS
CBSE–2015 (Outside Delhi)
Time allowed: 3 hours Maximum Marks: 80
GENERAL INSTRUCTIONS:
(i) All questions are compulsory.
(ii) This question paper contains of 30 questions.
(iii) Question No. 1-6 in Section A are very short answer type questions carrying 1 mark each.
(iv) Question No. 7-12 in Section B are short answer type questions carrying 2 marks each.
(v) Question No. 13-22 in Section C are long answer-I type questions carrying 3 marks each.
(vi) Question No. 23-30 in Section D are long answer-II type questions carrying 4 marks
each.
SECTION A
Questions number 1 to 6 carry 1 mark each. A
x x+3
Q.1. In the Fig., DABC, DE || BC, find the value of x. D E

x+1 x+5

Q.2. Not in Current Syllabus. B C


Q.3. In a continuous frequency distribution, the median of the data is 21. If each observation is
increased by 5, then find the new median.
Q.4. If the quadratic equation px2 – 2 5 px + 15 = 0 has two equal roots, then find the value
of p.
Q.5. Two different dices are tossed together. Find the probability that the product of the two
numbers on the top of the dice is 6.

MATHEMATICS—2015 (OUTSIDE DELHI)  9

Click here to access full book!


Q
Q.6. In Fig., PQ is a chord of a circle with centre O and PT is a tangent. If
–QPT = 60°, find –PRQ. O

R
SECTION B P T
Questions number 7 to 12 carry 2 marks each.
Q.7. Check whether 4n can end with the digit 0 for any natural number n.
Or
Q.7. Find the largest number which divides 70 and 125 leaving remainder 5 and 8 respectively.
Q.8. A 6.5 m long ladder is placed against a wall such that its foot is at a distance of 2.5 m from
the wall. Find the height of the wall where the top of the ladder touches it.
Q.9. If x = a cos q – b sin q and y = a sin q + b cos q, then prove that
a2 + b2 = x2 + y2.
Q.10. In Fig., two tangents RQ and RP are drawn from an external P
point R to the circle with centre O. If –PRQ = 120°, then prove O R
that OR = PR + RQ.
Q
Or
A
Q.10. In Fig., a triangle ABC is drawn to circumscribe a circle of
radius 3 cm, such that the segments BD and DC are O
respectively of lengths 6 cm and 9 cm. If the area of DABC is 3 cm
2 B C
54 cm , then find the lengths of sides AB and AC. D
Q.11. Solve the following quadratic equation for x: 6 cm 9 cm
4x2 + 4bx – (a2 – b2) = 0
Q.12. In an AP, if S5 + S7 = 167 and S10 = 235, then find the AP, where Sn denotes the sum of its
first n terms.

SECTION C
Questions number 13 to 22 carry 3 marks each.
Q.13. Not in Current Syllabus
Q.14. 7x – 5y – 4 = 0 is given. Write another linear equation, so that the lines represented by the
pair are:
(i) intersecting (ii) coincident (iii) parallel
Q.15. State whether the given pairs of triangles are similar or not. In case of similarity mention
the criterion.
(a) A (b)
D P L
16
cm 45º 57º
cm

cm
cm

9
15

17

B C Q 78º N
9 cm O 5 cm 45º
R M

sin q − 2 sin 3q
Q.16. Prove that: = tan q.
2 cos3 q - cos q

10  SHIV DAS CBSE SECONDARY SERIES (X)

Click here to access full book!


Q.17. Find the mean and median for the following data:
Class 0-4 4-8 8 - 12 12 - 16 16 - 20
Frequency 3 5 9 5 3
Q.18. The 14th term of an AP is twice its 8th term. If its 6th term is –8, then find the sum of its first
20 terms.
Q.19. The points A(4, 7), B(p, 3) and C(7, 3) are the vertices of a right triangle, right-angled at B.
Find the value of p.
Or
Q.19. If the coordinates of points A and B are (–2, –2) and (2, –4) respectively, find the
3
coordinates of P such that AP = AB, where P lies on the line segment AB.
7
Q.20. The probability of selecting a red ball at random from a jar that contains only red, blue and
1
orange balls is . The probability of selecting a blue ball at random from the same jar is
1 4
. If the jar contains 10 orange balls, find the total number of balls in the jar.
3
Q.21. Find the area of the minor segment of a circle of radius 14 cm, when its central angle is 60º.
Also find the area of the corresponding major segment. [Use p = 22/7]
Q.22. Due to sudden floods, some welfare associations jointly requested the government to get
100 tents fixed immediately and offered to contribute 50% of the cost. If the lower part of
each tent is of the form of a cylinder of diameter 4.2 m and height 4 m with the conical
upper part of same diameter but of height 2.8 m, and the canvas to be used costs `100 per
sq. m, find the amount, the associations will have to pay. [Use p = 22/7]

SECTION D
A
Questions number 23 to 30 carry 4 marks each.
D
Q.23. Not in Current Syllabus.
Q.24. In the fig., –BED = –BDE and E divides BC in the ratio 2 : 1. B C
Prove that AF × BE = 2 AD × CF. E
1
Q.25. If sec q – tan q = x, show that sec q + tan q = and hence find the
x
values of cos q and sin q.
Or F
Q.25. Prove the following trigonometric identities:
sin A (1 + tan A) + cos A (1 + cot A) = sec A + cosec A.
Q.26. Mode of the following frequency distribution is 65 and sum of all the frequencies is 70.
Find the missing frequencies x and y.
Class 0–20 20–40 40–60 60–80 80–100 100–120 120– 140 140-160
Frequency 8 11 x 12 y 9 9 5
Q.27. Find the 60thterm of the AP 8, 10, 12, ..., if it has a total of 60 terms and hence find the sum
of its last 10 terms.
Q.28. Prove that the tangent drawn at the mid-point of an arc of a circle is parallel to the chord
joining the end points of the arc.
Q.29. The angle of elevation of an aeroplane from a point A on the ground is 60°. After a flight
of 15 seconds, the angle of elevation changes to 30°. If the aeroplane is flying at a constant
height of 1500 3 m, find the speed of the plane in km/hr.
Q.30. Not in Current Syllabus.
✽ • •• • ✽
MATHEMATICS—2015 (OUTSIDE DELHI)  11

Click here to access full book!


SOLUTIONS

1. In DABC, DE || BC ...[Given fi 240º = 2–PRQ


AD AE 240º
DB
=
EC
...[Thales’ theorem –PRQ = = 120º
2
x x+3 7. 4n = (22)n = 22n
=
x+1 x+5
The only prime in the factorization of 4n is 2.
x(x + 5) = (x + 3)(x + 1)
There is no other primes in the factorization
x2 + 5x = x2 + 3x + x + 3
of 4n = 22n (By uniqueness of the fundamental
x2 + 5x – x2 – 3x – x = 3
Theorem of Arithmetic).
\ x = 3 cm
5 does not occur in the prime factorization of
2. Not in Current Syllabus.
4n for any n.
3. New median = 21 + 5 = 26
\ 4n does not end with the digit zero for any
4. The given quadratic equation can be written as
natural number n.
px2 – 2 5 px + 15 = 0 Or
a = p, b = –2 5 p, c = 15 7. It is given that on dividing 70 by the
For equal roots, D = 0 required number, there is a remainder 5. This
D = b2 – 4ac means that 70 – 5 = 65 is exactly divisible by
the required number.
0 = ( −2 5p ) – 4 × p × 15
2
Similarly, 125 – 8 = 117 is also exactly
0 = 4 × 5p2 – 60p divisible by required number.
0 = 20p2 – 60p fi 20p2 = 60p 65 = 5 × 13 5 65 3 117
60 p 117 = 32 × 13 13 13 3 39
p = 20 p = 3 \ p=3 \ HCF = 13 1 13 13
1
5. Total outcomes = 6n = 62 = 36 \ Required number = 13
Possible outcomes having the product of the 8. Let AC be the ladder and A

two numbers on the top of the dice as 6 are AB be the wall.


(3 × 2, 2 × 3, 6 × 1, 1 × 6), i.e, 4. 13 m
AC = 6.5 m = m 6.
5
4 1 2
\ P(Product of two numbers is 6) = =
36 9 5
6. PQ is the chord of the circle & PT is tangent. BC = 2.5 m = m C B
2 2.5 m
 Tangent to a circle is
\ –OPT = 90º ...
In rt. DABC, AB2 + BC2 = AC2
^ to the radius through
...[Pythagoras theorem
the point of contact

( 25 ) = ( 132 )
2 2
Now, –QPT = 60º...[Given
AB2 +
–OPQ = –OPT – –QPT Q
–OPQ = 90º – 60º = 30º 169 25
In DOPQ,
O AB2 = −
4 4
OP = OQ ...[radii R
169 − 25 144
–OQP = –OPQ = 30º P = = = 36
T 4 4
...[In a D, equal sides have equal –s opp. them
\ Required height, AB = 6 m
In DOPQ, –OQP + –OPQ + –POQ = 180º
9. RHS = x2 + y2
...[Angle Sum property of D
= (a cos q – b sin q)2 + (a sin q + b cos q)2
\ –POQ = 120º ...[–
–POQ = 180º – (30º + 30º)
= a2 cos2 q + b2 sin2 q – 2ab cos q sin q +
fi Reflex –POQ = 360º – 120º = 240º
a2 sin2 q + b2 cos2 q + 2ab sin q cos q
\ Reflex –POQ = 2–PRQ
= a2 (cos2 q + sin2 q) + b2 (sin2 q + cos2 q)
We know that the angle substended by an arc at
... the centre of a circle is twice the angle subtended = a2 + b2 = LHS  cos2 q + sin2 q = 1
...[
by it at any point on the remaining part of the circle

12  SHIV DAS CBSE SECONDARY SERIES (X)

Click here to access full book!


10. Const.: Join OP and OQ. fi 6x + 90 = 108
–OPR = –OQR = 90º... Tangent ^ to the radius fi 6x = 18 fi x =3
through the point of contact
fi AB = x + 6 = 3 + 6 = 9 cm
PR = RQ ... Tangents drawn from an fi AC = x + 9 = 3 + 9 = 12 cm
external point are equal
\ AB = 9 cm, AC = 12 cm and BC = 15 cm
Given: –PRQ = 120º 11. Given. 4x2 + 4bx + b2 – a2 = 0
1 1 fi (2x + b)2 – (a)2 = 0 ...[x2 – y2 = (x + y)(x – y)
–PRO = –PRQ = × 120º = 60º
2 2 fi (2x + b + a) (2x + b – a) = 0
Now, In DOPR,
fi (2x + b + a) = 0 or (2x + b – a) = 0
fi –OPR + –POR + –PRO = 180º D Rule
...[D
fi 2x = –(a + b) or 2x = (a – b)
fi 90º + –POR + 60º = 180º
( a + b) ( a − b)
fi –POR + 150º = 180º P fi x= − or x =
2 2
fi –POR = 30º
R (a + b ) (a − b )
PR 1
O Hence solution for x = − ,
fi = sin 30° = 2 2
OR 2 Q 12. Given: S5 + S7 = 167
fi OR = 2PR 5 7
fi OR = PR + QR ...[
 PR = RQ fi [2a + (5 – 1)d] + [2a + (7 – 1)d] = 167
2 2
...Hence proved n
...[Sn = (2a + (n – 1)d
Or 2
10. Given: OD = 3 cm fi
5 7
[2a + 4d] + [2a + 6d] = 167
Construction: Join OA, OF, OE, OC, OD & 2 2
OB. fi 5(a + 2d) + 7(a + 3d) = 167
Let AF = AE = x cm Tangents drawn fi 5a + 10d + 7a + 21d = 167
BD = BF = 6 cm ... from an external fi 12a + 31d = 167 ...(i)
point are equal 10
CD = CE = 9 cm fi S10 = (2a + (10 – 1)d) = 235
Let OF = OE = OD = 3 cm ...[Radii
2
fi 5[2a + 9d] = 235
Ax
cm
fi 10a + 45d = 235 ...(ii)
m
xc

E
F
9c
Solving (i) and (ii), a = 1, d = 5
O m \ a1 = 1
m
6c

B
3 cm
C
a2 = a + d fi a2 = 1 + 5 = 6
6 cm
D
9 cm
a3 = a + 2d fi a3 = 1 + 10 = 11
Hence A.P. is 1, 6, 11...
\ AB = AF + BF = x + 6 ...(i)
13. Not in Current Syllabus.
AC = AE + CE = x + 9 ...(ii)
14. 7x – 5y – 4 = 0
BC = DB + CD = 6 + 9 = 15 cm ...(iii)
In DABC, Area of DABC = 54 cm2 ...[Given (i) 7x + 3y + 2 = 0 ...  Here,
a1 b 1
π
Ar(DAOB + DAOC + DBOC) = 54 a2 b2
1 a b c
...[Area of D = × Base × height (ii) 14x – 10y – 8 = 0 ...  Here, ..1 = 1 = 1
2 a2 b2 c2
1 1
fi × AB × OF + × AC × OE
2 2 a1 b1 c 1
1 (iii) 7x – 5y + 3 = 0 ...  Here, = π
+ × BC × OD = 54 a2 b2 c2
2
AO 16 BO 9
1 1 1 15. (a) = and = .....
fi × (6 + x) × 3 + × (x + 9)× 3 + × 15 × 3 DO 9 CO 5
2 2 2
AO BO
1 ≠ \ Given Ds are not similar.
fi [x + 6 + x + 9 + 15] × 3 = 54 DO CO
2 (b) In DPQR, –P + –Q + –R = 180º
...[From (i), (ii) & (iii)
...[Angle-Sum Property of a D
1
fi [2x + 30] × 3 = 54 45º + 78º + –R = 180º
2
–R = 180º – 45º – 78º = 57º
MATHEMATICS—2015 (OUTSIDE DELHI)  13

Click here to access full book!


In DLMN, –L + –M + –N = 180º –8 = a + 5d fi –d + 5d = –8 ...[From (i)
...[Angle-Sum Property of a D 4d = –8 fi d = –2
57º + 45º + –N = 180º Value of d put in equation (i), we get
–N = 180º – 57º – 45º = 78º a = –d fi a = –(–2)
Here, –P = –M ...[each = 45º Now, a = 2, d = –2
–Q = –N ...[each = 78º Now, Sum of first 20 terms,
–R = –L ...[each = 57º 20
\ DPQR ~ DMNL ...[By AAA similarity theorem S20 = [2 × 2 + (20 – 1)(–2)]
2
sin θ − 2 sin 3 θ n
16. LHS = ...[Sn = (2a + (n – 1)d
2 cos 3 θ − cos θ 2
S20 = 10[4 + 19(–2)]
sin θ (1 − 2 sin 2 θ) S20 = 10[4 – 38] = –340
=
cos θ(2 cos 2 θ − 1) 19. In right DABC, AB2 + BC2 = AC2
...[Pythagoras’ theorem
sin θ (1 − 2 sin 2 θ)
= 2  cos2q = 1 – sin2 q
...[ fi (p – 4)2 + (3 – 7)2 + (7 – p)2 + (3 – 3)2
cos θ[2(1 − sin θ) − 1]
= (7 – 4)2 + (3 – 7)2
tan θ (1 − 2 sin 2 θ) tan θ (1 − 2 sin 2 θ) ...[by using distance formula
= = fi (p – 4)2 + (–4)2 + (7 – p)2 = (3)2 + (–4)2
(2 − 2 sin 2 θ − 1) (1 − 2 sin 2 θ)
fi 16 + p2 – 8p + 16 + 49 + p2 –14p = 9 + 16
= tan q = RHS (Hence proved) fi 81 + 2p2 – 22p = 25 A(4, 7)
17. fi 2p2 – 22p + 56 = 0
Class Frequency Xi fiXi c.f. or p2 – 11p + 28 = 0
(fi) fi p2 – 7p – 4p + 28 = 0
0–4 3 2 6 3 fi p(p – 7) – 4(p – 7) = 0
4–8 5 6 30 cf = 8 fi (p – 7) (p – 4) = 0
8–12 f=9 10 90 17
C(7, 3) B (p, 3)
12–16 5 14 70 22
16–20 3 18 54 25 fi p–7=0 or p–4=0
Sfi = 25 SfiXi fi p=7 or p=4
= 250 Since p π 7
Σ f i Xi 250 \ p=4
Mean (X) = = = 10 Or
Σf i 25
n 25 3
Here, = = = 12.5 19. AP = AB fi AP : PB = 3 : 4
2 2 7
\ Median class is 8 – 12. (–2, –2) 3:4 (2, –4)
n A P (x, y) B
− c. f .
Median = l + 2 ×h Applying the section formula,
f
...[Here l = 8, h = 4, cf = 8, f = 9 mx2 + nx1 my2 + ny1
x= y=
12.5 − 8 4.5 m+n m+n
=8+ ×4 =8+ ×4
9 9 3(2) + 4( −2) 3( −4) + 4( −2)
x= y=
= 8 + 2 = 10 3+4 3+4
18. Let a = First term, d = Common difference 6 − 8 −2 −12 − 8 20
fi x= = fi y= =−
a14 = 2.a8 ...[Given 7 7 7 7
fi a + 13d = 2 (a + 7d)
( )
...[
 an = a + (n – 1)d
2 20
fi a + 13d = 2a + 14d \ P = −7 ,− 7
fi 1a – 2a = 14d – 13d
fi –1a = d fi a = –d ...(i)
a6 = –8 ...[Given

14  SHIV DAS CBSE SECONDARY SERIES (X)

Click here to access full book!


1 1 Area of canvas (1 tent)
20. P(Red) = , P (blue) = = Area of cone + Area of cylinder
4 3
= prl + 2prH fi pr(l + 2H)
As we know, Total Probability = 1
22 46.2
fi P(orange) = 1 –
1 1
− =
5 = × 2.1(3.5 + 2 × 4) = × (11.5)
7 7
4 3 12
= 6.6 × 11.5 = 75.9 m2
Total no. of orange balls
fi P(orange) = Area of 100 tents = 75.9 m2 × 100 = 7590 m2
Total no. of balls
5 10
Cost of 100 tents = 7590 × `100 = `759000
fi = \ Associations have to pay 50% of the cost
12 Total no. of balls
10 × 12 50
\ Total no. of balls = = 24 = `7,59,000 × = `3,79,500
5 100
23. Not in Current Syllabus
21. 24. Given: –BED = –BDE and E divides BC
in 2 : 1.
O BE 2
cm
14
cm = ...(i)
14 60º EC 1
A B Construction:
Draw CG || DF
A
r = 14 cm, q = 60°
G
1 2
\ Area of minor segment = pr2 – r sin q D
2
22 60 1
= × 14 × 14 × − × 14 × 14 × 3
7 360 2 2 B C

( )
2 E 1
308
= – 49 3 cm2 = 17.9 cm2 (Approx.)
3
\ Area of major segment
= Area of circle – Area of minor segment
= pr2 – ( 3083 – 49 3 ) Prove that:
F

× 14 × 14 – ( 3 – 49 3 )
22 308
= AF × BE = 2AD × CF
7
308
To Prove: –BED = –BDE ...[Given
= 616 – + 49 3 BD = BE ...(ii)
3
...[Sides opposite to equal angles
= ( 1540
3
+ 49 3 ) cm 2 In DCBG, DE
BD BE
|| CG ...[By construction

or 598.10 cm2 (Approx.) fi = ...[Thales’ theorem


DG EC
22. Let slant height of cone be l
BD
Let height of cone be h = 2.8 m fi = 2 ...[From (i)
DG 1
Let radius of cone be r =
4.2
m = 2.1 m 2DG = BD fi 2DG = BE ...[From (ii)
2 1
Let height of cylinder be H = 4 m DG = BE ...(iii)
2
2.8 m

Using pythagoras’ theorem, In DADF, CG || DF ...[By construction


Slant height (l) AG AC
fi = ...[Thales’ theorem
2 2 2 2 GD CF
= h + r = (2.8) + (2.1) AG AC
4m

fi +1= + 1 ...[Adding one on both sides


GD CF
= 7.84 + 4.41 = 12.25
= 3.5 m 4.2 cm

MATHEMATICS—2015 (OUTSIDE DELHI)  15

Click here to access full book!


AG + GD AC + CF sin A + cos A
fi = = sin2 A + cos2 A = 1
[

GD CF cos A sin A
AD AF sin A cos A 1 1
fi = fi AF × GD = AD × CF = + = +
GD CF sin A cos A sin A cos A cos A sin A
fi AF × BE/2 = AD × CF ...[From (iii) = sec A + cosec A = RHS (Hence proved)
fi AF × BE = 2AD × CF ...(Hence proved)
26. Class interval f
25. Part I: As we know, sec2 q – tan2 q = 1
(sec q + tan q) (sec q – tan θ) = 1 0 – 20 8
(sec q + tan q) (x) = 1 ...[Given
20 – 40 11
40 – 60 x f0
(sec q + tan q) = 1/x ...(Hence proved)
60 – 80 12 f1
Part II:
sec q – tan q = x ...(i) 80 – 100 y f2
100 – 120 9
1
sec q + tan q = ...(ii) 120 – 140 9
x
140 – 160 5
1
2sec q =x+ ...[By adding 54 + x + y = 70
x
1 x + 1
2
1 ( x 2 + 1) Now, 54 + x + y = 70
fi sec q = 2  x  = fi
 cos θ 2x fi x + y = 70 – 54 = 16 ...(i)
2x Mode = 65 ...[Given
Taking reciprocal, \ cos q = 2
x +1 \ Modal Class is 60 – 80.
2
 2x  f 1 − f0
sin2 q = 1 – cos2 q fi sin2 q = 1 –  2  Mode = l + 2 f − f − f × h
 x + 1 1 0 2

4x 2 ...[l = 60, h = 20, f0 = x, f1 = 12, f2 = y


fi sin2 q = 1 –
( x 2 + 1)2 12 − x
65 = 60 + 2(12) − x − y × 20
( x 2 + 1)2 − 4 x 2
fi sin2 q = 12 − x
( x 2 + 1)2
65 – 60 = 24 − ( x + y ) × 20
x 4 + 2 x 2 + 1 − 4x 2
fi sin2 q = 12 − x
( x 2 + 1)2 5= × 20 ...[From (i)
24 − 16
4 2
x − 2x + 1  ( x − 1)
2 2
12 − x 2
fi sin2 q = ...  5= × 20 fi 5× = 12 – x
( x 2 + 1)2
4 2
 = x − 2 x + 1 8 5
( x 2 − 1)2
fi x = 12 – 2 = 10
x2 − 1
fi sin2 q = \ sin q = x + y = 16 ...[From (i)
( x 2 + 1)2 x2 + 1
10 + y = 16 fi y = 16 – 10 = 6
Or \ x = 10, y = 6
25. LHS = sin A (1 + tan A) + cos A (1 + cot A) 27. a = 8, d = a2 - a1 = 10 – 8 = 2, n = 60
 sin A   cos A  a60 = a + 59d = 8 + 59(2) = 126
= sin A  1 + cosA  + cos A  1 + 
   sinA 
\ Sum of its last 10 terms = S60 – S50
= sin A (cos A )
cos A + sin A + cos A sin A + cos A
sin A ( ) =
n
2
n
(a + an) – (2a + (n – 1)d)
2
sin A cos A 60 50
= ( cos A + sin A ) + (sin A + cos A) = (8 + a60) – (2 × 8 + (50 – 1)2)
cos A sin A 2 2
 sin A cos A  = 30 (8 + 126) – 25 (16 + 98)
= (sin A + cos A)  cos A + sin A 
  = 4020 – 25 × 114
 sin 2 A + cos 2 A  = 4020 – 2850 = 1170
= (sin A + cos A)  
 cos A sinA 
16  SHIV DAS CBSE SECONDARY SERIES (X)

Click here to access full book!


28. B is the mid point BL = CM = 1500 3
of arc (ABC). BL
In DOAF and DOCF,
O In DABL, = tan 60°
1 2 AL
A F C
OA = OC ...[Radius fi 1500 3 = 3
B x
D E
OF = OF ...[Common
fi x = 1500 m ...(i)
\ –1 = –2 In DAMC,
...[Equal angles opposite equal sides
CM
\ DOAF ≅ DOCF (SAS) = tan 30°
AL + LM
\ –AFO = –CFO = 90° ...[CPCT
1500 3 1
fi –AFO = –DBO = 90° ...[CPCT fi 1500 + LM = ...[From (i)
3
...[Tangent is ^ to the radius through the point of contact
fi (1500 + LM) = 1500(3)
But these are corresponding angles.
fi LM = 4500 – 1500 = 3000 m
\ AC || DE
29. Let AL = x, Distance 3000
\ Speed= = = 200 m/s.
B 15 seconds
Time 15
C
200
or, × 60 × 60
1000
1500 3
º = 720 km/hr.
60
30º
A
M
30. Not in Current Syllabus.
x L

✽ • •• • ✽

MATHEMATICS—2015 (OUTSIDE DELHI)  17

Click here to access full book!


Series : WX1YZ/4  Code No. 30/4/1
Candidates must write the Q.P. Code
Roll No.  on the title page of the answer-book.

• Please check that this question paper contains 8 printed pages.


• Code number given on the right hand side of the question paper should be written on the
title page of the answer-book by the candidate.
• Please check that this question paper contains 38 questions.
• Please write down the Serial Number of the question in the answer book before attempting it.
• 15 minutes time has been allotted to read this question paper. The question paper will be
distributed at 10.15 a.m. From 10.15 a.m. to 10.30 a.m., the candidates will read the question
paper only and will not write any answer on the answer-book during this period.

MATHEMATICS (Standard)–Theory
CBSE – 2023 (SERIES: WX1YZ/4)
Time Allowed: 3 hours Maximum Marks: 80
GENERAL INSTRUCTIONS:
Read the following instructions carefully and follow them:
(i) This question paper contains 38 questions. All questions are compulsory.
(ii) Question paper is divided into FIVE Sections — Section A, B, C, D and E.
(iii) In Section A—Question no. 1 to 18 are Multiple Choice Questions (MCQ) and Question number 19 and
20 are Assertion-Reason based questions of 1 mark each.
(iv) In Section B—Question number 21 to 25 are Very Short Answer (VSA) type questions of 2 marks each.
(v) In Section C—Question number 26 to 31 are Short Answer (SA) type questions, carrying 3 marks each.
(vi) In Section D—Question number 32 to 35 are Long Answer (LA) type questions carrying 5 marks each.
(vii) In Section E—Question number 36 to 38 are case study based integrated units of assessment questions
carrying 4 marks each. Internal choice is provided in 2 mark questions in each Case-study.
(viii) There is no overall choice. However, an internal choice has been provided in 2 questions in Section B,
2 questions in Section C, 2 questions in Section D and 3 questions in Section E.
(ix) Draw neat figures wherever required. Take p = 22/7 wherever required if not stated.
(x) Use of calculators is NOT allowed.

SET I Code No. 30/4/1

Section-A (Multiple choice questions)


Section A consists of Multiple Choice Type Questions of 1 Mark each.
Q.1. The ratio of HCF to LCM of the least composite number and the least prime number is:
1
(a) 1 : 2 (b) 2 : 1 (c) 1 : 1 (d) 1 : 3
Q.2. The roots of the equation x2 + 3x – 10 = 0 are: 1
(a) 2, –5 (b) –2, 5 (c) 2, 5 (d) –2, –5
136

Click here to access full book!


Q.3. The next term of the A.P.: 6 , 24 , 54 is: 1
(a) 60 (b) 96 (c) 72 (d) 216
Q.4. The distance of the point (–1, 7) from x-axis is: 1
(a) –1 (b) 7 (c) 6 (d) 50
Q.5. What is the area of a semi-circle of diameter ‘d’?1
1 1 1 1
(a) 16 pd2 (b) 4 pd2 (c) 8 pd2 (d) 2 pd2
Q.6. The empirical relation between the mode, median and mean of a distribution is: 1
(a) Mode = 3 Median – 2 Mean (b) Mode = 3 Mean – 2 Median
(c) Mode = 2 Median – 3 Mean (d) Mode = 2 Mean – 3 Median
Q.7. The pair of linear equations 2x = 5y + 6 and 15y = 6x – 18 represents two lines which
are:  1
(a) intersecting (b) parallel
(c) coincident (d) either intersecting or parallel
Q.8. If a, b are zeroes of the polynomial x2 – 1, then value of (a + b) is: 1
(a) 2 (b) 1 (c) –1 (d) 0
Q.9. If a pole 6 m high casts a shadow 2 3 m long on the ground, then sun’s elevation is:  1
(a) 60º (b) 45º (c) 30º (d) 90º
Q.10. sec q when expressed in terms of cot q, is equal to :  1
2
2 2 1 − cot θ
(a) 1 + cot θ (b) 1 + cot 2 θ (c) 1 + cot θ (d) cot θ
cot θ cot θ
Q.11. Two dice are thrown together. The probability of getting the difference of numbers on
their upper faces equals to 3 is: 1
(a) 1 (b) 2 (c) 1 (d) 1
9 9 6 12
Q.12. In the given figure, DABC ~ DQPR. A
6 cm
C R
If AC = 6 cm, BC = 5 cm,
QR = 3 cm and PR = x; then the
value of x is:1
x
5 cm

3 cm

(a) 3.6 cm
(b) 2.5 cm
(c) 10 cm
(d) 3.2 cm Q P
B
Q.13. The distance of the point (–6, 8) from origin is: 1
(a) 6 (b) –6 (c) 8 (d) 10
Q.14. In the given figure, PQ is a tangent to the circle with centre O. Q
If ∠OPQ = x, ∠POQ = y, then x + y is: 1
(a) 45º x y
P
(b) 90º O

(c) 60º
(d) 180º

Click here to access full book!


Q.15. In the given figure, TA is a tangent to the circle with centre O
such that OT = 4 cm, ∠OTA = 30º, then length of TA is: 1
(a) 2 3 cm O

(b) 2 cm 4c
m
30º
(c) 2 2 cm T A
(d) 3 cm
Q.16. In DABC, PQ || BC. If PB = 6 cm, AP = 4 cm, AQ = 8 cm, find the A
length of AC.
(a) 12 cm P Q
(b) 20 cm
(c) 6 cm B C
(d) 14 cm
Q.17. If a, b are the zeroes of the polynnomial p(x) = 4x2 – 3x – 7, then  1 + 1  is equal to: 1
α β 
7 −7 3 −3
(a) 3 (b) 3 (c) 7 (d) 7

s
Q.18. A card is drawn at random from a well-shuffled pack of 52 cards. The probability that

a
the card drawn is not an ace is: 1

iv D
4 12
(a) 1 (b) 9
(c) 13 (d) 13

Sh
13 13
Assertion–Reason Based Questions
In the following questions 19 and 20, a statement of Assertion (A) is followed by a statement
of Reason (R). Choose the correct answer out of the following choices:
(a) Both (A) and (R) are true and (R) is the correct explanation of (A).
(b) Both (A) and (R) are true and (R) is not the correct explanation of (A).
(c) (A) is true and (R) is false.
(d) (A) is false, but (R) is true.
Q.19. Assertion (A): The probability that a leap year has 53 Sundays is 2 . 1
7
Reason (R): The probability that a non-leap year has 53 Sundays is 5 .
7
Q.20. Assertion (A): a, b, c are in A.P. if and only if 2b = a + c. 1
Reason (R): The sum of first n odd natural numbers is n2.

Section-B
Section B consists of Very Short Answer (VSA) type of questions of 2 marks each.
Q.21. Two numbers are in the ratio 2 : 3 and their LCM is 180. What is the HCF of these numbers?
 2
Q.22. If one zero of the polynomial p(x) = 6x2 + 37x – (k – 2) is reciprocal of the other, then find
the value of k. 2
Q.23. (A) Find the sum and product of the roots of the quadratic equation 2x2 – 9x + 4 = 0. 2
Or
(B) Find the discriminant of the quadratic equation 4x2 – 5 = 0 and hence comment on
the nature of roots of the equation. 2

138 n SHIV
S-(138) n Shiv
DASDas
CBSE
CBSE
SECONDARY
Secondary Series
SERIES[X]
[X]

Click here to access full book!


Q.24. If a fair coin is tossed twice, find the probability of getting ‘atmost one head’. 2
2 2 2
5 cos 60 º + 4 sec 30 º − tan 45º
Q.25. (A) Evaluate :  2
sin 2 30 º + cos 2 30 º
Or
(B) If A and B are acute angles such that sin(A – B) = 0 and 2 cos (A + B) – 1 = 0, then
find angles A and B. 2
Section-C
Section C consists of Short Answer (SA) type questions of 3 marks each.
Q.26. (A) How many terms are there in an A.P. whose first and fifth terms are –14 and 2,
reespectively and the last term is 62. 3
Or
(B) Which term of the A.P.: 65, 61, 57, 53, ..................... is the first negative term? 3
Q.27. Prove that 5 is an irrational number. 3
Q.28. Prove that the angle between the two tangents drawn from an external point to a circle is
supplementary to the angle subtended by the line-segment joining the points of contact at
the centre. 3

s
sin A − 2 sin 3 A

a
Q.29. (A) Prove that: = tan A 3
2 cos 3 A − cos A

hiv D
Or
(B) Prove that: sec A (1 – sin A) (sec A + tan A) = 1. 3

S
Q.30. Two concentric circles are of radii 5 cm and 3 cm. Find the length of the chord of the larger
circle which touches the smaller circle.
Q.31. Find the value of ‘p’ for which the quadratic equation px(x – 2) + 6 = 0 has two equal real
roots.
3

3
Section-D
Section D consists of Long Answer (LA) type questions of 5 marks each.
Q.32. (A) A straight highway leads to the foot of a tower. A man standing on the top of the
75 m high tower observes two cars at angles of depression of 30º and 60º, which are
approaching the foot of the tower. If one car is exactly behind the other on the same
side of the tower, find the distance between the two cars. (use 3 = 1.73) 5
Or
(B) From the top of a 7 m high building, the angle of elevation of the top of a cable tower
is 60º and the angle of depression of its foot is 30º. Determine the height of the tower.5
Q.33. (A) D is a point on the side BC of a triangle ABC such that ∠ADC = ∠BAC, prove that
CA2 = CB.CD. 5
Or
(B) If AD & PM are medians of triangles ABC and PQR, respectively where
AB AD
DABC ~ DPQR, prove: PQ = PM . 5
Q.34. A student was asked to make a model shaped like a cylinder with two cones attached to
its ends by using a thin aluminium sheet. The diameter of the model is 3 cm and its total
length is 12 cm. If each cone has a height of 2 cm, find the volume of air contained in the
model.5

MATHEMATICS—2023 (SERIES: WX1YZ/4) n 139

Click here to access full book!


Q.35. The monthly expenditure on milk in 200 families of a Housing Society is given below:5
1000– 1500– 2000– 2500– 3000– 3500– 4000– 4500–
Monthly Exp. (in `)
1500 2000 2500 3000 3500 4000 4500 5000
No. of Families 24 40 33 x 30 22 16 7
Find the value of x and also, find the median and mean expenditure on milk.
Section-E
Section E consists of three Case Study Based questions of 4 marks each.
Q.36. Two schools ‘P’ and ‘Q’ decided to award prizes to their students for
two games of Hockey `x per student and Cricket `y per student. School
‘P’ decided to award a total of `9,500 for the two games to 5 and 4
students respectively; while school ‘Q’ decided to award `7,370 for the
two games to 4 and 3 students respectively.
Based on the above information, answer the following questions:
(i) Represent the given information algebraically (in terms of x and y).1
(ii) (a) What is the prize amount for hockey? 2
Or
(b) Prize amount on which game is more and by how much? 2
(iii) What will be the total prize amount if there are 2 students each from two games?1
Q.37. Jagdish has a field which is in the shape of a right angled

a s
trainagle AQC. He wants to leave a space in the form of a Y

iv D
square PQRS inside the field for growing wheat and the
remaining for growing vegetables (as shown in the figure). A

Sh
In the field, there is a pole marked as O. B
Based on the above information, answer the following
S R
questions:
(i) Taking O as origin, coordinates of P are (–200, 0) and
of Q are (200, 0). PQRS being a square, what are the
coordinates of R and S? 1 X¢ C P O Q X
(ii) (a) What is the area of square PQRS? 2 (–200, 0) (200, 0)
Or
(b) What is the length of diagonal PR in square PQRS? 2
(iii) If S divides CA in the ratio K : 1, what is the value of K, where point A is (200, 800)? 1
Q.38. Governing council of a local public development authority
I
of Dehradun decided to build an adventurous playground PARKING

on the top of a hill, which will have adequate space for 7 AREA
PLAYGROUND Units
parking.
After survey, it was decided to build rectangular II
playground, with a semi-circular area allotted for parking 14 Units

at one end of the playground. The length and breadth of


the rectangular playground are 14 units and 7 units, respectively. There are two quadrants
of radius 2 units on one side for special seats.
Based on the above information, answer the following questions:
(i) What is the total perimeter of the parking area? 1
(ii) (a) What is the total area of parking and the two quadrants? 2
Or
(b) What is the ratio of area of playground to the area of parking area? 2
(iii) Find the cost of fencing the playground and parking area at the rate of `2 per unit. 1

140 n SHIV
S-(140) n Shiv
DASDas
CBSE
CBSE
SECONDARY
Secondary Series
SERIES[X]
[X]

Click here to access full book!


SET II Code No. 30/4/2
Note: Except for the following questions, all the remaining questions have been asked in Set-I.
Section-A
Q.1. Which of the following is true for all values of q (0º ≤ q ≤ 90º)? 1
(a) cos2 q – sin2 q = 1 (b) cosec2 q – sec2 q = 1
2 2
(c) sec q – tan q = 1 (d) cot2 q – tan2 q = 1
Q.2. +If k + 2, 4k – 6 and 3k – 2 are three consecutive terms of an A.P., then the value of k is:1
(a) 3 (b) –3 (c) 4 (d) –4
Q.11. For the following distribution: 1
Class 0–5 5 – 10 10 – 15 15 – 20 20 – 25
Frequency 10 15 12 20 9
The sum of lower limits of median class and modal class is:
(a) 15 (b) 25 (c) 30 (d) 35
Q.12. The length of tangent drawn to a circle of radius 9 cm from a point 41 cm from the

Das
centre is: 1
(a) 40 cm (b) 9 cm (c) 41 cm (d) 50 cm

h i v
Q.13. In the given figure, O is the centre of the circle and PQ is R P

S
the chord. If the tangent PR at P makes an angle of 50º 50º
with PQ, then the measure of ∠POQ is:  1
O
(a) 50º (b) 40º
(c) 100º (d) 130º Q

Q.14. A bag contains 5 red balls and n green balls. If the probability of drawing a green ball
is three times that of a red ball, then the value of n is: 1
(a) 18 (b) 15 (c) 10 (d) 20
Q.18. The distance of the point (–1, 7) from x-axis is:  1
(a) –1 (b) 7 (c) 6 (d) 50
Section-B
5 1
Q.21.
(A) Evaluate: 2
+ 2
= cot 2 45º + 2 sin 2 90 º  2
cot 30 º sin 60 º
Or
(B) If q is an acute angle and sin q = cos q, find the value of tan2 q + cot2 q – 2.  2

Section-C
Q.29. (A) The sum of first 15 terms of an A.P. is 750 and its first term is 15. Find its 20th term. 3
Or
(B) Rohan repays his total loan of `1,18,000 by paying every month starting with the
first instalment of `1,000. If he increases the instalment by `100 every month, what
amount will be paid by him in the 30th instalment? What amount of loan has he paid
after 30th instalment? 3
Q.30. Prove that 3 is an irrational number.3

MATHEMATICS—2023 (SERIES: WX1YZ/4) n 141

Click here to access full book!


Section-D
Q.32. From a solid cylinder of height 20 cm and diameter 12 cm, a conical cavity of height 8 cm
and radius 6 cm is hallowed out. Find the total surface area of the remaining solid.5
C

Q.35. (A) In the given figure, ∠ADC = ∠BCA; prove that DACB ~ DADC.
Hence find BD if AC = 8 cm and AD = 3 cm. 5

Or A D B

(B) If a line is drawn parallel to one side of a triangle to intersect the other two sides in
distinct points, then prove that the other two sides are divided in the same ratio. 5

SET III Code No. 30/4/3


Note: Except for the following questions, all the remaining questions have been asked in Set-I & Set II.
Section-A
Q.7. The next term of the A.P. : 7 , 28 , 63 is :  1

s
(a) 70 (b) 80 (c) 97 (d) 112

D a
Q.8. (sec2
q – 1) (cosec2
q – 1) is equal to: 1

i v
(a) –1 (b) 1 (c) 0 (d) 2

S h
Q.15. For the following distribution:
Marks (Below) 10 20 30 40 50 60
Number of Students 3 12 27 57 75 80
The modal class is:
(a) 10-20 (b) 20-30 (c) 30-40 (d) 50-60
Q.16. In the given figure, PT is a tangent at T to the circle with
centre O. If ∠TPO = 25º, then x is equal to: 1 T
(a) 25º
(b) 65º P
25º O
(c) 90º x

(d) 115º
C
Q.17. In the given figure, PQ || AC. If BP = 4 cm, AP = 2.4 cm and Q
BQ = 5 cm, then length of BC is: 1
(a) 8 cm (b) 3 cm 5c
m
(c) 0.3 cm (d) 25 cm
3 2.4 cm 4 cm
A P B

Q.18. The points (–4, 0), (4, 0) and (0, 3) are the vertices of a: 1
(a) right triangle (b) isosceles triangle
(c) equilateral triangle (d) scalene triangle

142 n SHIV
S-(142) n Shiv
DASDas
CBSE
CBSE
SECONDARY
Secondary Series
SERIES[X]
[X]

Click here to access full book!


Section-B
(A) Evaluate: 2 sec2 q + 3 cosec2 q – 2 sin q cos q if q = 45º.2
Q.22.
Or
If sin q – cos q = 0, then find the value of sin4 q + cos4 q.
(B) 2

Section-C
Q.26. Find the value of ‘p’ for which one root of the quadratic equation px2 – 14x + 8 = 0 is 6 times the
other. 3
Q.27. From an external point, two tangents are drawn to a circle. Prove that the line joining the external
point to the centre of the circle bisects the angle between the two tangents.3

Section-D
(A) In a DPQR, N is a point on PR, such that QN ^ PR. If PN × NR = QN2, prove that ∠PQR = 90º.
Q.32.
 5
Or
A

(B)
In the given figure, DABC and DDBC are on the same base BC.
ar ( ∆ABC ) AO B
If AD intersects BC at O, prove that = . 5

s
ar ( ∆DBC ) DO

a
O C

hiv D
S
D

Q.33. A wooden article was made by scooping out a hemisphere from each end of a solid
cylinder, as shown in the figure. If the height of the cylinder is 10 cm and its base is of
radius 3.5 cm, find the total surface area of the article.5

________________

MATHEMATICS—2023 (SERIES: WX1YZ/4) n 143

Click here to access full book!


2023 (Series : WX1YZ/4)
9. (a) In ∆ABC, 
Set-I : 30/4/1 AB
A


⇒ tan q = BC
1. (a) Least Composite number is 4 = 22

6m
Least Prime number is 2 = 21 6

⇒ tan q =
HCF (4, 2) is 21 = 2 2 3 θ
B C
LCM (4, 2) is 22 = 4 ⇒ tan q =
3
2 3m

HCF 2 1 ⇒ tan q = tan 60º    ∴ q = 60º


∴ LCM = 4 = 2 i.e., 1 : 2

10. (c) As we know, sec2 q = 1 + tan2 q
2. (a) Given. x2 + 3x – 10 = 0 1 2
⇒ x2 + 5x – 2x – 10 = 0 ⇒ sec2 q = 1 + ⇒ sec2 q = cot θ2 + 1
cot 2 q cot θ
⇒ x(x + 5) –2(x + 5) = 0
cot 2 θ + 1 2
1 + cot θ
⇒ (x – 2) (x + 5) = 0
∴ sec q =
Zeroes are x – 2 = 0 or x + 5 = 0 cot 2 θ = cot θ
∴ x=2 or x = –5 11. (c) Two dice can be thrown as 6 × 6 = 36
3. (b) 6 = 1 × 6 = 1 6 ways.

s
Difference of 3 can be obtained as

iv D
24 = 4 × 6 = 2 6 (1, 4), (4, 1), (2, 5), (5, 2), (3, 6), (6, 3),
54 = 9 × 6 = 3 6 i.e., 6 ways

Sh
∴ Next term = 4 6 = 4 × 4 × 6 = 96 \ Required Probability = 6 or 1 .

36 6
4. (b) Distance between (–1, 7) from x-axis is
12. (b) Given. AC = 6 cm, BC = 5 cm,
7 (ordinate).
QR = 3 cm and PR = x
5. (c) Here, radius, r = diameter = d ...(i) DABC ~ DQPR ...[Given
2 2
1 2 ∴ AC = BC 
...[CPST

∴ Area of semi-circle = 2 pr QR PR
2 2 6 5
= 1 π  d  = pd  ...[From (i)
⇒ 3=x ⇒ 2x = 5
2 2 8
5
6. (a) As we know, \ x = 2 = 2.5 cm
3 Median = Mode + 2 Mean 13. (d) Distance from the Origin
∴ Mode = 3 Median – 2 Mean
7. (c) Given. 2x – 5y = 6 = (0 + 6)2 + (0 − 8)2
and 6x – 15y = 18 = (6 )2 + ( −8)2 = 36 + 64 = 100
b
Here, a1= 2= 1 ; 1 = −5 = 1 ; = 10 cm
a2 6 3 b2 −15 3
c1 6 1 14. (b)
c= =
18 3
Q
2
a b c 1 y
∴ a 1 = b1 = c 1 = 3 P
x
O
2 2 2
Hence Coincident.
8. (d) We have x2 – 1
∴ a = 1, b = 0, c = –1 ...(i)
Given. OQ is the radius and PQ is the
−b −0

∴ Sum of zeroes, a + b = a = 1 = 0 tangent

144 n SHIV
S-(144) n Shiv
DASDas
CBSE
CBSE
SECONDARY
Secondary Series
SERIES[X]
[X]

Click here to access full book!


∠OQP = 90º ...[Tangent is ^ to the (ii) Monday and Tuesday
radius through the point of contact (iii) Tuesday and Wednesday
In DPQO, x + y + ∠OQP = 180º (iv) Wednesday and Thursday,
 ...[Angles-sum-property of a D (v) Thursday and Friday
⇒ x + y + 90º = 180º ∴ x + y = 90º (vi) Friday and Saturday
(vii) Saturday and Sunday
15. (a) Construction: Join OA.
Out of 7 cases, 2 have Sundays
2
∴ P (53 Sundays) = 7
O A non-leap year is composed of 365
m
days. There are 52 weeks and 1 day in
4c 365 days.
30º
T
A As a result, there are 52 Sundays in
∠OAT = 90º ...[Tangent is ^ to the the a non-leap year. But one leftover day
radius through point of contact apart from those 52 weeks can be either
TA a Monday, Tuesday, Wednesday,
In rt. DOAT, cos 30º = OT Thursday, Friday, Saturday or a
3 TA Sunday.

⇒ = ⇒ 2 TA = 4 3
2 4 So, P(53 Sundays in a non-leap year)
1 5

s

∴ TA = 2 3 cm = 7≠7

D a
16. (b) In DABC, PQ || BC A (A) is true and (R) is False

hiv

4 cm

...[Given 20. (b) Assertion (A) : a, b, c are in AP.


8c
m

S
AP AQ P Q ∴ b–a=c–b

∴ =
PB QC

...[Since common difference is same
6 cm

...[Thales’ Theorem 2b = a + c is true.


4 8
B C Reason (R) : 1 + 3 + 5 + .......... + 2n – 1

=
6 QC ∴ Sn = n (a1 + an) = n (1 + 2n – 1) = n2,

2 2

2QC = 24 ⇒ QC = 12 cm
which is also True.
∴ AC = AQ + QC = 8 + 12 = 20 cm But it is not the correct explanation of
17. (d) We have 4x2 – 3x – 7 Assertion.
∴ a = 4, b = –3, c = –7 21. Let numbers be 2x and 3x.
−b −( −3) 3 Given. LCM (2x, 3x) is 6x
Here, a + b = a = 4
=
4
 ...(i)
⇒ 6x = 180 ⇒ x = 30
and ab = c = −7  ...(ii) ∴ HCF (2x, 3x) is x = 30
a 4
3 22. Given. 6x2 + 37x – (k – 2)
 1 1 β + α 4 −3
∴ Now,  α + β  = αβ = −7 = 7
∴ a = 6, b = 37, c = –(k – 2)
4 1
...[From (i) & (ii) Let zeroes be a and a .

constant term
18. (d) P(not an ace) = 1 – P(ace) = 1 – 4 = 12 Product of zeroes =
c
= a
52 13 coefficient of x 2
19. (c) Leap year = 366 days = 52 weeks and 2 ∴ a × 1 = −( k − 2 )
days α 6
Thus, a leap year always has 52 −k + 2
⇒1= 6
Sunday.
The remaining 2 days can be: ⇒ 6 = –k + 2
(i) Sunday and Monday ∴ k = 2 – 6 = –4

MATHEMATICS—2023 (SERIES: WX1YZ/4) n 145

Click here to access full book!


23. (A) Given. 2x2 – 9x + 4 = 0 ⇒ –14 +(n – 1)4 = 62
...[From (i)
Here a = 2, b = –9, c = 4
⇒ (n – 1)4 = 62 + 14
−b −( −9 ) 9 76

∴ Sum of roots = a = 2
=
2 ⇒ (n – 1) = 4 = 19
c 4 ⇒ n = 19 + 1 = 20
and Product of roots = a = 2 = 2
∴ Numbers of terms, n = 20
Or Or
(B) Given. –5=0 4x2 (B) a = 65, d = 61 – 65 = –4
Here a = 4, b = 0, c = –5 an < 0 ...[First negative term
As we know, D = b2 – 4ac ⇒ a + (n – 1)d < 0
= (0)2 – 4(4)(–5) ⇒ 65 + (n – 1) (–4) < 0
= 80 > 0 ⇒ 65 < 4 (n – 1)
∴ Roots are real and unequal. ⇒ 65 < n – 1 ⇒ 65 + 1 < n

4 4
24. S = {HH, HT, TH, TT} = 4
65 + 4
Atmost 1 head, HT, TH, TT i.e., 3
⇒ 4
<n \ n > 69 or 17.75
4
3 Hence First negative term is 18th term.
∴ P(atmost 1 head) = 4
27. See Q. 35, 2020 (Series: JBB/2). [Page 107
5 cos 2 60 º + 4 sec 2 30 º − tan 2 45º
25. (A)

s
28. Given. PA and PB are two
sin 2 30 º + cos 2 30 º
A

a
tangent from P which

iv D
2 2 2
1  2 
5   + 4   − ( 1)
2 make ∠APB. O 1 3 P
2  3

Sh
=  To Prove. ∠1 + ∠3 = 180º 4
1 ...[∵ sin2 q + cos2 q = 1
Proof. ∠2 = 90º ...(i) B
5 4 15 + 64 − 12 67 ∠4 = 90º ...(ii)
= 4 + 4 × 3 − 1 = 12
=
12
...[Tangent is ^ to the radius through
Or the point of contact
(B) Given. sin (A – B) = 0 In Quadrilateral OAPB,
⇒ sin (A – B) = sin 0 ∴ ∠1 + ∠2 + ∠3 + ∠4 = 360º
⇒ A–B=0  ...[Sum of all angles of a Quadrilateral is 360º
⇒ A = B ...(i) ∠1 + 90º + ∠3 + 90º = 360º ...[From (i) & (ii)
and 2 cos (A + B) – 1 = 0 ...[Given \ ∠1 + ∠3 = 360º – 180º
1 = 180º (Hence Proved)
⇒ cos (A + B) = 2

3
⇒ cos (A + B) = cos 60º
29. (A) L.H.S. sin A3 - 2 sin A
2 cos A - cos A
⇒ A + B = 60º

sin A(1 - 2 sin 2 A )
⇒ A + A = 60º
...[From (i) =
cos A( 2 cos 2 A - 1)
⇒ 2A = 60º   ⇒ A = 60 º = 30º
2 sin A(1 - 2 sin 2 A )
\ A = B = 30º = 
cos A[2(1 - sin 2 A) - 1]
26. (A) Given. 1st term, a = –14, a5 = 2
...[∵ cos2 q = 1 – sin2 q
and an = 62
sin A(1 - 2 sin 2 A )
a5 = a + 4d =
cos A( 2 - 2 sin 2 A - 1)
⇒ –14 + 4d = 2 ⇒ 4d = 16
sin A(1 - 2 sin 2 A ) sin A
\ d = 16 = 4
...(i) = =
cos A(1 - 2 sin 2 A ) cos A
4
Now, an = 62 = tan A = R.H.S. (Hence Proved)
⇒ a +(n – 1)d = 62 Or

146 n SHIV
S-(146) n Shiv
DASDas
CBSE
CBSE
SECONDARY
Secondary Series
SERIES[X]
[X]

Click here to access full book!


(B) L.H.S. = sec A (1 – sin A)(sec A + tan A) 1 75

⇒ = ⇒ BD = 75 3
 1   1 sin A  3 BD
=  (1 – sin A)  + 
 cosA   cos A cos A  ∴ Distance between two cars,
(1 − sin A )(1 + sin A ) 1 − sin 2 A CD = BD – BC
= =
cos 2 A cos 2 A = 75 3 – 25 3 = 50 3 m
2
=
cos A
= 1 = R.H.S. (Hence Proved) = 50(1.73) ...[∵ 3 = 1.73 (Given)
cos 2 A = 86.5 m
30. ∠OMB = 90°  ...[∵ Tangent is ⊥ to the
Or
radius through the point of contact. (B) Let AB be the E

In rt. ∆OMB, building and


EC be the cable
OM2 + MB2 = OB2 60° D
tower.
A
O 5 30°
...[Pythagoras’ theorem c5m

7 cm
In rt. ∆ABC,
3 cm

7 cm
(3)2 + (MB)2 = (5)2 3
30°
B AB
⇒ 9 + (MB)2 = 25
A
M tan 30° = B C
BC
⇒ MB2 = 25 – 9 = 16 ⇒ MB = +4 cm 1 7
Here, AM = MB = 4 cm
∴ = ⇒ BC = 7 3 m
3 BC
 ...[∵ ⊥ from the centre bisects the chord
∴ AB = AM + MB = 4 + 4 = 8 cm Here, AD = BC = 7 3 m

Das
DE
31. We have, px (x – 2) + 6 = 0 In rt. ∆ADE, tan 60° =
AD

v
⇒ px2 – 2px + 6 = 0, p ≠ 0

i
DE
∴ 3 =
⇒ DE = 21 m

h
For two equal roots, …[Given 7 3

S
1
∴ b2 – 4ac = 0 (a = p, b = –2p, c = 6)
∴ Height of cable tower, CE = CD + DE
⇒ (–2p)2 – 4 (p)(6) = 0
= 7 + 21  ...[∵ CD = AB = 7 m
⇒ 4p2 – 24p = 0 ⇒ 4p(p – 6) = 0 = 28 m
⇒ 4p = 0 or p–6=0 33. (A) Given: D is a point A
⇒ p=0 or p=6 on the side BC of a
But p ≠ 0 Standard form of a quad. eqn. ∆ABC such that
∴p=6 ... ax2 + bx + c = 0, a ≠ 0
∠ADC = ∠BAC.
32. (A) Let AB be the tower. To prove:
B C
C and D be the two cars. CA2 = CB.CD D

A Proof: In ∆ABC and ∆DAC,


30° ∠BAC = ∠ADC ...[Given
60°
∠C = ∠C ...[Common
75 m

∴ ∠ABC = ∠DAC ...[3rd angle


∴ ∆ABC ~ ∆DAC
30° 60°
D C B ∴ BC = AC 
... In similar ∆s Corresponding
sides are proportional
AC DC
AB
In rt. ∆ABC, tan 60° = ⇒ AC2 = CB.CD (Hence Proved)
BC
Or
3 75

∴ = ⇒ 3 BC = 75 (B) P
1 BC A

75 3 75 3

⇒ BC = × = = 25 3 m 1
3 3 3 B CQ
2
R
D
AB M
In rt. ∆ABD, tan 30° = Given: ∆ABC ~ ∆PQR
BD

MATHEMATICS—2023 (SERIES: WX1YZ/4) n 147

Click here to access full book!


∠1 = ∠2 n 200
...[In ~ ∆s corresponding angles are equal Here, = = 100
2 2
AB BC AC
= = ∴ Median Class is 2500 – 3000 Èl = 2500
PQ QR PR n
- c. f . Íc. f . = 97
... Í
... In similar ∆s Corresponding ∴ Median = l + 2 ×h f = 28
 sides are proportional. f Í
ÍÎ h = 500
In ∆ABD and ∆PQM,
AB 2BD ... ∵ AD and PM are medians
= 2500 + ( 100 - 97
28
¥ 500 )
=
PQ 2QM  ∵ BC = 2BD, QR = 2QM = 2500 +
3 × 500
= 2500 +
375
28 7
AB BD

⇒ = = 2500 + 53.57
PQ QM
= `2553.57 (approx.)
∠1 = ∠2 …[∵ ∆ABC ~ ∆PQR(Given)]
∴ ∆ABD ~ ∆PQM …[SAS Σfd ( −17500 )
Now, Mean = a + = 2750 +
AB BD AD Σf 200
Hence = =
PQ QM PM = 2750 – 87.50 = `2662.50
 ... In ~ ∆s corresponding 36. (i) School P : 5x + 4y = `9500...(i)
sides are proportional
School Q : 4x + 3y = `7370...(ii)
34. Let H be the height 12 cm (ii) (a) Multiplying (i) by 3 and (ii) by 4 and
subtracting, we get

s
of cylinder and h be H=8 cm

a
the height of cones 15x + 12y = 28,500

iv D
respectively. h=2 cm 2 cm 16x + 12y = 29,480
– – –

Sh
r be the common radius for cylinder as
well as for cones. –x = – 980
∴ H = 12 – 2 – 2 = 8 cm, h = 2 cm, r = 3 cm. ∴ x (Prize amount for Hockey) = `980
2 Or
Volume of air contained in the model (b) Putting the value of x in (i),
 = Vol. of cyl. + 2(Vol. of cone) 5(980) + 4y = 9500
(1 )
= πr2H + 2 3 πr 2h = πr2 H + 3 h ( 2
)

⇒ 4900 + 4y = 9500
⇒ 4y = 9500 – 4900
22 3 3  2  4600
= × × × 8 + × 2  ⇒ y=
= `1150
7 2 2  3  4
99 24 + 4
= 14 × 3 (
33
)
= 14 × 28 = 66 cm3
∴ Prize amount of circket is more than
Hockey Prize.
35.
Difference = y – x = `1150 – 980 = `170
Monthly x f c.f d= x – fd
Exp. (in `) 2750 (iii) Total prize would be = 2x + 2y
1000-1500 1250 24 24 –1500 –36000 = 2(x + y)
1500-2000 1750 40 64 –1000 –40000 = 2(980 + 1150) = 2(2130) = `4260
2000-2500 2250 33 97 –500 –16500
2500-3000 a = 2750 x 125 0 0 37. Y
3000-3500 3250 30 155 500 15000
3500-4000 3750 22 177 1000 22000 A (200,800)
4000-4500 4250 16 193 1500 24000
4500-5000 4750 7 Same 200 2000 14000 ) B
00
∑f = 200 ∑fd = 0, - 4
R
–17500 20 S
(–
Given. ∑ f = 200
24 + 40 + 33 + x + 30 + 22 + 16 + 7 = 200 X¢ C P O Q X
⇒ 172 + x = 200   \  x = 200 – 172 = 28 (–600, 0) (–200, 0) (200, 0)

148 n SHIV
S-(148) n Shiv
DASDas
CBSE
CBSE
SECONDARY
Secondary Series
SERIES[X]
[X]

Click here to access full book!


(i) Coordinates of R(200, 400) Or
S(–200, 400) Area of playgorund L×B
(ii) (a) Area of Square, PQRS = side × side (b) = 1
Area of parking πr 2
= 400 × 400 = 1,60,000 sq. units 2
Or 14 × 7 14 × 7 × 2 × 2
= 1 22 7 7 = = 56 : 11
(b) Diagonal of a square = side 2 × × × 11 × 7
= 400 2 units 2 7 2 2
Alternatively, (iii) Perimeter of playground & parking
By Using pythagoras’ theorem, we have = 7 + 14 + 14 + πr
PR2 = PQ2 + QR2 22 7
= 35 + × = 35 + 11 = 46 units
⇒ PR2 = (400)2 + (400)2 7 2
⇒ PR2 = 160000 + 160000 ∴ Total cost of fencing = 46 × `2 = `92
\ PR = 320000 = 4 × 4 × 2 × 100 × 100
= 4 × 100 2 = 400 2 units Set-II : 30/4/2
(iii) (–600, 0) K (–200, 400) 1 (200, 800)
C S A 1. (c) Identity : sec2 θ = 1 + tan2 θ
Coordinates of S = Coordinates of S ∴ sec2 θ – tan2 θ = 1
2. (a) k + 2, 4k – 6, 3k – 2 are in A.P.
 200 K − 600 800 K + 0 
∴  = (–200, 400)

Das
 , ∴ 4k – 6 – (k + 2) = 3k – 2 – (4k – 6)
K+1 K+1 
⇒ 4k – 6 – k – 2 = 3k – 2 – 4k + 6
 ...[Using section formula

v
 ...[Common difference is same

i
800 K
⇒ = 400
⇒ 3k – 8 = –k + 4 ⇒ 3k + k = 4 + 8

h
K+1

S
12
⇒ 800 K = 400 K + 400
⇒ 4k = 12 k=
∴ =3
4
⇒ 400 K = 400 ∴ K=1 11. (a) CIass fi cifi
38. I
PARKING
0–5 10 10
7 AREA 5–10 15 Max. 25
PLAYGROUND Units
10-15 12 37
II 15–20 20 57
20–25 9 same 66
∑f = 66
14 Units

7
(i) Radius, r = units n 66
2 Here, = = 33
∴ Total perimeter of the parking are 2 2
1 ∴ Median Class is 10 – 15.
= 7 + (2πr) = 7 + 22 × 7 = 18 units ⇒ Lower limit of Median Class = 10
2 7 2
(ii) (a) Let radius of quadrants be r1. Total Since Maximum frequency is 15
area of parking and two quadrants ∴ Modal Class 5 – 10.
1 2 1 2  r1 = 2 units (given)
Lower limit of Modal Class = 5
= πr + 2  πr1   ...  ∴ Required Sum = 10 + 5 = 15
2 4  7
r = units
1
 2 12. (a) A
= π [r2 + r21] 9 cm
2
2
1 22  7   P
= ×   + ( 2 )2  O 41 cm
2 7  2  
11  49  11  65  715
=  + 4 =   = units2 ∠OAP = 90° ...[∵ Tangent is ⊥ to the radius
7 4  7  4 28  through the point of contact

MATHEMATICS—2023 (SERIES: WX1YZ/4) n 149

Click here to access full book!


In ∆OAP, OA2 + AP2 = OP2 29. (A) Given. a = 15 and S15 = 750
...[Pythagoras’ theorem n
As we know, Sn = [2a + (n + 1)d]
⇒ 92 + AP2 = 412 2
⇒ AP2 = 412 – 92 ∴ 750 = 15 [2(15) + (15 – 1)d]

⇒ AP2 = 1681 – 81 = 1600 2
2
∴ AP = 1600 = 40 cm
⇒ 30 + 14d = 750 × ⇒ 14d = 100 – 30
15
70
13. (c) R P
⇒ d=
=5
14
50º
∴ a20 = a + 19d  ...[an = a + (n – 1)d
O
= 15 + 19(5) = 15 + 95 = 110
Q Or
(B) Amount paid in 30 instalments
Here, ∠OPR = 90° ...[∵ Tangent is ⊥ to the
radius through the point of contact
= 1000 + 1100 + 1200 + .....(30 terms)
n
∴ ∠RPQ + ∠OPQ = 90°
∴ Sn = [2a + (n – 1)d] a = 1000
2
⇒ ∠OPQ = 90° – 50° = 40° ... d = 100
30
In ∆POQ, OP = OQ...[Radii = (2000 + 29 × 100) n = 30
2
∠OPQ = ∠PQO = 40° ...[Angles opposite to
 equal sides are equal
= 15(2000 + 2900) = 15(4900)
= `73,500

s
In ∆POQ, ∠QPO + ∠PQO + ∠POQ = 180°

a
∴ Amount of loan he has to pay after the

iv D
 ...[Angle–sum–Property of a ∆
30th instalment = 1,18,000 – 73,500
⇒ 40° + 40° + ∠POQ = 180°
= `44,500

Sh
∴ ∠POQ = 180° – 80° = 100°
30. See Q. 14, 2016 (Delhi). [Page 23
14. (b) Total Balls = 5 + n
p(Green ball) = 3[p(Red ball)] ...[given 32. Removed
6 cm
n  5 

∴ = 3  ⇒ n = 15 8 cm
20 cm

5+n 5 + n
18. (b) The distance between (–1, 7) from
6 cm
x–axis is 7(ordinate).
5 1
21. (a) Given. + 2
Here r = 6 cm
2 sin 60°
cot 30° Height of the cylinder, H = 20 cm
– cot2 45° + 2 sin2 90°
Height of the cone, h = 8 cm
⇒ 5tan2 30° + cosec2 60° – (1)2 + 2(1)2

Slant height of cone, l = r 2 + h 2 = 6 2 + 82
2 2
 1   2  5 4

⇒ 5  +  –1+2 ⇒ + +1 r 2 + h 2== 6 2 + 82

 3  3 3 3
= 36 + 64 = 100 = 10 cm
5 + 4 + 3 12

⇒ = =4 ∴ Total Surface Area of the remaining
3 3
solid
Or
= ar. (Cylinder + Base + cone)
(b) Given. sin θ = cos θ
sin q = 2πrH + πr2 + πrl

⇒ =1 ⇒ tan θ = 1 = πr (2H + r + l)
cos q
1 1 22
Then, cot θ = = =1 = × 6 (2(20) + 6 + 10)
tan θ 1 7
Now, tan2 θ + cot2 θ – 2 = (1)2 + (1)2 – 2 22
= × 6 × 56 = 1056 cm2 0
=2–2=0 7

150 n SHIV
S-(150) n Shiv
DASDas
CBSE
CBSE
SECONDARY
Secondary Series
SERIES[X]
[X]

Click here to access full book!


35. (A) C
∴ ar(∆BDE) = ar(∆CDE) ...(iii)
 ...[∵ Ds on the same base and between the
 same parallel sides are equal in area
AD AE
From (i), (ii) and (iii) =
DB EC
Hence Proved.
A D B

Proof: In ∆ACB and ∆ADC, Set-III : 30/4/3


∠CAB = ∠DAC  ...[Common
7. (d) a1 = 7 × 1 = 1 7
∠BCA = ∠ADC ...[Given
∴ ∆ACB ∼ ∆ADC ...[AA similarly criterion a2 = 28 = 4 × 7 = 2 7
AC AB a3 = 63 = 9×7 = 3 7

∴ = ...[CPST
AD AC ∴ a4 =
4th term
8 AB

⇒ = = 4 7
3 8
64 = 4× 4×7
⇒ 3AB = 64 ⇒ AB =
3 = 112
Now, BD = AB – AD

Das
64 64 − 9 55 8. (b) We have (sec2 θ – 1)(cosec2 θ – 1)
=
–3= =
3 3 3 = tan2 θ.cot2 θ

i v
– 1

∴ BD = 18.3 or 18 cm

h
1
3

S
= tan2 θ × =1
tan 2 q
(B) Given. In ∆ABC, DE  BC
AD AE 15. (c) Marks c.f. C.I. f
To prove: =
DB EC Below 10 3 0–10 3
Const.: Draw EM ⊥ AD and DN ⊥ AE. Below 20 12 10–20 9
Join BE and CD. Below 30 27 20–30 15
Below 40 57 30–40 30 (Maximum)
A
Below 50 75 40–50 18
Below 60 80 50–60 5
M N

∴ Modal Class is 30 – 40.
D E
16. (d)
T

B C 25º O
P
Proof: In ∆ADE and ∆BDE, x

ar ( ∆ADE) = ½ ×AD × EM = AD ...(i)


ar ( ∆BDE) ½ × DB × EM DB
∠PTO = 90° ...[∵ Tangent is ⊥ to the radius
...[Area of ∆ = ½ × base × corresponding altitude  through the point of contact
In ∆ADE and ∆CDE,
Here, x = ∠TPO + ∠PTO 
ar ( ∆ADE) ½ ×AE × DN AE ...[Exterior angle theorem
= = ...(ii)
ar ( ∆CDE) ½ × EC × DN EC ∴ x = 25° + 90° = 115°

∵ DE  BC  ...[Given

MATHEMATICS—2023 (SERIES: WX1YZ/4) n 151

Click here to access full book!


17. (a) C 4 4
 1   1 
=   +  
Q  2   2
1 1 2 1
5c = + = = .
m 4 4 4 2

2.4 cm 4 cm 26. Given. px2 – 14x + 8 = 0


B
A P Let roots be α and 6α.
∠In ∆ABC, PQ || AC...[Given Here, a = p, b = –14, c = 8
BP BQ
∴ =  ...[Thales’ theorem −b c
PA QC Sum of roots = Product of roots =
a a
4 5 −( −14 ) 8
⇒ = ∴ α + 6α = α × (6α) =
2.4 QC p p
⇒ 4QC = 12 ⇒ QC = 3 cm 14 8
⇒ 7α = 6α2 =
Now, BC = BQ + QC p p
= 5 + 3 = 8 cm 2 2
⇒ α = ...(i)  2  8
p 6 p = ...[From (i)
18. (b) Let A(–4, 0), B(4, 0) and C(0, 3)   p
6× 4 8

s
AB = ( 4 + 4 )2 + (0 − 0 )2 = 64 ⇒ =

a
p2 p

iv D
= 8 units ⇒ 8p = 24

Sh
BC =
2 2
(0 − 4 ) + ( 3 − 0 ) = 16 + 9 ∴ p=3
25 = 5 units 27. A

AC = (0 + 4 )2 + ( 3 − 0 )2 = 16 + 9
O P
= 5 units
Here, BC = AC
∴ ∆ABC is an isosceles ∆. B

22. (A) 2 sec2 θ + 3 cosec2 θ – 2 sin θ cos θ To Prove: ∠APO = ∠BPO


= 2sec2 45° + 3 cosec2 45° – 2 sin 45° cos 45° Construction: Join OA and OB.
2 2  1   1  Proof: ∆OAP and ∆OBP,
= 2 ( 2 ) + 3 ( 2 ) – 2    
 2  2 OP = OP...[Common
= 2(2) + 3(2) – 1
OA = OB ...[Radii of same circle
PA = PB ...[Tangents drawn from an

=4+6–1=9
 external point are equal
Or
∆OAP ≅ ∆OBP ...[By SSS ≅ rule
(B) Given. sin θ – cos θ = 0
∴ ∠APO = ∠BPO
⇒ sin θ = cos θ
(Hence Proved) ...[By CPCT
sin q
⇒ =1 32. (A)
cos q Q
⇒ tan θ = tan 45°
2 3
∴ θ = 45° ...(i)
Now, sin4 θ + cos4 θ 6 5
4
P 1
(sin 45°)4 + (cos 45°)4 ...[From (i) N
R

152 n SHIV
S-(152) n Shiv
DASDas
CBSE
CBSE
SECONDARY
Secondary Series
SERIES[X]
[X]

Click here to access full book!


Proof: PN × NR = QN2  ...[Given ar( D ABC) AO
To prove: ar( D DBC) = DO
PN QN
=
QN NR Construction: Draw AM & DN ⊥ BC.
∠6 = ∠5...[QN ⊥ PR, each 90° Proof: In ∆AMO and ∆DNO,
∆PNQ ∼ ∆QNR  ...[By SAS similarity ∠AMO = ∠DNO ...[Each = 90°,

∠1 = ∠3 ...(i)  ∠AOM = ∠DON ...[Vertically opposite ∠s


...[CPST
∠2 = ∠4 ...(ii) ∴ ∆AMO ~ ∆DNO ...[AA corollary
AO AM
In ∆PQR, ∠1 + ∠PQR + ∠4 = 180° DO = DN ...(i) [In similar ∆s corresponding
 ...[Angle–sum–Property of a ∆  sides are proportional
ar( D ABC) ½×BC×AM
⇒ ∠3 + ∠2 + ∠3 + ∠2 = 180° = 
ar( D DBC) ½×BC× DN
...[From (i) & (ii)
...[\ ar ∆ = ½ base × altitude
⇒ 2∠2 + 2∠3 = 180°
ar( D ABC) AM
⇒ 2(∠2 + ∠3) = 180°
⇒ ar( D DBC) = DN

180° ar( D ABC) AO


∴ ∠PQR =
∴ =  ...[From (i)
2 ar( D DBC) DO
= 90° (Hence Proved)  (Hence Proved)

s
Or 33. Here, h = 10 cm and r = 3.5 cm

a
∴ Total Surface area of the article

D
32. (B) Given: DABC and DDBC are two ∆s on

hiv
the same base BC and AD intersects = C.S. area of cyl. + r = 3.5 cm

BC at O. 2(C.S. area of hemisphere)

S
A C = 2πrh + 2(2πr2)
= 2πr(h + 2r)

10 cm
N
= 2 × 22 × 35 (10 + 3.5 × 2)
M
O
7 10
B D = 22 × 17 = 374 cm2

___________________

MATHEMATICS—2023 (SERIES: WX1YZ/4) n 153

Click here to access full book!


Series : WX1YZ/6  Code No. 30/6/1
Candidates must write the Q.P. Code
Roll No.  on the title page of the answer-book.

• Please check that this question paper contains 8 printed pages.


• Code number given on the right hand side of the question paper should be
written on the title page of the answer-book by the candidate.
• Please check that this question paper contains 38 questions.
• Please write down the Serial Number of the question in the answer book before
attempting it.
• 15 minutes time has been allotted to read this question paper. The question
paper will be distributed at 10.15 a.m. From 10.15 a.m. to 10.30 a.m., the
candidates will read the question paper only and will not write any answer
on the answer-book during this period.

s
MATHEMATICS (Standard)–Theory
CBSE – 2023 (SERIES: WX1YZ/6)
iv D a
Time Allowed: 3 hours
GENERAL INSTRUCTIONS: Sh
Read the following instructions carefully and follow them:
(i) This question paper contains 38 questions. All questions are compulsory.
Maximum Marks: 80

(ii) This Question Paper is divided into FIVE Sections—Section A, B, C, D and E.


(iii) In Section-A Question number 1 to 18 are Multiple Choice Questions (MCQs) and Question
number 19 & 20 are Assertion-Reason based questions of 1 mark each.
(iv) In Section-B Question number 21 to 25 are Very Short-Answer-I (SA-I) type questions of 2 marks
each.
(v) In Section-C Question number 26 to 31 are Short Answer-II (SA-II) type questions carrying
3 marks each.
(vi) In Section-D Question number 32 to 35 are Long Answer (LA) type questions carrying 5 marks
each.
(vii) In Section-E Question number 36 to 38 are Case Study / Passage based integrated units of
assessment questions carrying 4 marks each. Internal choice is provided in 2 marks questions in
each case-study.
(viii) There is no overall choice. However, an internal choice has been provided in 2 questions in
Section-B, 2 questions in Section-C, 2 questions in Section-D and 3 questions in Section-E.
22
(ix) Draw neat figures wherever required. Take p = 7
wherever required if not stated.
(x) Use of calculator is NOT allowed.

154 n SHIV
S-(154) n Shiv
DASDas
CBSE
CBSE
SECONDARY
Secondary Series
SERIES[X]
[X]

Click here to access full book!


SET I Code No. 30/6/1
Section-A
Section A consists of 20 questions of 1 mark each.
32
Q.1. If p2 = 50 , then p is a/an 1


(a) whole number (b) integer (c) rational number (d) irrational number
Q.2. The distance of the point (–6, 8) from x-axis is 1
(a) 6 units (b) –6 units (c) 8 units (d) 10 units
Q.3. The number of quadratic polynomials having zeroes –5 and –3 is 1
(a) 1 (b) 2 (c) 3 (d) more than 3
Q.4. The point of intersection of the line represented by 3x – y = 3 and y-axis is given by 1
(a) (0, –3) (b) (0, 3) (c) (2, 0) (d) (–2, 0)
Q.5. The circumferences of two circles are in the ratio 4 : 5. What is the ratio of their radii?  1

(a) 16 : 25 (b) 25 : 16 (c) 2 : 5 (d) 4 : 5
Q.6. If a and b are the zeroes of the polynomial x2 – 1, then the value of (a + b) is  1

s
(a) 2 (b) 1 (c) –1 (d) 0

D a
2
Q.7. cos2 θ − 1 , in simplified form, is: 1

hiv
sin θ sin 2 θ
(a) tan2 q
(b) sec2 q (c) 1 (d) –1

S
Q.8. If ∆PQR ∼ ∆ABC; PQ = 6 cm , AB = 8 cm and the perimeter of ∆ABC is 36 cm, then the
perimeter of ∆PQR is  1
(a) 20.25 cm (b) 27 cm (c) 48 cm (d) 64 cm
Q.9. If the quadratic equation ax2 + bx + c = 0 has two real and equal roots, then ‘c’ is equal to
1
−b b −b 2 b2

(a) (b) 2 a (c) 4a (d) 4a
2a
Q.10. In the given figure, DE || BC. If AD = 3 cm, AB = 7 cm and
EC = 3 cm, then the length of AE is 1
(a) 2 cm
(b) 2.25 cm
(c) 3.5 cm
(d) 4 cm

Q.11. A bag contains 5 pink, 8 blue and 7 yellow balls. One ball is drawn at random from the
bag. What is the probability of getting neither a blue nor a pink ball?  1
1 2 7 13

(a) (b) 5 (c) 20 (d) 20
4
Q.12. The volume of a right circular cone whose area of the base is 156 cm2 and the vertical
height is 8 cm, is 1
(a) 2496 cm 3 (b) 1248 cm 3 (c) 1664 cm 3 (d) 416 cm3

MATHEMATICS—2023 (SERIES: WX1YZ/6) n 155

Click here to access full book!


Q.13. 3 chairs and 1 table cost `900, whereas 5 chairs and 3 tables cost `2,100. If the cost of 1
chair is ` x and the cost of 1 table is ` y, then the situation can be represented algebraically
as1
(a) 3x + y = 900, 3x + 5y = 2100
(b) x + 3y = 900, 3x + 5y = 2100
(c) 3x + y = 900, 5x + 3y = 2100
(d) x + 3y = 900, 5x + 3y = 2100
Q.14. In the given figure, PA and PB are tangents from external
point P to a circle with centre C and Q is any point on the
circle. Then the measure of ∠AQB is 1
(a) 62½º
(b) 125º
(c) 55º
(d) 90º
Q.15. A card is drawn at random from a well shuffled deck of 52 playing cards. 1
The probability of getting a face card is
1 3 4 1

(a) (b) 13 (c) 13 (d) 13
2
Q.16. If q is an acute angle of a right angled triangle, then which of the following equations is not

s
true?  1

iv D a
(a) sin q cot q = cos q (b) cos q tan q = sin q
(c) cosec2 q – cot2 q = 1 (d) tan2 q – sec2 q = 1

Sh
Q.17. If the zeroes of the quadratic polynomial x2 + (a + 1) x + b are 2 and –3, then  1
(a) a = –7, b = –1 (b) a = 5, b = –1
(c) a = 2, b = –6 (d) a = 0, b = –6
Q.18. If the sum of the first n terms of an A.P be 3n2 + n and its common difference is 6, then its
first term is  1
(a) 2 (b) 3 (c) 1 (d) 4
Assertion—Reason Based Questions: In question numbers 19 and 20, a statement of Assertion (A)
is followed by a statement of Reason (R).
Choose the correct option out of the following:
(a) Both A and R are true and R is the correct explanation of A.
(b) Both A and R are true and R is not the correct explanation of A.
(c) A is true but R is false.
(d) A is False but R is true.
Q.19. Assertion (A): If 5 + 7 is a root of a quadratic equation with rational co-efficients, then its
other root is 5 – 7 . 1
Reason (R): Square roots of a quadratic equation with rational co-efficients occur in conjugate
pairs.
Q.20. Assertion (A): For 0 < q ≤ 90º, cosec q – cot q and cosec q + cot q are reciprocal of each other.
 1
Reason (R): cosec2 q – cot2 q = 1.

156 n SHIV
S-(156) n Shiv
DASDas
CBSE
CBSE
SECONDARY
Secondary Series
SERIES[X]
[X]

Click here to access full book!


Section-B
Section B consists of Very Short Answer (VSA) type of questions of 2 marks each.
Q.21. (a) Show that 6n cannot end with digit 0 for any natural number ‘n’.  2
Or
(b) Find the HCF and LCM of 72 and 120.
Q.22. A line intersects y-axis and x-axis at point P and Q respectively. If R(2, 5) is the mid-point of
line segment PQ, then find the coordinates of P and Q. 2
Q.23. Find the length of the shadow on the ground of a pole of height 18 m when angle of elevation
6
q of the sun is such that tan q = 7 . 2

Q.24. In the given figure, PA is a tangent to the circle drawn from the A

external point P and PBC is the secant to the circle with BC as 130°
diameter. P C
B
If ∠AOC = 130º, then find the measure of ∠APB, where O is the O

centre of the circle.2

D a s

x+
hiv
Q.25. (a) In the given figure, ABC is a triangle in which DE || BC. If

2
AD = x, DB = x – 2, AE = x + 2 and EC = x – 1, then find the value

S
D E
of x. 2

x–
x–
Or

1
B C

D C


(b) Diagonals AC and BD of trapezium ABCD with AB || DC
O
intersect each other at point O. Show that OA = OB .
OC OD

A
Section-C B

Section C consists of Short Answer (SA) type of questions of 3 marks each.


Q.26. Find the ratio in which the line segment joining the points A(6, 3) and B(–2, –5) is divided
by x-axis. 3
Q.27. (a) Find the HCF and LCM of 26, 65 and 117, using prime factorisation.  3
Or
(b) Prove that 2 is an irrational number.
A

Q.28. In the given figure, E is a point on the side CB produced of an isosceles


triangle ABC with AB = AC. If AD ⊥ BC and EF ⊥ AC, then prove
F
that triangle ∆ABD ∼ ∆ECF.  3

E B D C

MATHEMATICS—2023 (SERIES: WX1YZ/6) n 157

Click here to access full book!


3
Q.29. (a) The sum of two numbers is 15. If the sum of their reciprocals is 10 , find the two numbers.
 3
Or
(b) If a and b are roots of the quadratic equation x2 –7x + 10 = 0, find the quadratic equation

whose roots are a2 and b2. 3
2
Q.30. Prove that : 1 + sec A = sin A . 3
sec A 1 − cos A
Q.31. In a circle of radius 21 cm, an arc subtends an angle of 60° at the centre. Find the area of the
sector formed by the arc. Also, find the length of the arc. 3
Section-D
Section D Consists of Long Answer (LA) type questions of 5 marks each.
P

Q.32. (a) Two tangents TP and TQ are drawn to a circle with centre O
T
from an external point T. Prove that ∠PTQ = 2 ∠OPQ. 5 O

a s
Or Q

h i v D
A

S
B P C

(b) A circle touches the side BC of a DABC at a point P and touches AB and
AC when produced at Q and R respectively.  Q R
1
Show that AQ = 2 (Perimeter of DABC).

Q.33. A solid is in the shape of a right-circular cone surmounted on a hemisphere, the radius of
each of them being 7 cm and the height of the cone is equal to its diameter. Find the volume
of the solid. 5
Q.34. (a) The ratio of the 11th term to the 18th term of an AP is 2 : 3. Find the ratio of the 5th term to
the 21st term. Also, find the ratio of the sum of first 5 terms to the sum of first 21 terms.
 5
Or
(b) If the sum of first 6 terms of an AP is 36 and that of the first 16 terms is 256, find the sum
of first 10 terms.
Q.35. 250 apples of a box were weighed and the distribution of masses of the apples is given in
the following table : 5
Mass (in grams) 80–100 100–120 120–140 140–160 160–180
Number of apples 20 60 70 x 60
(i) Find the value of x and the mean mass of the apples. 3
(ii) Find the modal mass of the apples. 2

158 n SHIV
S-(158) n Shiv
DASDas
CBSE
CBSE
SECONDARY
Secondary Series
SERIES[X]
[X]

Click here to access full book!


Section-E
3 Case Study Based Questions. Each question is of 4 marks.
Q.36. A coaching institute of Mathematics conducts
classes in two batches I and II and fees for rich and
poor children are different. In batch I, there are 20
poor and 5 rich children, whereas in batch II, there
are 5 poor and 25 rich children. The total monthly
collection of fees from batch I is `9000 and from
batch II is `26,000. Assume that each poor child
pays ` x per month and each rich child pays ` y per
month.
Based on the above information, answer the following questions :
(i) Represent the information given above in terms of x and y.1
(ii) Find the monthly fee paid by a poor child. 2
Or
Find the difference in the monthly fee paid by a poor child and a rich child. 2
(iii) If there are 10 poor and 20 rich children in batch III, what is the total monthly
collection of fees from batch III? 1

s
Q.37. Radio towers are used for transmitting a range of communication A

D a
services including radio and television. The tower will either act as an

hiv
antenna itself or support one or more antennas on its structure. On a
similar concept, a radio station tower was built in two Sections A and

S
B
B. Tower is supported by wires from a point O.
Distance between the base of the tower and point O is 36 cm. From
point O, the angle of elevation of the top of the Section B is 30º and the
angle of elevation of the top of Section A is 45°.
Based on the above information, answer the following questions : 45º
30º
(i) Find the length of the wire from the point O to the top of P 36 cm O
Section B. 1
(ii) Find the distance AB. 2
Or
Find the area of ∆OPB,
(iii) Find the height of the Section A from the base of the tower. 1
Q.38. “Eight Ball” is a game played on a pool table
with 15 balls numbered 1 to 15 and a “cue ball”
that is solid and white. Of the 15 numbered
balls, eight are solid (non-white) coloured and
numbered 1 to 8 and seven are striped balls
numbered 9 to 15.
The 15 numbered pool balls (no cue ball) are
placed in a large bowl and mixed, then one ball
is drawn out at random.
Based on the above information, answer the following questions :
(i) What is the probability that the drawn ball bears number 8? 1
(ii) What is the probability that the drawn ball bears an even number? 2

MATHEMATICS—2023 (SERIES: WX1YZ/6) n 159

Click here to access full book!


Or
What is the probability that the drawn ball bears a number, which is a multiple of 3?
(iii) What is the probability that the drawn ball is a solid coloured and bears an even
number?1

SET II Code No. 30/6/2

Note: Except for the following questions, all the remaining questions have been asked in Set-I.
Q.6. The LCM of smallest 2-digit number and smallest composite number is 1
(a) 12 (b) 4 (c) 20 (d) 40
Q.7. The distance of the point (–4, 3) from y-axis is 1
(a) –4 (b) 4 (c) 3 (d) 5
Q.8. If one zero of the polynomial x2 + 3x + k is 2, then the value of k.1
(a) –10 (b) 10 (c) 5 (d) –5
Q.16. A box contains 90 discs, numbered from 1 to 90. If one disc is drawn at random from the
box, the probability that it bears a prime number less than 23 is 1
7 1 4 9
(a) (b) 9 (c) 45 (d) 89

s
90

D a
Q.17. The coordinates of the point where the line 2y = 4x + 5 crosses x-axis is 1

v
( 4) ( ) ( −45 , 0) ( −25 , 0)
i
−5 5

h
(a) 0 , (b) 0 , 2 (c) (d)

S
Q.18. (cos4 A–sin4 A) on simplification, gives 1
(a) 2 sin2 A – 1 (b) 2 sin2 A + 1
(c) 2 cos2 A + 1 (d) 2 cos2 A – 1
Q.24. Find the points on the x-axis, each of which is at a distance of 10 units from the point
A(11, –8). 2

Q.26. In the given figure, AB and CD are diameters of a circle with centre O
perpendicular to each other. If OA = 7 cm, find the area of shaded
region. 3

Q.27. If sin q + cos q = p and sec q + cosec q = q, then prove that q(p2 – 1) = 2p. 3
Q.34. A solid is in the shape of a right-cirular cone surmounted on a hemisphere, the radius of
each of them being 3.5 cm and the total height of the solid is 9.5 cm. Find the volume of the
soild. 5
Q.35. (a) Find the sum of integers between 100 and 200 which are  5
(i) divisible by 9
(ii) not divisible by 9.
Or
(b) Solve the equation : –4 + (–1) + 2 + 5 + ..... + x = 437.

160 n SHIV
S-(160) n Shiv
DASDas
CBSE
CBSE
SECONDARY
Secondary Series
SERIES[X]
[X]

Click here to access full book!


SET III Code No. 30/6/3
Note: Except for the following questions, all the remaining questions have been asked in Set-I and Set II.
Q.1. The distance between the points (0, 5) and (–3, 1) is: 1
(a) 8 units (b) 5 units (c) 3 unit (d) 25 units
x
Q.2. If tan q = y , then cos q is equal to 1
x y x y
(a) (b) (c) (d)
x2 + y 2 x2 + y 2 x2 − y 2 x2 − y 2

Q.3. The zeroes of the polynomial 3x2 + 11x –4 are: 1


(a) 1 , – 4 (b) −1 , 4 (c) 1 , 4 (d) −1 , – 4
3 3 3 3
Q.16. If ‘p’ is a root of the quadratic equation x2 – (p + q) x + k = 0 then the value of ‘k’ is 1
(a) p (b) q (c) p + q (d) pq
Q.17. Cards bearing numbers 3 to 20 are placed in a bag and mixed thoroughly. A card is taken
out of the bag at random. What is the probability that the number on the card taken out is
an even number? 1
9 1 5 7

s
(a) 17 (b) 2 (c) 9 (d) 18

D a
Q.18. The condition for the system of linear equations ax + by = c; lx + my = n have a unique

hiv
solution is 1

S
(a) am ≠ bl (b) al ≠ bm (c) al = bm (d) am = bl
Q.21. Find the ratio in which the y-axis divides the line segment joining the points (5, –6) and
(–1,–4). 2
Q.26. Prove that (sin q + cos q) (tan q + cot q) = sec q + cosec q. 3
Q.27. (a) A natural number, when increased by 12, equals 160 times its reciprocal. Find the
number.  3
Or
(b) If one root of the quadratic equation x2 + 12x – k = 0 is thrice the other root, then find the
value of k.
Q.32. (a) The sum of first seven terms of an A.P. is 182. If its 4th term and the 17th term are in the
ratio 1 : 5, find the A.P.  5
Or
(b) The sum of first q terms of an A.P. is 63q – 3q2. If its pth term is – 60, find the value of p.
Also, find the 11th term of this A.P.
Q.33. (a) Prove that a parallelogram circumscribing a circle is a rhombus.  5
Or


(b) In the given figure, tangents PQ and PR are drawn to a
circle such that ∠RPQ = 30°. A chord RS is drawn parallel
to the tangent PQ. Find the measure of ∠RQS.
________________

MATHEMATICS—2023 (SERIES: WX1YZ/6) n 161

Click here to access full book!


2023 (Series : WX1YZ/6)

∆PQR ∼ ∆ABC  ...[Given
Set-I : 30/6/1 Peri. ( ∆PQR ) PQ
=  Ratio of the
32 16 Peri. ( ∆ABC ) AB perimeters of two
1. (c) We have, p2 = 50 = 25 ... similar ∆s is equal
Peri. ( ∆PQR ) 6
⇒ = to the ratio of their
4 36 8 corresponding sides
\ p = ± 5 (Rational Number)
3
2. (c) 8 units (ordinate) \ Peri. (∆PQR) = 4 × 36 = 27 cm
3. (d) Sum of zeroes, S = –5 + (–3) = –8 9. (d) D = 0  ...[Since Roots are real and equal
Product of zeroes, P = (–5) × (–3) = 15 b2
⇒ b2 – 4ac = 0 ⇒ b2 = 4ac    \ c =
\ Quadratic Polynomial = k (x2 – Sx + P) 4a
= k (x2 + 8x + 15)  10. (b) In ∆ABC, DE || BC ...[Given
 ...[Where k is non-zero real number AD AE
\ Infinitely many Quadratic Polynomials =
DB EC
...[Thales’ theorem
are possible. ⇒ 3 = AE ⇒ 4 AE = 9
4. (a) Given. 3x – y = 3 7−3 3
9

s
⇒ 0–y=3 (At Y-axis Put x = 0) \ AE = 4 = 2.25 cm

a
\ y = –3 \ Point = (x, y) = (0, –3)

D
11. (c) Total Balls = 5 + 8 + 7 = 20

v
5. (d) Let c1 and c2 be the circumferences

i
\ P (neither a blue nor a pink ball)

Sh
of two circles respectively.
Given.
c1 4
=
c2 5
2 πr 4
⇒ 2 πr1 = 5
2
( = 1–
Blue and Pink balls
Total balls )
\ r1 : r2 = 4 : 5 ( 20 )
= 1– 8 + 5 = 1– 13 = 7
20 20
6. (d) Given. x2 – 1 = 0 12. (d) Volume of cone
Here, a = 1, b = 0, c = –1 1
− b − coefficient of x = 3 × (Area of Base) × height
\ a + b = a = coefficient of x 2
1
= 3 × 156 × 8 = 416 cm3
−0
= 1 =0 13. (c) 3x + 1y = `900
cos 2 θ 1 5x + 3y = `2100
7. (d) 1st Method : −
14. (a) AC & BC are
sin 2 θ sin 2 θ
− sin 2 θ radii and PA and PB are
2 −(1 − cos 2 θ)
= cos θ2 − 1 = 2
= sin 2 θ = –1 tangents.
sin θ sin θ
cos 2 θ 1 ∠PAC = ∠PBC = 90º
2nd Method : −  ...[Tangent is ⊥ to the radius
sin 2 θ sin 2 θ
through the point of contact
= cot2 q – cosec2 q
= –(cosec2 q – cot2 q) = –1 In quad. PACB,
55º + 90º + ∠ACB + 90º = 360º
8. (b) P A
...[Angle-sum-property of a quadrilateral
⇒ ∠ACB = 360º – 235º = 125º
m

1 The angle subtended


6c

8c

Now, ∠AQB = 2 ∠ACB


by an arc at the centre
is double the angle
...
125 1 subtended by it at any
Q R B C = 2 = 62 ° point on the remaining
2
part of circle.

162 n SHIV
S-(162) n Shiv
DASDas
CBSE
CBSE
SECONDARY
Secondary Series
SERIES[X]
[X]

Click here to access full book!


15. (b) Face Cards = 4 + 4 + 4 = 12 x y
(jacks) (queens) (kings) ⇒ 2=2 ⇒ =5
2
12 3 ⇒ x = 4 ⇒ y = 10

\ P (Face Card) = 52 = 13
\ Coordinates of P (0, y) = (0, 10)
16. (d) Identity: sec2 q – tan2 q = 1
Coordinates of Q (x, 0) = (4, 0)
so, tan2 q – sec2 q ≠ 1
AB A
17. (d) Given. Zeroes of quadratic polynomial, 23. In a DACB, tan q = BC
x2 + (a + 1) x + b

18 m
Pole
a = 2 and b = –3 ⇒ 6 = 18 ...[ tan q = 76 ...[Given q
7 BC
Here, A = 1, B = a + 1, C = b Shadow
C B

−B C
⇒ 6 BC = 18 × 7
\ Sum of 0 = A \ Product of 0 = 18 × 7
A \ Length of shadow, BC = 6 = 21 m
−( a + 1) b 24.
⇒ 2 + (–3) = 1 ⇒ 2(–3) = 1
⇒ –1 = –(a + 1) \ b = –6
\ a=1–1=0
18. (d) Sn = 3n2 + n  ...[Given
Put n = 1, S1 = 3(1)2 + 1 = 3 + 1 = 4
\ First term = a = S1 = 4

s
Given. OA is the radius and PA is the tangent.

a
19. (a) Both (A) and (R) are true and (R) is

D
⇒ ∠PAO = 90º ...[Tangent is ⊥ to the radius

hiv
the correct explanation of (A).  through the point of contact
20. (a) Both (A) and (R) are true and (R) is \ ∠APO + ∠PAO = ∠AOC

S
the correct explanation of (A). ...[Exterior angle theorem
(cosec q – cot q) × (cosec q + cot q) ⇒ ∠APB + 90º = 130º
= cosec2 q – cot2 q \ ∠APB = 130º – 90º = 40º
= 1 R is True. 25. (a) In ∆ABC, DE || BC...[Given
21. (a) If the number 6n, for any n, were \ AD = AE ...[Thales’ theorem A
to end with the digit zero, then it would be BD EC
x x+2

x+
divisible by 5. That is, the prime factorisation ⇒ x−2 = x−1
x

2
of 6n will contain the prime number 5. This is
⇒ x (x – 1) = (x – 2) (x + 2) D E
not possible because Prime factorisation of
⇒ x2 – x = x2 – 4
2

x–
6n does not contain 5 as a factor.
x–

1
\ x = 4 units
6n = (2 × 3)n = 2n × 3n B C
Or
So, there is no natural number (n) for which
6n ends with the digit zero. (b) In DAOB and D C
DCOD,
4 3
Or
3 2 2 72 2 120 AB || CD ...[Given O
(b) 72 = 2 × 3
3
120 = 2 × 3 × 5 2 36 2 60 ∠1 = ∠3
1 2
2 18 2 30 ...[Alternate interior angles A B
\ HCF = 23 × 31 3 9 3 15 ∠2 = ∠4 
= 24 3 5 \ DAOB ∼ DCOD  ...[By AA corollary
\ LCM = 23 × 32 × 5 = 360
BO AO
22. Let P (0, y) and Q (x, 0). = … In similar Ds corresponding
DO CO  sides are proportional.
Mid point of PQ = R

0+x y + 0 \ OA = OB  (Hence proved)
, = (2, 5)  ...[mid-point theorem OC OD
 2 2 

MATHEMATICS—2023 (SERIES: WX1YZ/6) n 163

Click here to access full book!


26. (6, 3) k (x, 0) 1 (–2, –5) In DABD and DECF,
A C B
∠3 = ∠1  …[Each 90°

Let C (x, 0) be any point on x-axis. ∠4 = ∠2 …[From (i)


Let AC : CB = m2 : m1= k : 1 \ DABD ∼ DECF  …[AA similarity criterion
−5 k + 3 0
29. (a) Let two numbers be x and 15 – x.
Using section formula, k+1
=
1 According to question,
1 1 3 15 − x + x 3
⇒ –5k + 3 = 0 ⇒ –5k = –3 ⇒ x + 15 − x = 10 ⇒ x(15 − x ) = 10
3
\ k= 5 ⇒ 150 = 3x (15 – x) ⇒ 50 = 15x – x2
3 ⇒ x2 – 15x + 50 = 0
\ Required Ratio = k : 1 = 5 : 1 or = 3 : 5
⇒ x2 – 10x – 5x + 50 = 0
27. (a) Prime factorisation of 26 = 2 × 13
⇒ x(x – 10) –5 (x – 10) = 0
65 = 5 × 13
⇒ (x – 5) (x – 10) = 0
117 = 32 × 13
⇒ x – 5 = 0 or x – 10 = 0
\ HCF = 13
⇒ x = 5 or x = 10
\ LCM = 2 × 5 × 32 × 13 = 1170
When x = 5, numbers are 5, 10
Or
When x = 10, numbers are 10, 5
(b) Let us assume, to the contrary, that 2 Or
is rational. That is, we can find integers a (b) Given. x2 – 7x + 10 = 0

s
and b (¹ 0) that 2 = a .

a
b
a = 1, b = –7, c = 10

iv D
−b −( −7 )
Suppose a and b have a common factor Sum of roots, a + b = a = 1 =7 ...(i)

Sh
other than 1, then we can divide by the common c
factor and assume that a and b are coprime. Product of roots, ab = a = 10  ...(ii)
So, b 2 = a For required Polynomial,
Squaring on both sides, & rearranging, we Sum of roots, S = a2 + b2 = (a + b)2 – 2ab
get 2b2 = a2. Therefore, a2 is divisible by 2 and = (7)2 – 2(10) ...[From (i) and (ii)
a is also divisible by 2. So, we can write a = 2c = 49 – 20 = 29
for some integer c. Substituting for a, we get Product of roots, P = a2b2
2b2 = 4c2, that is b2 = 2c2. = (ab)2  ...[From (ii)
This means that b2 is divisible by 2 and so = (10)2 = 100
b is also divisible by 2. Therefore, a and b have at \ Required Quadratic Polynomial,
least 2 as a common factor. But this contradicts x2 – Sx + P = 0    \ x2 – 29x + 100 = 0
the fact that a and b are coprime. 1 + sec A
This contradiction has arisen because of our 30. L.H.S. sec A
incorrect assumption that 2 is rational. 1 1 cos A + 1
+
1 cos A = cos A
So we conclude that 2 is irrational. = 1 1
28. A cos A cos A
cos A + 1 cos A
= cos A ¥ 1
= cos A + 1  …(i)
F sin 2 A
1 R.H.S. 1 - cos A
4 3 2
(1 - cos 2 A) (1 + cos A)(1 - cos A)
E B D C = 1 - cos A
=
(1 - cos A)
In DABC,
AB = AC …[Given = 1 + cos A  …(ii)
\ ∠4 = ∠2  ...(i) [Angles opposite equal sides From (i) and (ii), L.H.S. = R.H.S.

164 n SHIV
S-(164) n Shiv
DASDas
CBSE
CBSE
SECONDARY
Secondary Series
SERIES[X]
[X]

Click here to access full book!


31. Here r = 21 cm, q = 60º Now, Perimeter of DABC = AB + BC + AC

m
q = AQ – BQ + BP + PC + AR – CR

c
21
Area of sector = 360º pr2 60º = AQ – BP + BP + CR + AQ – CR
60º 22  ...[Using (i), (ii) and (iii)
= 360º ¥ 7 × 21 × 21 \ Perimeter of DABC = 2AQ
= 231 cm2 AQ = 1 (Perimeter of DABC)
q 2
Length of an arc = (2pr) (Hence Proved)
360º
60º 22 33. Given. r = 7 cm, h = 2r = 14 cm
= 360º ¥ 2 ¥ 7 × 21 = 22 cm
Vol. of solid = Vol. of cone + Vol. of hemisphere
32. (a) Given. A circle with centre O, an 1 2 1
= 3 pr2h + 3 pr3 = 3 pr2 (h + 2r)
external point T and two tangents TP and TQ to
the circle, where P, Q are the points of contact. = 1 22
¥
3 7
× 7 × 7 (14 + 14)
22 ¥ 7 ¥ 28 4312 14 cm
P = 3 = 3
= 1437.3 cm3 7 cm
T θ O
a 2
34. (a); a11 = 3 ...[Given
18

Das
Q
⇒ a + 10 d 2 ...[∵a = a + (n – 1) d
= n
a + 17 d 3

i v
To Prove. ÐPTQ = 2ÐOPQ ⇒ 3a + 30d = 2a + 34d

h
Proof : Let ÐPTQ = q

S
Now, TP = TQ ⇒ 3a – 2a = 34d + –30d
∵ Lengths of tangents drawn from \ a = 4d  ...(i)
... 
an external Point to a ciircle are equal a a + 4d
Now, a 5 = a + 20 d
21
So, TPQ is an isosceles triangle
1 4d + 4d
\ ÐTPQ = ÐTQP = 2 (180º – q) = 4 d + 20 d  ...[From (i)

1 8d 1
= 90º – q...(i) = 24 d = 3
2
Also, ÐOPT = 90° \ a5 : a21 = 1 : 3
∵ Tangent at any point of a circle is ⊥
...  5
 to radius through the pt. of contact S5 (2 a + 4 d )
 2
Then, S = 21
So, ÐOPQ = ÐOPT – ÐTPQ = 90° – 90 º - 2 q ( 1
) 21
2
(2 a + 20 d )
n
 ...[From (i) È
 ÍÎ∵ Sn = 2 (2 a + (n - 1)d
1 1
ÐOPQ = 2 q Þ ÐOPQ = ÐPTQ
2 5 È 2 ¥ 4d + 4d ˘
\ ÐPTQ = 2ÐOPQ (Hence Proved) = 21 ¥ ÍÎ 2 ¥ 4 d + 20 d ˙˚ ...[From (i)
Or 5 12 d 5
A = 21 ¥ 28d = 49  
(b) Since the tangents from
an external point to a circle are \ S5 : S21 = 5 : 49
equal in length. P Or
B C
AQ = AR ∵ Tangent at any point Q (b) Given. S6 = 36
 a circle is ⊥ R
BQ = BP ... of
to radius through 6 n
CP = CR  the pt. of contact ⇒ 2 (2a + 5d) = 36 ... ÈÍ∵ Sn = (2 a + (n - 1)d
Î 2

MATHEMATICS—2023 (SERIES: WX1YZ/6) n 165

Click here to access full book!


⇒ 2a + 5d = 12 ⇒ 2a = 12 – 5d ...(i) 36. (i) A.T.Q. 20x + 5y = 9,000  ...(A)
and S16 = 256 ...[Given 5x + 25y = 26,000 ...(B)
1
16 (ii) Multiplying (A) by 1 and (B) by 5 ,
⇒ (2a + 15d) = 256
2 we have
256
⇒ 2a + 15d = 8 20x + 5y = 9,000 ...(A)
⇒ 12 – 5d + 15d = 32 ...[From (i)
x + 5y = 5,200  ...(C)
– – –
⇒ 10d = 32 – 12
20 19x = 3,800

\ d = 10 = 2
3800
From (i), 2a = 12 – 5(2) \ x = 19 = `200
⇒ 2a = 2 \ a = 1 From (C), 200 + 5y = 5200
n ⇒ 5y = 5200 – 200
As we know, Sn = 2 [2a + (n – 1)d]
5000
10 \ y=
\ S10 = 2 (2a + 9d) 5 = `1,000
\ Monthly fee paid by a poor child,
= 5 (2(1) + 9(2))
x = `200
= 5 (2 + 18) = 5 (20) = 100 Or
35. \ Required difference = y – x

s
= `(1,000 – 200) = `800 

a
Mass No. of

iv D
fi xi fi xi (iii) Total collection of Fees from Batch III
(in grams) apples
= 10x + 20y

Sh
80-100 20 20 90 1,800
100-120 60 f0 60 f0 110 6,600 = 10(200) + 20(1,000) ...[From Point (ii)
120-140 70 f1 70 f1 130 9,100 = 2,000 + 20,000 = `22,000
140-160 x 40 f2 150 6,000 37. (i) In rt. DOPB, A
160-180 60 60 170 10,200
Sfi = 250 250 Sfixi = 33,700 OP
cos 30º = OB
Total Apples = 250 ...[Given 3 36
⇒ = B
⇒ 20 + 60 + 70 + x + 60 = 250 2 OB
⇒ 210 + x = 250 72 3
⇒ OB = ¥
3 3
\ x = 250 – 210 = 40
45º
Σf i x i 72 3 30º
\ OB = = 24 3 cm P 36 cm O
(i) Mean Mass = Σf 3
i BP
33700
(ii) In rt. DOPB, tan 30º = PO
= 250
= 134.8 gms 1 BP
\
3
=
36
⇒ 3 BP = 36
(ii) Since Maximum Frequency is 70
36 3
So Modal class in 120 – 140 ⇒ BP = ¥
3 3
f1 - f0
\ Modal mass = l + 2 f 1 - f 0 - f 2 × h 36 3
⇒ BP = 3
= 12 3 cm
70 - 60 AP
= 120 + 140 - 60 - 40 × 20 l = 120 In rt. DAPO, tan 45º = PO
f0 = 60 AP
10 ¥ 20
…Here, f = 70
1
\ 1 = 36 ⇒ AP = 36 cm
= 120 + 40 f2 = 40
h = 20 \ AB = AP – BP = 36 – 12 3
= 120 + 5 = 125 gms = 12 ( 3 - 3 ) cm or 12 3 ( 3 - 1) cm

166 n SHIV
S-(166) n Shiv
DASDas
CBSE
CBSE
SECONDARY
Secondary Series
SERIES[X]
[X]

Click here to access full book!


Or 17. (c) Given. 2y = 4x + 5
1 ∴ 2(0) – 5 = 4x  ...[To cut x-axis, y = 0
ar. (DOPB) = 2 × Base × height
-5
1 ∴ x = 4 
= 2 × PO × PB


1
= ¥ 36 ¥ 12 3
∴ Point (x, y) = ( -54 , 0)
2
18. (d) Given. cos4 A – sin4 A
= 216 3 cm2
= (cos2 A)2 – (sin2 A)2
(iii) From (ii) above AP = 36 cm
= (cos2 A – sin2 A) (cos2 A + sin2 A)
Alternatively, AP = AB + BP
= [(cos2 A – (1– cos2 A)]. (1)
AP = AB + BP
= [(cos2 A – 1 + cos2 A]. 1 = 2 cos2 A – 1
= 12 3 ( 3 - 1) + 12 3
24. Let B(x, 0) be any point on x-axis.
= 36 - 12 3 + 12 3 = 36 cm
AB = 10 units ...[Given
38. Non-White balls = 8 (nos. 1 to 8) (AB)2 = 100  ...[Squaring both sides
Striped balls = 7 (nos. 9 to 15) ⇒ (x – 11)2 + (0 + 8)2 = 100
White solid cue ball = 1 ⇒ (x – 11)2 + 64 = 100
Total number of balls = 8 + 7 = 15 ⇒ (x – 11)2 = 100 – 64
1 ⇒ (x – 11)2 = 36
(i) P(number 8 ball) = 15 

s
⇒ x – 11 = ±6 ...[Taking Square-root on both sides

a
 ...[There is one eight number ball
⇒ x = 11 ±6

D
(ii) Even balls 2, 4, 6, 8, 10, 12, 14 i.e 7 balls

hiv
⇒ x = 11 + 6 or x = 11 – 6
7 \ x = 17 or x = 5  
∴ P(even number) = 15

S
∴ B = (17, 0) or (5, 0)
Or
“Multiples of 3” are 3,6,9,12,15 i.e 5 balls 26. Area of shaded region
5 1 = Area of Semi-circle – Ar. (DABC)
∴ P(multiple of 3) = 15 = 3 1 1
= 2 pr2 – 2 × AB × OC
(iii) Solid coloured and bear even number
balls are 2, 4, 6, 8 i.e 4 balls 1 22 1
= 2 ¥ 7 × 7 × 7 – 2 × 14 × 7
4
∴ Required probability = 15 È∵ AB = 7 + 7 = 14 cm ˘
 ÍÎ OC = r = 7 cm ˙˚

Set-II : 30/6/2 = 77 – 49 = 28 cm2


6. (c) Smallest 2 digit number = 10 = 2 × 5 27. We have, p = sin q + cos q
Smallest composite number = 4 = 22 q = sec q + cosec q
∴ LCM (10, 4) = 22 × 5 = 20 L.H.S. q (p2 – 1)
7. (b) Distance of (–4, 3) from y-axis is 4.
= (sec q + cosec q) [(sin q + cos q)2 – 1]
Note : Distance is not negative.
8. (a) Let P(x) = x2 + 3x + k = ( 1
+
1
cos q sin q
)
∴ x = 2 ...[∵2 is a zero of P(x)
(sin2 q + cos2 q + 2sin q cos q – 1)
2

( )
⇒ (2) + 3(2) + k = 0 sin q + cos q
⇒ 4+6+k=0 ∴ k = –10 = cos q sin q (2 sin q cos q)
16. (c) Let x be the Prime number less than 23 ...[∵sin2 q + cos2 q = 1
Such numbers are 2, 3, 5, 7, 11, 13, 17, 19 i.e 8 = 2 (sin q + cos q)
8 4 = 2p = R.H.S (Hence Proved)
∴ P(x) = 90 = 45

MATHEMATICS—2023 (SERIES: WX1YZ/6) n 167

Click here to access full book!


7 ⇒ n (3n – 11) = 874
34. r = 3.5 cm or 2 cm  ⇒ 3n2 – 11n – 874 = 0
⇒ 3n2 – 57n + 46n – 874 = 0

6 cm
Height of cone, h = 9.5 – 3.5

9.5 cm
= 6 cm 3.5 cm
⇒ 3n(n – 19) + 46 (n – 19) = 0

3.5 cm
∴ Vol. of solid = Vol. of cone ⇒ (n – 19) (3n + 46) = 0
+ Vol. of hemisphere ⇒ n – 19 = 0 or 3n + 46 = 0
1 2 2 -46
= pr h + 3 pr3 \ n = 19 or n = 3 (Not possible)
3
...[n must be a rational number
= 1 pr2 (h + 2r)
3 Now, x = an
1 22 7 7
( 7
= 3 ¥ 7 ¥ 2 ¥ 2 ¥ 6 + 2 ¥ 2 ) x=
a + (n – 1)d
= –4 + (19 – 1)(3) = –4 + 54 = 50
77 1001 - b ± b 2 - 4 ac
= 6 × 13 = 6 = 166.83 cm3 *Note : 2nd Method using x =
2a
35. (a) (i) Nos. divisible by 9 between 100 &
200 are Set-III : 30/6/3
108, 117, 126, .......... an 1. (b); Given. A (0, 5) and B (–3, 1)
Here a = 108, d = 117–108 = 9, an = 198
2 2
∴ a + (n – 1) d = an ∴ AB = ( - 3 - 0 ) + (1 - 5)

s
⇒ 108 + (n – 1) 9 = 198

a
...[Using distance formula

iv D
⇒ (n – 1)9 = 198 – 108
= 9 + 16 = 25 = 5 units
90

Sh
⇒ (n – 1)= 9 ∴ n = 11 x
2. (b); Given. tan q = y
n
Now, Sn = 2 (a1 + an) As we know, sec2 q = 1 + tan2 q
11 x2
S11 = 2 (108 + 198) sec2 q = 1 +
y2
11
= 2 × 306 = 11 × 153 = 1683 y 2 + x2
⇒ sec2 q =
(ii) Let S be the sum of integers between y2
100 and 200. x2 + y 2 x2 + y 2
∴ S = 101 + 102 + 103 + .......... + 199 ⇒ sec q = 2
=
y
y
Here a1 = 101, an = 199, 1 y
n = 199 – 101 + 1 = 99 \ cos q = secq = kx
x2 + y2 q
99 x P ky
∴ S = 2 (101 + 199) Alternatively, tan q = =
y B
99 In rt. D, H2 = P2 + B2
= 2 (300) = 99 × 150 = 14,850
= k2x2 + k2y2
Now, Sum not divisible by 9 ...[Pythagoras’ theorem
= 14850 – 1683 = 13167 ⇒ H2 = k2 (x2 + y2)
Or
(b) Here, a = –4, d = –1 – (–4) = 3, an = x ⇒ H = k x2 + y2
Given. Sn = 437 B ky y
n ∴ cos q = H = =
∴ [2a + (n – 1) (d)] = 437 k x2 + y 2 x2 + y2
2
n 3. (a) Given. 3x2 + 11x – 4
⇒ [2(–4) + (n – 1)(3)] = 437 ⇒ 3x2 + 12x – x – 4
2
⇒ n (–8 + 3n – 3) = 874 ⇒ 3x (x + 4) – 1(x + 4)

168 n SHIV
S-(168) n Shiv
DASDas
CBSE
CBSE
SECONDARY
Secondary Series
SERIES[X]
[X]

Click here to access full book!


⇒ (x + 4) (3x – 1) ⇒ (x – 8) (x + 20) = 0
∴ Zeroes are: x + 4 = 0 or 3x – 1 = 0 ⇒ x – 8 = 0 or x + 20 = 0
x = –4 or x = 1/3 ⇒ x = 8 or x = –20 (not possible)
16. (d) Given. x2 – (p + q) x + k = 0 Natural number must be positive
Since p is one of the root,  ...[Given ∴ Required number, x = 8.
∴ p2 – (p + q) p + k = 0 Or
⇒ p2 – p2 – pq + k = 0 ∴ k = pq (b) Given. x2 + 12x + k = 0. Let roots be a
17. (b) We have, 3, 4, 5 ......... 20 and 3a
Total cards = 18 Here a = 1, b = 12, c = –k
Even number cards are -b c
4,6,8,10,12,14,16,18,20 i.e., 9 Sum of roots = Product of roots = a
a
9 1 -12 -k
∴ P(even numbers) = 18 = 2 a + 3a = 1
a(3a) =
1
18. (a) Given. ax + by = c 4a = –12 3a2 = –k
lx + my = n ∴ a = –3 ...(i) 3(–3)2 = –k ...[From (i)
a b
For unique solution: 1 π 1 ∴ k = –27
a2 b2
a b 32. (a) S7 = 182
⇒ π ∴ am ≠ bl
l m
⇒ 7 (2a + 6d) = 182  ...[∵Sn =
n

Das
21. Let C(0,y) be any point on y-axis. 2 2 (2a + (n – 1) d)
(5, –6) k (0, y) 1 (–1, –4) 182
⇒ (a + 3d) = 7 ⇒ a + 3d = 26

i v
A C B

h
⇒ a = 26 – 3d  ...(i)

S
Let AC : CB = k : 1
a 1
Coordinates of C = Coordinates of C Now, a 4 = 5
( )
17
- k + 5 , - 4 k - 6 = (0, y) ...[Using formula
k+1 k+1 ⇒ a + 3d = 1  ...[∵an = a + (n – 1) d
-k + 5 a + 16 d 5
⇒ k + 1 = 0 ⇒ –k + 5 = 0
⇒ 26 - 3d + 3d = 1  ...[From (i)
∴ k=5 26 - 3d + 16 d 5
∴ Required Ratio = k : 1 = 5 : 1 26 1 ⇒ 26 + 13d = 130
⇒ =
26 + 13d 5
26. L.H.S = (sin q + cos q) (tan q + cot q)
= (sin q + cos q) ( sin q cos q
+
cos q sin q ) ⇒ 13d = 130 – 26
104
∴ d = 13 = 8
⇒ 13d = 104

Ê 2 2 ˆ
= (sin q + cos q) Á sin q + cos q ˜ From (i), a = 26 – 3 (8) = 26 – 24 = 2
Ë cos q sin q ¯
sin q + cos q
∴ AP is a, a + d, a + 2d, a + 3d, .......... 2, 10,
= sin q cos q  ...[∵sin2 q + cos2 q = 1 18, 26......
sin q cos q Or
= sin q cos q + sin q cos q (b) Given. Sq = 63q – 3q2
= sec q + cosec q = R.H.S (Hence Proved) Put q = 1, S1 = 63(1) – 3(1)2 = 63 – 3 = 60
27. (a) Let the number be x. Put q = 2, S2 = 63(2) – 3(2)2 = 126 – 12 = 114
A.T.Q., (x + 12) = 160 ()
1
x
a1 = S1 = 60
a2 = S2 – S1 = 114 – 60 = 54
x (x + 12) = 160

d = a2 – a1
⇒ x2 + 12x – 160 = 0 = 54 – 60 = –6
⇒ x2 + 20x – 8x – 160 = 0 Now, ap = –60

MATHEMATICS—2023 (SERIES: WX1YZ/6) n 169

Click here to access full book!


⇒ a + (p – 1)d = –60  ...[∵a = a1 = 60 (b)
⇒ 60 + (p – 1)(–6) = –60 S R
⇒ (p – 1)(–6) = –60 – 60 1 5
O 4
- 120
⇒ p – 1 = -6 2
3 30º
∴ p = 20 + 1 = 21 P
Q
Now, a11 = a + 10d
Given. ∠RPQ = 30º
a11 = 60 + 10(–6) = 60 – 60 = 0
To find the value of = ∠RQS
33. (a) D R C Proof: PQ = PR ...[Tangents drawn from an
 external point are equal
S ∴ In DPQR, ∠3 = ∠4 
Q
...[Angles opposite to equal sides

A B
∴ In DPQR, ∠3 + ∠4 + ∠30º = 180º ...(i)
P  ...[Angles sum Property of a D
Given. ABCD is a || gm. ⇒ ∠3 + ∠3 + 30º = 180º  ...[From (i)
To prove. ABCD is a rhombus.
⇒ 2∠3 = 180º – 30º = 150º
Proof. In || gm, opposite sides are equal
150
∴ AB = CD ⇒ ∠3 = 2 = 75º

a s
and AD = BC …(i)

iv D
∴ AP = AS ∴ ∠3 = ∠4 = 75º ...(ii)
È Tangents drawn
PB = BQ Í from an external RS || PQ  ...[Given

Sh
... Í
CR = CQ Í point are equal ∠5 = ∠3 = 75º  ...(iii)
ÍÎin length
h
DR = DS [Alternate Interior angles
By adding these tangents, PQ is a tangent and QR is a chord.
(AP + PB) + (CR + DR) ∠1 = ∠3 = 75º
= AS + BQ + CQ + DS ...[Angles in the alternate segments
⇒ AB + CD = (AS + DS) + (BQ + CQ) In DQRS, ∠1 + ∠2 + ∠5 = 180º 
⇒ AB + CD = AD + BC  ...[Angles-sum-Property of a D
⇒ AB + AB = BC + BC …[From (i)
⇒ 75º + ∠2 + 75º = 180º  ...[From (ii) and (iii)
⇒ 2AB = 2BC
⇒ ∠2 = 180º – 75º – 75º
⇒ AB = BC  …(ii)
∴ ∠2 or ∠RQS = 30º
From (i) and (ii), AB = BC = CD = DA
\ || gm ABCD is a rhombus.
Or
___________________

170 n SHIV
S-(170) n Shiv
DASDas
CBSE
CBSE
SECONDARY
Secondary Series
SERIES[X]
[X]

Click here to access full book!


Get the Book @ Amazon Special Discount
Access more papers with their solutions
(Sample Papers and Board Papers)

For more information feel free to contact us

Contact: 78400 40400, 81309 11616


Email: info@shivdas.in
Click here to access full book!

You might also like